You are on page 1of 134

Examination C

Cardiovascular/Pulmonary and Lymphatic I Examination

After an ST-elevated myocardial infarction 3 days ago, under what circumstances should a patient be ad-
ministered an exercise test?

Choices:
1. An exercise test should not be performed for this patient.
2. Heart rate equal to 70% age predicted maximum.
3. Heart rate equal to 85% age predicted maximum.
4. Only a symptom-limited exercise test should be performed.

Teaching Points

Correct Answer: 2
It is safe to have this patient exercise, as long as the intensity doesn't exceed 70% of the patient's age pre-
dicted heart maximum. This amount of activity doesn't place too much demand on the healing myocar-
dium.

Incorrect Choices:
An intensity higher than 70% can extend the zone of necrosis into the zone of injury and/or zone of is-
chemia. This will delay healing or, in the worst case, extend the MI. Using symptoms alone may allow the
patient to exercise at too high or too Iowan intensity.

Type of Reasoning: Inductive


This question requires one to draw on clinical knowledge and judgment of cardiac exercise guidelines for
patients with post-myocardial infarction in order to arrive at a correct conclusion. This is an inductive
reasoning skill where clinical judgment is paramount to arriving at a correct conclusion. For this case,
testing is safe if the heart rate is equal to 70% age predicted maximum. Review cardiac exercise testing
guidelines for patients with myocardial infarction if answered incorrectly.

Neuromuscular I Examination

A newborn is examined at birth using the APGAR test. Based on the following results, the neonatal therapist
suspects that neurological complications are likely with an APGAR of:

Choices:
1. 3 at 10 minutes.
2. 8 at 5 minutes.
3. 9 at 1 minute.
4. 8 at 1 minute.

743
744 Exami nation C

Teaching Points

Correct Answer: 1
The APGAR score is based on heart rate (HR), respiration, muscle tone, reflex irritability (grimace),
and color (appearance) . APGAR scores are routinely assigned at 1 and 5 minutes and occasionally at
10 minutes' postbirth.

Incorrect Choices:
Scores between 0 and 3 at 1 and 5 minutes are extremely low and indicative of the need for resuscitation.
Neurological complications are likely with extremely low APGAR scores, particularly at 10 minutes.

Type of Reasoning: Inductive


The test taker must make a determination of the meaning of the APGAR score as likely to result in neu-
rological complications. This requires one to rely on knowledge of what the scale measures and of score
ranges that indicate possible neurological complications. This requires clinical judgment coupled with
recall of the properties of the scale, which is an inductive reasoning skill. If this question was answered
incorrectly, refer to information on the APGAR Scale.

Nonsystem I Safety, Professional Responsibilities, Research

A PT requested that a physical therapy assistant (PTA) perform ultrasound (US) to the shoulder of a patient.
During the treatment session, the patient experienced an electrical shock. The PT would be responsible for
any injury to the patient if it was the result of:

Choices:
1. The patient touching the US device during treatment.
2. The PTA failing to use a ground fault interrupter (GPI).
3. The PT having instructed the PTA to use a device that had malfunctioned on the previous day.
4. Faulty circuitry.

Teaching Points
Correct Answer: 3
The PT in this case correctly delegated the US treatment to the PTA. Every individual (PT, PTA) is liable for
their own negligence; however, supervisors may assume liability of workers if they provide faulty supervi-
sion or inappropriate delegation of responsibilities (not evident in this case). PTs are liable for use of de-
fective equipment if they contributed to its malfunction or continued to have it used in treatment without
having it checked.

Incorrect Choices:
The institution may assume liability if the patient was harmed as a result of an environmental problem
such as faulty circuitry or leakage current that would cause the patient to be shocked if they touched the
US unit. The standard of practice is such that a GFI is used during administration of US, which would
make the PTA primarily liable if a GFI was not used. The patient assumes no liability in this scenario.

Type of Reasoning: Evaluation


This question requires one to determine the value of the statements made in the question and then to
determine the believability of these statements as applied to who should be assigned responsibility for the
injury. This requires evaluative reasoning skill, in which beliefs and values must be weighed to arrive at a
correct conclusion. In this situation, the PT should have taken the malfunctioning unit out of circulation
to prevent anyone else from using it, thus making the PT primarily responsible.
Examination C 745

Ca r diovascular/Pulmona ry and Lymphatic I Int erventions

A patient with coronary artery disease received inpatient cardiac rehabilitation after a mild myocardial
infarction (MI). The patient is now enrolled in an outpatient exercise class that utilizes intermittent train-
ing. The BEST initial spacing of exercise/ rest intervals to safely stress the aerobic system is:

Choices:
l. 1:l.
2.5 :l.
3 . 2:l.
4 . 1O:l.

Teaching Points

Correct Answer: 3
Presuming that the exercise goals for inpatient cardiac rehabilitation are met, an exercise/ rest ratio of 2:1
can be used with this patient to begin exercise in an outpatient setting in a safe manner.

Incorrect Choices:
An exercise/rest ratio of 1:1 is appropriate for an initial prescription for inpatient rehabilitation programs
with a goal of achieving a 2: 1 ratio. Ratios of 5:1 or 10: 1 are too stressful to begin outpatient rehabilita-
tion. A 5:1 ratio may be a goal for later exercise programming.

Type of Reasoning: Inductive


One must determine through clinical judgment which exercise/ rest interval is BEST, given the patient's
diagnosis and status. Questions that encourage clinical judgment to determine a best course of action
require inductive reasoning skills. If this question was answered incorrectly, review information related to
appropriate aerobic exercise and exercise/ rest ratios for patients with coronary artery disease .


Musculoskeletal I Evaluation, Diagnosis

The BEST INITIAL intervention to improve functional mobility in an individual with a stable humeral neck
fracture is:

Choices:
l. Active resistive range of motion (ROM).
2. Isometrics for all shoulder musculature.
3. Pendulum exercises.
4. Heat modalities.

Teaching Points

Correct Answer: 3
This individual will typically be immobilized with a sling for a period of 6 weeks. After 1 week, the sling
should be removed to have the patient perform pendulum exercises to prevent shoulder stiffness.

Incorrect Choices:
Resistive exercises including isometrics are not indicated during this early period. Heat modalities may be
effective in reducing pain but do not improve mobility.
746 Examination C

Type of Reasoning: Inference


One must understand the nature of a stable humeral neck fracture and appropriate interventions in order
to arrive at the correct conclusion. One must draw conclusions based on the patient's fracture status,
which necessitates inferential reasoning skill. If this question was answered incorrectly, review informa-
tion on appropriate exercises for stable humeral fractures.

Musculoskeletal I Interventions

A patient with unilateral spondylolysis at L4 is referred for physical therapy. The patient complains of
generalized lower back pain when standing longer than 1 hour. Interventions for the subacute phase should
include strengthening exercise for the:

Choices:
1. Abdominals working from neutral to full flexion.
2. Multifidi working from neutral to full extension.
3. Abdominals working from full extension to full flexion.
4. Multifidi working from full flexion back to neutral.

Teaching Points
Correct Answer: 4
Performing strengthening exercises to the multifidi from flexion to neutral will not stress the pars defect.

Incorrect Choices:
Abdominal strengthening will not provide the segmental stability needed with this condition. Lumbar
extension beyond neutral and rotation will tend to aggravate the condition in the early stages of rehabili-
tation.

Type of Reasoning: Analysis


One must understand the nature of unilateral spondylolysis, including symptomatology, in order to
choose the best intervention approach. One must analyze the information presented and determine
which intervention approach will provide the most benefit, while not stressing the pars defect. This
requires analytical reasoning skill, in which the test taker weighs the information presented to choose the
best solution .


Musculoskeletal I Examination

A patient has fixed forefoot varus malalignment. Possible compensatory motion( s) or posture (s) might
include:

Choices:
1. Genu recurvatum.
2. Excessive subtalar pronation.
3. Ipsilateral pelvic external rotation.
4. Hallux varus.
Examination C 747

Teaching Points

Correct Answer: 2
Possible compensatory motions or postures for forefoot varus malalignment include excessive midtarsal
or subtalar pronation or prolonged pronation; plantarflexed first ray; hallux valgus; or excessive tibial;
tibial and femoral; tibial, femoral, and pelvic internal rotation; and/or all with contralateral lumbar spine
rotation.

Incorrect Choices:
The other compensatory motions or deformities are NOT typical of this problem.

Type of Reasoning: Inference


This question provides the diagnosis, and the test taker must determine what compensatory motions or
postures are likely to occur. Questions that challenge one to determine possible symptoms from a given
diagnosis require inferential reasoning skill. If this question was answered incorrectly, refer to information
on forefoot varus malalignment.

In teg um e nta ry I Ev al u a t io n, D ia gn o s is

A patient presents with fingertips that are rounded and bulbous. The nail plate is more convex than nor-
mal. These changes are recognized by the therapist as indicative of:

Choices:
1. Inflammation of the proximal and lateral nail folds.
2. Trauma to the nail bed.
3. Psoriasis.
4. Chronic hypoxia from heart disease.

Teaching Points

Correct Answer: 4
Chronic hypoxia from heart disease or lung cancer and hepatic cirrhosis leads to clubbing of the fingers,
characterized by fingertips that are rounded and bulbous and a nail plate that is more convex than nor-
mal.

Incorrect Choices:
Inflammation to the proximal and lateral nail folds (paronychia) is characterized by red, swollen, and
tender folds . Trauma to the nail bed commonly results in white spots that grow out slowly with the nail.
Psoriasis can result in small pits in the nails along with a circumscribed yellowish tan discoloration (oil
spot lesion).

Type of Reasoning: Analysis


For this question, the test taker must analyze the symptoms and draw a conclusion regarding the likely
diagnosis. This requires analytical reasoning skill. For this case, the symptoms are indicative of chronic
hypoxia from heart disease. Review symptoms of chronic hypoxia if answered incorrectly.
748 Exam ination C

Cardiovascular/Pulmonary and Lymphatic I Evaluation, Diagnosis

The PT is examining a patient for right neck pain and spasms. Several inflamed submandibular nodes are
noted. The nodes are approximately 1.0 cm in size, tender, and erythematous. The patient has no known
history of cancer or metabolic diseases. In this case, the FIRST thing the therapist should do is:

Choices:
1. Question the patient regarding impact of neck pain and emotional distress.
2. Question the patient regarding any recent dental or throat infections.
3. Apply superficial heat and begin manual lymphatic drainage.
4. Have another therapist confirm the findings before implementing treatment.

Teaching Points

Correct Answer: 2
Typically, lymph nodes are not palpable, but they can become palpable in the presence of infection or
metastases. Past medical history is instrumental to identifying when follow up is required by the physician.
Recent infections (especially dental or pharyngeal) can make the lymph nodes swollen, tender, erythema-
tous, and/or firm. Medical referral is necessary.

Incorrect Choices:
Treatment (manual lymph draining/ superficial heat) should not be instituted. While questioning the pa-
tient regarding impact of neck pain and emotional distress is an important part of the examination, it will
not influence the decision-making regarding enlarged lymph nodes. Having another therapist confirm the
findings should not be necessary unless the therapist is inexperienced. In any case, treatment should not
be implemented.

Type of Reasoning: Evaluation


This question requires the test taker to consider the symptoms presented and then determine the best
course of action. Questions of this nature often require evaluative reasoning skill, as one must weigh in-
formation and its significance to determine a best course of action. For this situation, the FIRST action for
the therapist is to question the patient regarding any recent dental or throat infections. Refer to examina-
tion guidelines for lymph nodes if answered incorrectly.


Neuromuscular I Evaluation, Diagnosis

A PT is reviewing a medical record prior to examining a patient for the first time. The suspected diagnosis
is multiple sclerosis. On the neurologist's note, the therapist finds the following: deep tendon reflex (DTR);
right quadriceps is 2+, left quadriceps is 4+ . The therapist concludes that:

Choices:
1. The right DTR is normaL the left is abnormal.
2. Both DTRs are abnormal and indicative of upper motor neuron (UMN) syndrome.
3. The right DTR is exaggerated, the left is clearly abnormal.
4. Both DTRs are abnormal and indicative of hyporeflexia.
Examination C 749

Teaching Points

Correct Answer: 1
DTRs are graded on a 1-4 scale. Scores include 0 (no response); 1+ (present but depressed); 2+ (normal);
3+ (increased, brisker than average; possibly but not necessarily abnormal); and 4+ (very brisk, hyperac-
tive, with clonus, abnormal). In this case, the right DTR is normal; the left is abnormal, and consistent
with strong hypertonicity.

Incorrect Choices:
The other choices do not correctly interpret these findings.

Type of Reasoning: Inference


This question requires one to draw conclusions about the information presented, which is an inferential
reasoning skill. Here, the test taker must understand the meaning of 2+ and 4+ DTRs of the quadriceps in
order to determine that the 2+ is normal and 4+ is abnormal. If this question was answered incorrectly,
review information on measurement of DTRs.

Cardiovascular/Pulmonary and Lymphatic I Examination

A patient has an episode of syncope in the physical therapy clinic. The therapist attempts to rule out ortho-
static hypotension as the cause of the fainting. This is BEST done by:

Choices:
1. Checking HR and blood pressure (BP) in supine after 5 minutes of rest and then repeating in a semi-
Fowler position.
2. Palpating the carotid arteries and taking HR; using the supine position for BP measurements.
3. Checking HR and BP at rest and after 3 and 5 minutes of cycle ergometry exercise.
4. Checking resting BP and HR in supine and sitting, then repeating measurements after the patient stands
for 1 minute.

Teaching Points

Correct Answer: 4
Orthostatic hypotension is a fall in BP with elevation of position; thus responses to movements (HR and
BP) are tested from supine to sitting or sitting to standing. A small increase or no increase in HR upon
standing may suggest baroreflex impairment. An exaggerated increase in HR upon standing may indicate
volume depletion.

Incorrect Choices:
The other choices do not challenge the system with adequate change of position (supine to semi-Fowler
or remaining supine or sitting).

Type of Reasoning: Inductive


One must determine through clinical judgment the BEST method to rule out orthostatic hypotension.
This requires knowledge of the nature of orthostatic hypotension and how it can be evaluated, which
requires inductive reasoning skills. In this case, checking resting BP and HR in sitting and repeating after
1 minute of standing is the BEST way to rule out the diagnosis.
750 Examination C

Integumentary I Evaluation, Diagnosis

An inpatient with a grade III diabetic foot ulcer is referred for physical therapy. Panafil has been applied
to the necrotic tissue BID. The wound has no foul smell; however, the therapist notes a green tinge on the
dressing. In this case, the therapist should:

Choices:
1. Document the finding and contact the physician immediately.
2. Begin a trial of acetic acid to the wound.
3. Document the finding and continue with treatment.
4. Fit the patient with a total contact cast.

Teaching Points

Correct Answer: 3
In this case, the therapist should document the findings and continue with treatment. Panafil is a kerato-
lytic enzyme used for selective debridement. A greenish or yellowish exudate can be expected.

Incorrect Choices:
If the exudate was green and had a foul smell, Pseudomonas aeruginosa should be suspected. The physician
will most likely order a different topical agent. Acetic acid would not be the topical agent of choice. A total
contact cast can be used only after the wound is free of necrotic tissue.

Type of Reasoning: Evaluation


The test taker must determine the significance of the information presented and evaluate the best course
of action, given this information. This requires evaluative reasoning skill, in which one must assign value
to the patient's wound status and judge how to proceed.

Neuromuscular I Evaluation, Diagnosis

A PT receives a referral to examine the fall risk of an elderly patient with Parkinson's disease who lives alone
and has had two recent falls . The activity that represents the MOST common risk factor associated with falls
in the elderly is:

Choices:
1. Climbing on a step stool to reach overhead objects.
2. Walking with a roller walker with hand brakes.
3. Dressing while sitting on the edge of the bed.
4. Turning around and sitting down in a chair.

Teaching Points
Correct Answer: 4
Most falls occur during normal daily activity. Getting up or down from a bed or chair, turning, bending,
walking, and climbing/ descending stairs are all high-risk activities.

Incorrect Choices:
Only a small percentage of individuals fall during clearly hazardous activities (e.g., climbing the step stool) .
Proper use of an assistive device reduces the risk of falls. Sitting does not typically present a fall risk.
Exa mination C 751

Type of Reasoning: Evaluation


One must reason out the MOST common risk factor for an elderly person with a history of falls. This
requires the evaluation of the information presented and determining the believability of the statement,
which encourages evaluative reasoning skill. In this question, most common risk factors for falls in the
elderly are activities th at are done during the normal daily mobility activities .


Metabolic/Endocrine I Evaluation , Diagnosis

A patient with Addison's disease is referred for physical therapy following a hip fracture. The therapist rec-
ognizes the cardinal symptom of Addison's disease, which is:

Choices:
1. Asthenia.
2. Tremors.
3. Weight gain.
4. Diarrhea.

Teaching Po ints

Correct Answer: 1
The cardinal symptom of Addison's disease is asthenia. The weakness is slowly progressive and debilitat-
ing.

Incorrect Choices:
Other symptoms include anorexia, weight loss, nausea and vomiting, abdominal pain, and syncope.
Other listed choices are not characteristic of Addison's disease.

Type of Reasoning: Inference


One must determine the cardinal symptom for Addison's disease in order to arrive at a correct conclu-
sion. This requires one to determine whether it is likely to be true of a diagnosis, which is an inferential
reasoning skill. For this situation, the cardinal symptom is asthenia. Review signs and symptoms of Ad-
dison's disease if answered incorrectly.


Musculoskeletal I Interventions

A 2 month-old child with bilateral hip dislocations is being discharged from an acute pediatric facility. The
PT has developed a home exercise program and now needs to instruct the parents. The MOST important
item for the therapist to assess before instructing the parents is:

Choices:
1. The financial reimbursement plan.
2. Their degree of anxiety and attention .
3. Their level offormal education.
4. The home environment.
------------------- ---

752 Examination C

Teaching Points

Correct Answer: 2
A needs assessment should include a determination of the level of anxiety and ability to attend to the in-
structions given. If anxiety is high and the parents are unable to attend to the therapist's instructions, risk
of failure to perform the home exercises correctly is high.

Incorrect Choices:
Although the other factors may also be considered in the development of a home exercise plan, they do
not represent immediate priorities for instruction.

Type of Reasoning: Inference


This question requires one to draw conclusions about the diagnosis of the child and the MOST important
item to assess before instructing the parents. This requires inferential reasoning skill, in which the test
taker must make assumptions based on the information presented and determine the most appropriate
course of action. In this case, it is most important to determine the parents' degree of anxiety and atten-
tion .


Musculoskeletal I Examination

A PT receives a referral for a young child that had been swung around while being held from the wrists.
The referral reads, "functional disuse following nursemaid's elbow." The original injury consisted of:

Choices:
1. Superior subluxation of the ulna from the annular ligament.
2. Inferior subluxation of the ulna from the annular ligament.
3. Superior subluxation of the radial head from the annular ligament.
4. Inferior subluxation of the radial head from the annular ligament.

Teaching Points

Correct Answer: 4
The inferior subluxation of the radial head from the annular ligament typically occurs with a forceful
longitudinal pull of the forearm in a child. It is also known as "baby sitter's elbow. "

Incorrect Choices:
The ulna is not subluxed from the humerus with this condition, and the subluxation is not superior.

Type of Reasoning: Inference


In order to arrive at a correct conclusion, the test take must recall the likely symptoms for nursemaid's
elbow. This requires inferential reasoning skill. For this situation, the likely presentation for this injury
includes inferior subluxation of the radial head from the annular ligament. Review symptoms of nurse-
maid's elbow if answered incorrectly.

Examination C 753

Cardiovascular/Pulmonary and Lymphatic I Evaluation, Diagnosis

Which is a typical early clinical manifestation of cystic fibrosis (CF)?

Choices:
1. Excessive appetite and weight loss.
2. Increased FEV 1 (forced expiratory volume in 1 sec) during pulmonary function testing.
3. Frequent recurrent urinary tract infections.
4. Increase in secretions of the endocrine system.

Teaching Points

Correct Answer: 1
CF is an inherited disorder affecting the exocrine glands of the hepatic, digestive, and respiratory systems.
The patient with CF is prone to chronic bacterial airway infections and progressive loss of pulmonary
function from progressive obstructive lung disease. Early clinical manifestations include an inability to
gain weight despite an excessive appetite and adequate caloric intake.

Incorrect Choices:
The pulmonary function test results in a patient with CF can have a mixed picture of both obstructive and
restrictive disease components. It is typical to have a decrease in the FEV, value over time, not an increase.
There will also be recurrent infections in patients with CF; however, these infections will be in the airways
and lung parenchyma. Urinary tract infections, though quite possible, are no different in frequency than
in any patient or nonpatient population. Finally, CF is a disorder of the exocrine glands, meaning those
glands that excrete secretions. The endocrine system, including the hypothamus, pituitary, thyroid, etc.,
are not particularly involved in the disease of CF. The pancreas is both an exocrine and an endocrine
gland. CF does affect the pancreas by decreasing the bicarbonate secretions, reducing the effectiveness of
pancreatic enzymes and leading to pancreatic insufficiency.

Type of Reasoning: Inference


This question provides the diagnosis, and the test taker must determine the symptomatology that is
expected. CF is a multisystem disorder, making the possible clinical manifestations quite varied. This
question requires inferential reasoning skill, in which one infers knowledge of CF to arrive at the correct
conclusion for what is expected, a gastrointestinal manifestation of poor weight gain despite adequ ate
caloric intake.

Neuromuscular I Examination

An infant demonstrates that the asymmetrical tonic neck reflex (ATNR) is NOT obligatory when he/she can
turn the head:

Choices:
1. To both sides and open the hand.
2. To one side and look at the extended arm on that side.
3. To one side and bring the opposite hand to mouth.
4. And bring the hand to mouth on the same side.
754 Examination C

Teaching Points

Correct Answer: 4
ATNR causes extension of upper extremity on the side the head is turned toward. Bringing the hand to the
mouth would not be possible with an obligatory reflex.

Incorrect Choices:
The other choices do not correctly define actions that are limited with an obligatory ATNR. The ATNR is a
total upper extremity response and not limited to hand opening or simply looking at the hand.

Type of Reasoning: Analysis


In this scenario, if the infant can turn the head and bring the hand to mouth on the same side, the reflex
is no longer obligatory. Analytical reasoning skill is used in this question because the test taker must
determine the meaning of the information presented and analyze what it means in relation to a correct
solution.

Nonsystem I Safety, Professional Responsibilities, Research

An elderly patient with back pain is referred to physical therapy. The patient has pancreatic cancer and
moderate senile dementia and is unaware of the diagnosis or prognosis, at the family's request. The patient
asks the therapist what is the matter and how long will physical therapy take place. The therapist's BEST
response is:

Choices:
1. "I will tell the doctor to answer all of your questions."
2. "Ask the nurse practitioner for this information."
3. "I'll treat the symptoms and see how well you do."
4. To discuss the diagnosis and prognosis with the patient.

Teaching Points

Correct Answer: 3
The therapist should respond empathetically and use the moment to talk to the patient about the symp-
toms and physical therapy treatment.

Incorrect Choices:
The diagnosis and prognosis should not be discussed. The family has determined not to tell the patient.
Any change would have to be initiated by the family and falls under the domain of the physician. Telling
the patient to ask either the physician or the nurse practitioner is not appropriate.

Type of Reasoning: Evaluation


This question requires a value judgment related to a BEST course of action for the PT. Value judgments
encourage evaluative reasoning skill and are often challenging to answer because they can pose an ethical
dilemma. In sensitive situations such as this one, the test taker must determine the responsibility of the
therapist. In this scenario, the discussion of the patient's diagnosis and prognosis falls within the domain
of the physician.
Examination C 755

"
Musculoskeletal I Interventions

During surgery to remove an apical lung tumor, the long thoracic nerve was injured. Muscle testing of the
serratus anterior demonstrates its strength to be 3+/5. The BEST initial exercises are:

Choices:
1. Standing arm overhead lifts using hand weights.
2. Supine arm overhead lifts using weights.
3. Sitting arm overhead lifts using a pulley.
4. Standing wall push-ups.

Teaching Points

Correct Answer: 4
The long thoracic nerve supplies the serratus anterior muscle. With a muscle grade of 3+/5, the patient
can then begin functional strengthening using standing wall push-ups, with resistance provided by the
patient's own body.

Incorrect Choices:
The other exercises would not be optimal or used INITIALLY for strengthening a fair plus serratus anteri-
or. Performing overhead exercises with resistance (weights or pulleys) will overload the weakened serratus
anterior muscle, causing the patient to compensate and potentially develop inappropriate movement
patterns.

Type of Reasoning: Analysis


One must recall innervations for the long thoracic nerve in order to arrive at the correct conclusion. This
requires analysis of the information presented and interpretation of that information, which requires ana-
lytical reasoning skill. Knowledge of neuropathology is helpful to arrive at the correct conclusion. If this
question was answered incorrectly, review appropriate exercises for serratus anterior strengthening.

"
Musculoskeletal I Evaluation, Diagnosis

A PT examination reveals posterior superior iliac spine (PSIS) is low on the left; anterior superior iliac spine
(ASIS) is high on the left; standing flexion test shows that the left PSIS moves first and farthest superiorly;
Gillet's test demonstrates that the left PSIS moves inferiorly and laterally less than right; long sitting test
shows that the left malleolus moves short to long; and the sitting flexion test is negative. In light of these
findings, the therapist's diagnosis is:

Choices:
1. Left anterior rotated innominate.
2. Left posterior rotated innominate.
3. Left upslip.
4. Iliac inflare on the left.
756 Examination C

Teaching Points

Correct Answer: 2
A posterior rotated innominate is a unilateral iliosacral dysfunction. The question outlines positive physi-
cal findings, both static and dynamic, found with this dysfunction. One of these positive findings alone
does not confirm the diagnosis of left rotated posterior innominate.

Incorrect Choices:
The findings are opposite to what would be found with a left anterior ilial rotation. If the patient had an
upslip, both the PSIS and the ASIS on the left would be elevated. If an inflare was present, the left PSIS
would appear more lateral and the ASIS would appear more medial.

Type of Reasoning: Analysis


Questions provide the symptoms, and the therapist must determine the diagnosis; this requires the use
of analytical reasoning skill. This is because of the high need to determine what the cluster of symptoms
indicates in terms of deficits and functioning. For this patient, the symptoms indicate a left posterior
rotated innominate, which should trigger a review of iliosacral dysfunction if this question was answered
incorrectly.

Integumentary I Examination

A patient recovering from a burn on the back of the hand is referred to physical therapy for mobilization
exercises. The therapist observes a 14-cm irregular area that is thick and pink. The therapist documents this
finding as:

Choices:
1. Atrophic scarring.
2. Hypertrophic scarring.
3. An excoriation.
4. A scale.

Teaching Points

Correct Answer: 2
Hypertrophic scars are thick (raised) and pink (or red).

Incorrect Choices:
Atrophic scars are thin and white. Excoriation is an abrasion or scratch mark. A scale is a flake of exfoli-
ated epidermis (e.g., dandruff, psoriasis, dry skin) .

Type of Reasoning: Analysis


This question requires one to determine the clinical findings based on the observation of the patient's skin.
This necessitates analytical reasoning skill because one must analyze the signs presented in order to deter-
mine the likely clinical finding. In this case, the therapist's observations are consistent with hypertrophic
scarring. If this question was answered incorrectly, review signs and symptoms of hypertrophic scarring,
especially in burns.
Exa mination C 757

Nonsystem I Equipment, Devices, Therapeutic Modalities

A patient presents with partial- and full-thickness burns on the chest and neck regions. The therapist de-
cides to apply transcutaneous electrical nerve stimulation (TENS) before debridement to modulate pain.
Which TENS mode should provide the BEST relief?

Choices:
l. Conventional (high-rate) TENS.
2. Acupuncture-like (low-rate) TENS.
3. Modulated TENS.
4. Brief intense TENS.

Teaching Points

Correct Answer: 4
Brief intense TENS is used to provide rapid-onset, short-term relief during painful procedures. The pulse
rate and pulse duration are similar to conventional TENS; however, the current intensity is increased to
the patient's tolerance.

Incorrect Choices:
In this situation, intensity is the primary determinant of pain relief. Conventional TENS does not use as
high of an intensity as brief intense TENS and the application time is longer. Acupuncture-like TENS does
not give immediate relief of pain, because it has a long onset. Modulation is used to prevent accommoda-
tion, not to provide relief of pain.

Type of Reasoning: Deductive


One must recall the protocols for TENS use in order to determine the mode that would provide the BEST
relief of pain for this patient. This requires factual recall of information, which necessitates deductive
reasoning skill. In this case, brief intense TENS is BEST because it provides rapid-onset, short-term relief
during painful procedures.

Neuromuscular I Interventions

A patient recovering from traumatic brain injury (TBI) demonstrates difficulties in feeding resulting from
an unstable posture while sitting. The therapist determines that modification is necessary to ensure optimal
function. The FIRST body segment or segments that the therapist would align is/ are the:

Choices:
l. Pelvis .
2 . Head.
3. Lower extremities.
4. Trunk.
758 Examination C

Teaching Points

Correct Answer: 1
Modification of the pelvic position in a neutral posture promotes good lumbar and trunk alignment.
Many postural problems are correctable by aligning the pelvis first and achieving a stable base.

Incorrect Choices:
Modifying the position of the head, trunk or lower extremities may be necessary but only after achieving
a stable base.

Type of Reasoning: Inductive


The test taker must determine through clinical judgment the FIRST body segment( s) to align in order to
promote good trunk alignment. This requires inductive reasoning skill, in which knowledge of biome-
chanics and neuropathology is beneficial for arriving at the correct conclusion .


Neuromuscular I Interventions

A patient demonstrates some out-of-synergy movements in the right upper extremity indicative of stage 4
recovery after a left cerebrovascular accident (CVA). The proprioceptive neuromuscular facilitation (PNF)
pattern that represents the BEST choice to promote continued recovery of the right upper extremity is:

Choices:
1. Chop, reverse chop with right arm leading.
2. Bilateral symmetrical D1 thrust and reverse thrust.
3. Lift, reverse lift with right arm leading.
4. Bilateral symmetrical D2F and D2E, elbows straight.

Teaching Points

Correct Answer: 1
Both chop and reverse chop patterns move the affected arm out of synergy.

Incorrect Choices:
Thrust is an out-of-synergy pattern, and reverse thrust is in synergy. Lift is an out-of-synergy pattern, and
reverse lift is in synergy. The same is true for bilateral symmetrical D2F (out of synergy) and D2E (in syn-
ergy) .

Type of Reasoning: Inference


In order to arrive at the correct conclusion, one must recall the PNF patterns presented and then match
these patterns to the stage of recovery of the patient. The chop, reverse chop with right arm leading en-
courages out-of-synergy movement, which is most beneficial during stage 4 of recovery. If this question
was answered incorrectly, refer to PNF patterns and interventions for stage 4 of stroke recovery.

Exam ination C 759

Cardiovascular/Pulmonary and Lymphatic I Interventions

An elderly individual has limited endurance as a result of a sedentary lifestyle. There is no history of cardi-
orespiratory problems. After an exercise tolerance test, which was negative for coronary heart disease, the
BEST initial exercise prescription for this individual would be:

Choices:
1.30-50% HRmax.
2.60-90% HRm ax.
3.40-50% HRmax .
4. 35-50% ofV0 2max

Teaching Points
Correct Answer: 2
An appropriate initial exercise prescription for an asymptomatic elderly individual with general de-
conditioning is 60-90% of HRmax , which is equivalent to 50-85% ofV0 2max or 50-85% of HR reserve
(Karvonen's formula). This is within the established intensity guidelines for adults for aerobic exercise
training. Duration should be discontinuous, and exercise should be performed most days of the week
(ACSM Guidelines for Exercise Testing and Prescription, 8 'h ed. Philadelphia: Lippincott Williams & Wilkins) .

Incorrect Choices:
All other choices are too conservative. The exercise tolerance test has ruled out coronary heart disease.

Type of Reasoning: Analysis


One must analyze the information presented and then determine, given the patient's health status, the
MOST beneficial initial exercise prescription parameters. Analytical reasoning skill is utilized because
the test taker must weigh the information presented and recall the typical prescription parameters for an
asymptomatic individual.

Cardiovascular/Pulmonary and Lymphatic I Interventions

A patient is admitted to a coronary care unit with a mild myocardial infarction (MI). After 2 days, the pa-
tient is referred to physical therapy for inpatient cardiac rehabilitation. During an initial exercise session on
the unit, the patient reports chest pain, appears anxious, and wants to go back to bed to rest. The therapist's
BEST initial course of action is to terminate the exercise and:

Choices:
1. Sit the patient down and monitor vital signs carefully during the rest period.
2. Contact the attending physician immediately and continue to monitor vital signs.
3. Assist the patient back to bed and contact the charge nurse on the floor.
4. Assign the PTA to assist the patient back to bed and monitor vital signs carefully.
760 Examination C

Teaching Points

Correct Answer: 1
If the chest pain (angina) is exercise induced, this is an indication to terminate the exercise session (myo-
cardial demand is exceeding myocardial oxygen supply). Recovery is expected after a period of rest.

Incorrect Choices:
If the patient is still anxious, after the rest, it is reasonable to return the patient to the room and inform the
nurse. This should be done by the therapist personally in order to carefully monitor the patient's status.
The PTA should not be expected to evaluate chest pain or reach a determination about its significance. This
is not an emergency situation.

Type of Reasoning: Evaluation


One must make a judgment call for the BEST course of action for this patient, given the symptoms pre-
sented. Questions that require one to make value judgments necessitate evaluative reasoning skill and can
be challenging to answer, because the key to finding the correct solution goes beyond factual knowledge.
For this patient, it is best to have the patient sit down and monitor vital signs.

Integumentary I Evaluation, Diagnosis

A patient with a lO-year history of scleroderma is referred for physical therapy to improve functional status
and endurance. The patient was recently treated with corticosteroids for a bout of myositis. Examination
findings reveal limited ROM and fibrotic soft tissue along with hyperesthesia. The BEST choice for initial
intervention is:

Choices:
1. Soft tissue mobilization and stretching.
2. Closed-chain and modified aerobic step exercises.
3. Treadmill walking using body weight support at an intensity of 40% HRmax'
4. Active range of motion (AROM) exercises and walking in a therapeutic pool.

Teaching Points

Correct Answer: 4
Scleroderma (progressive systemic sclerosis) is a chronic, diffuse disease of connective tissues causing
fibrosis of skin, joints, blood vessels, and internal organs. Patients typically demonstrate symmetrical skin
thickening and visceral involvement of the gastrointestinal tract, lungs, heart, and kidneys along with hy-
persensitivity to touch. The BEST choice for initial intervention is to exercise in the pool. The warmth and
buoyancy of the water will enhance the patient's movements and decrease pain.

Incorrect Choices:
The other choices are too aggressive at this time and risk increasing the patient's pain, thereby limiting any
benefits in flexibility and endurance.

Type of Reasoning: Inductive


One must determine through clinical judgment the BEST initial intervention approach for this patient.
Knowledge of the nature of scleroderma and expected symptomatology is key to arriving at the correct
conclusion, which in this case is AROM exercises and walking in a therapeutic pool. If this question was
answered incorrectly, refer to information on intervention approaches for patients with scleroderma.

Examination C 761

Systems Interactions I Evaluation, Diagnosis

A patient is referred to physical therapy after a fall injury (fractured left hip with operative reduction, in-
ternal fixation [ORIF]) . Medical history reveals a diagnosis of stage 1 Alzheimer's disease. At this stage, the
behaviors the therapist would NOT expect to find are:

Choices:
1. Anxiety and irritability.
2. Profound communication deficits.
3. Memory loss.
4. Difficulty concentrating.

Teaching Points
Correct Answer: 2
Profound communication deficits (inability to speak), global deterioration of mental functions (delu-
sions, hallucinations, fragmented memory), agitation, and pacing (sundowning) are all characteristic of
late stages of this disease.

Incorrect Choices:
Early stage 1 Alzheimer's disease is characterized by memory loss, absentmindedness, anxiety and irritabil-
ity, difficulty concentrating, and occasional word-finding problems.

Type of Reasoning: Inference


One must infer information from the diagnosis presented and the stage of the disease provided. This
necessitates inferential reasoning skills, in which one must draw conclusions from the information
presented to choose the best solution of what is NOT expected in early stage l.

Neuromuscular I Interventions

A patient recovering from stroke with minimal lower extremity weakness and spasticity is able to walk with-
out an assistive device. The therapist observes that as the patient walks, there is noticeable hip hiking on the
affected side during swing phase. The BEST initial intervention is:

Choices:
1. Partial wall squats using a small ball held between the knees.
2. Standing and marching with manual pressure applied downward on the pelvis.
3. Marching while sitting on a therapy ball.
4. Bridging exercises progressing to sit-to-stand training.

Teaching Points

Correct Answer: 3
Hip hiking is a compensatory response for weak hip and knee flexors or extensor spasticity. Active exercises
for the hip and knee flexors (marching) is the most appropriate intervention.
762 Examination C

Incorrect Choices:
Downward manual pressure on the pelvis strengthens hip hikers. The other choices focus on strengthening
hip and knee extensors.

Type of Reasoning: Inductive


The test taker must utilize clinical judgment to determine first what the patient's deficits are (related to the
therapist's observation of the patient's gait) and then the BEST initial intervention for this patient. One
must evaluate the four possible intervention choices given to determine which intervention is BEST for
addressing the patient's deficits, which requires inductive reasoning skills. In this scenario, marching while
sitting on a ball will BEST address the patient's weak hip and knee flexors or extensor spasticity.

Neuromuscular I Interventions

The therapist is instructing a patient with traumatic brain injury (TBI) how to lock the brakes on a wheel-
chair. The patient is right-handed, and the right upper extremity is more affected than the left. To obtain
optimal results, the BEST motor learning training strategy is to:

Choices:
1. Guide the patient's right hand through the locking motions, then the left.
2. Verbally talk the patient through the locking motions, practicing with both hands simultaneously.
3. Have the patient practice locking the brakes first with the left hand and then with the right.
4. Have the patient practice brake locking using the left hand to assist the right.

Teaching Points

Correct Answer: 3
Using the motor learning strategy of transfer of training is best to use with this patient. Practice is per-
formed with the less affected extremity first and then progressed to use of the more affected extremity.

Incorrect Choices:
Guided movement (manual or verbal) represents a less active approach than the transfer of training ap-
proach. The more passive the performance, the slower the learning will take place. Bilateral tasks are more
difficult than performing the task with one limb at a time.

Type of Reasoning: Inductive


The test taker must determine the BEST training strategy for a patient with traumatic brain injury, which
necessitates inductive reasoning skill and, therefore, clinical judgment. For this patient, it is important
to have him/her practice locking the brakes first with his/her left hand and then his/ her right in order to
encourage effective transfer of training. Knowledge of effective motor learning strategies for patients with
brain injury is beneficial for arriving at the correct conclusion.
Examination C 763

Musculoskeletal I Examination

In neural tension testing, what position will BEST bias the tibial nerve?

Choices:
1. Straight leg raise with dorsiflexion and eversion.
2. Straight leg raise with dorsiflexion and inversion.
3. Straight leg raise with plantarflexion and eversion.
4. Straight leg raise with plantarflexion and inversion.

Teaching Points

Correct Answer: 1
A straight leg raise with dorsiflexion and eversion will best bias the tibial nerve. This is the optimal posi-
tion for neural tissue provocation of the tibial nerve. Neural tension techniques are used to decrease
adverse mechanical tension on the nerves. Peripheral nerves can often become trapped within the tissues,
where there can be a pull on the nerve with movement. This technique frees up the nerve so that it can
slide in its sheath.

Incorrect Choices:
Straight leg raise with dorsiflexion and inversion and straight leg raise with plantarflexion and inversion
will best bias the sural nerve and peroneal (fibular) nerve, respectively. Straight leg raise with plantarflex-
ion and eversion will not isolate a specific nerve.

Type of Reasoning: Inductive


This question requires the test taker to utilize clinical judgment in order to arrive at a correct conclusion.
In this case, the test taker must draw upon knowledge of neural tension testing to determine the best posi-
tion to bias the tibial nerve. This is an inductive reasoning skill. For this situation, the therapist should
position the extremity in a straight leg raise with dorsiflexion and eversion to bias the tibial nerve. Review
neural tension testing techniques if answered incorrectly.


Cardiovascular/Pulmonary and Lymphatic I Evaluation, Diagnosis

A therapist is examining a patient with chronic obstructive pulmonary disease (COPD) GOLD stage III.
What would be a clinical finding that the therapist would expect for this patient?

Choices:
1. Use of supplemental oxygen.
2. Weight gain.
3. Muscle wasting.
4. Decreased anteroposterior-to-lateral chest ratio.
764 Examination C

Teaching Points

Correct Answer: 3
Muscle wasting is a common manifestation of COPD . The cause of muscle wasting is not clear, but it is
not simply a malnutrition problem. The results of this muscle wasting are peripheral weakness, impaired
functional abilities, poor quality of life, and a poor prognostic sign.

Incorrect Choices:
Supplemental oxygen is typically found in patients in stage N of GOLD, not stage III. Weight loss is a com-
mon finding in patients with COPD, especially as the disease progresses. The energy demands of a person
with COPD are higher than those of a person without COPD for the same activity. A usual finding is that
persons with COPD are less active than their nondiseased counterparts. As the disease progresses, the person
with COPD cannot maintain his/her independence with any further decrease in his/her activity and weight
loss ensues. As lung destruction increases with worsening COPD, there are less elastic recoil properties of the
lung to pull the thorax back into what we recognize as a usual thoracic configuration. A barreled chest, or an
increased anteroposterior-to-lateral diameter of the chest, is a common finding in advanced, stage III COPD.

Type of Reasoning: Inference


One must infer from the symptoms presented what clinical finding would likely be found in a patient
with severe (stage III) COPD. This requires knowledge of pulmonary disorders and the typical findings
in order to determine, through the process of elimination, what finding is characteristic of that stage of
COPD . If this question was answered incorrectly, refer to typical symptoms of COPD.

Musculoskeletal I Examination

What muscle length test for the tensor fascia lata is recommended in a patient with decreased muscle length
of the rectus femoris?

Choices:
1. Ober test (knee flexed) .
2. Modified aber test (knee extended).
3. Ely's test.
4. FAIR (flexion, abduction, internal rotation) test.

Teaching Points

Correct Answer: 2
The modified aber test is performed with the knee extended, which allows the patient to attain the neces-
sary hip extension required to assess the tensor fascia lata.

Incorrect Choices:
The aber test is performed with the knee in flexion. Given the decreased muscle length of the rectus
femoris, the patient will not be able to attain the necessary hip extension required to assess the tensor
fascia lata. Ely's test identifies tightness of the rectus femoris, which is a given. The modified Ober test will
accommodate for the decreased muscle length of the rectus femoris. The FAIR test is performed to assess
for piriformis syndrome.

Type of Reasoning: Inductive


This question requires clinical judgment in order to determine the best muscle length test for the tensor
fascia lata tightness. Questions that draw on one's clinical knowledge and skill often require inductive
reasoning skills. For this case, the modified aber test is the best test, as it will accommodate for the de-
creased muscle length of the rectus femoris. Review muscle length tests, especially the modified aber test,
if answered incorrectly.
Examination C 765

Musculoskeletal I Interventions

During a postural screen for chronic shoulder pain, the therapist observes excessive internal rotation of the
shoulders and winging of the scapula during overhead motion. Intervention should focus on:

Choices:
1. Strengthening of pectoral muscles and stretching of upper trapezius.
2. Strengthening of upper trapezius and stretching of pectoral muscles.
3. Strengthening middle and lower trapezius and stretching of pectoral muscles.
4. Strengthening of rhomboids and stretching of upper trapezius.

Teaching Points

Correct Answer: 3
Abnormal posture that produces excessive internal rotation of the shoulders may result in chronic
shoulder impingement syndrome due to a loss of scapular stability with overhead motion. Shoulder pain
is likely to continue until a balance between anterior and posterior trunk musculature is achieved. The
anterior chest muscles (pectorals) are shortened and need stretching and posterior trunk muscles (middle
and lower trapezius) are stretched and need strengthening.

Incorrect Choices:
The pectoralis major and minor need to be stretched not strengthened. Stretching of the upper trapezius
will not change this condition.

Type of Reasoning: Analysis


Knowledge of shoulder kinesiology and pathology is beneficial to arriving at the correct conclusion for
this answer. An understanding that excessive internal rotation of the shoulders and scapular winging
could be caused by a weak middle and lower trapezius and shortened pectoral muscles results from a
solid understanding of shoulder pathology and kinesiology. Through analytical reasoning, the test taker
must determine which muscles are most likely to be affected and, therefore, produce these symptoms.

Integumentary I Interventions

Reexamination of a patient with a dermal ulcer over the coccyx reveals a wound exposing the deep fascia.
There is no necrotic tissue, exudate is minimal, and the borders of the ulcer are diffusely covered with
granulation tissue. Previous treatment has included daily whirlpoo l and wet-to-dry dressings with normal
saline. Based on the these findings, intervention should consist of:

Choices:
1. Continuation of the same treatments.
2. Whirlpool and hydrogel dressings.
3. Calcium alginate dressings.
4. Wound irrigation with pressures below 15 psi.

Teaching Points

Correct Answer: 4
Low-pressure wound irrigation helps to decrease colonization and prevent infection.
766 Examination C

Incorrect Choices:
If the ulcer is clean, whirlpool can damage incipient granulation tissue and should be discontinued. Wet-
to-dry dressings help remove necrotic tissue. Calcium alginate is used in the presence of heavy exudates,
which is not the case here. Hydrogel would be best because it is nonadherent, keeps wounds moist, and
protects granulation buds; however, a whirlpool is inappropriate.

Type of Reasoning: Inductive


One must utilize clinical judgment and knowledge of wound care approaches to determine the best inter-
vention approach for this patient, which requires inductive reasoning skill. For this patient, wound irriga-
tion with pressures below 15 psi is most appropriate for the type of wound described. If this question was
answered incorrectly, review information on wound care procedures for this type of wound .


Musculoskeletal I Evaluation, Diagnosis

The spinal defect shown in the diagram should be managed with avoidance of lumbar spinal:

Choices:
1. Flexion.
2. Rotation.
3. Extension.
4. Lateral flexion .

Teaching Points

Correct Answer: 3
With spondylolisthesis, there is typically an anterior slippage of one vertebra on the vertebra below. Be-
cause of the anterior shearing forces acting at the vertebra caused by the wedge shape of the vertebra and
gravity, spinal extension positions should be avoided.

Incorrect Choices:
Flexion, rotation, and lateral flexion will cause the bony structures to separate and will not cause any
negative compressive loads to the damaged structures.

Type of Reasoning: Analysis


One must analyze the spinal defect displayed in the picture and determine the diagnosis in order to deter-
mine what lumbar position should be avoided. This requires analytical reasoning skill, in which the precise
meaning of the spinal defect must be determined first, before determining the appropriate management of
this patient. If this question was answered incorrectly, refer to intervention approaches for spondylolisthesis.
Examination C 767

Neuromuscular I Evaluation, Diagnosis

A computer specialist is unable to work because of weakness and altered sensation in the dominant right
hand. The patient complains of pain and tingling of the thumb, index finger, long finger, and radial half
of the ring finger. The therapist observes thenar weakness and atrophy. Strength, reflexes, and sensation
are within normal limits throughout the remainder of the right upper extremity. The therapist determines
these signs and symptoms are characteristic of:

Choices:
1. Pronator teres syndrome.
2. Cervical root compression.
3. Ulnar nerve compression.
4. Carpal tunnel syndrome.

Teaching Points

Correct Answer: 4
The pattern of motor and sensory loss corresponds to the median nerve distribution in the hand. The
most likely cause is carpal tunnel syndrome.

Incorrect Choices:
Pronator teres syndrome (also a median nerve problem) produces similar deficits along with involvement
of the flexors of the wrist and fingers. Cervical root compression would also produce proximal deficits in
strength and sensation. Ulnar nerve compression (ulnar nerve palsy) would produce motor deficits of the
flexor carpi ulnaris and medial half of the flexor digitorum profundus, resulting in claw hand. Sensory
loss is to the ulnar side ofthe hand and/or arm.

Type of Reasoning: Analysis


This question provides the symptoms, and the test taker must determine the most likely diagnosis. This
necessitates analytical reasoning skill, in which one must determine the exact meaning of the symptoms
presented to arrive at the correct conclusion. If this question was answered incorrectly, review information
on symptoms of carpal tunnel syndrome.

Neuromuscular I Evaluation, Diagnosis

A patient presents with rapidly progressive symmetrical weakness that started in the distal lower extremity
muscles but now has ascended to include proximal trunk and upper extremity muscles. The motor seg-
ments of the lower cranial nerves are also showing impairment. The patient complains of abnormal sensa-
tions of tingling and burning of the affected extremities. Consciousness, cognition, and communication are
all normal. These signs and symptoms are characteristic of:

Choices:
1. Postpolio syndrome.
2. Guillain-Barre syndrome.
3. Multiple sclerosis (MS).
4. Amyotrophic lateral sclerosis (ALS) .
Examination C

Teaching Points

Correct Answer: 2
These signs and symptoms are characteristic of Guillain-Barre syndrome, a peripheral neuropathy in
which there is inflammation and demyelination of peripheral motor and sensory nerve fibers. Early in its
progression, either upper or lower motor signs may predominate.

Incorrect Choices:
In almost all cases, patients with ALS show features of both UMN and lower motor neuron (LMN) dys-
function. Postpolio syndrome is an LMN syndrome that does not present with sensory paresthesias and is
typically asymmetrical. MS will present with UMN signs: spasticity and hyperreflexia.

Type of Reasoning: Analysis


This question provides the symptoms, and the test taker must determine what the symptoms most likely
indicate. This requires analytical reasoning skill, in which one evaluates the exact meaning of the symp-
toms presented to arrive at the correct conclusion. If this question was answered incorrectly, review infor-
mation on symptoms of Guillain-Barre syndrome.

Cardiovascular/Pulmonary and Lymphatic I Interventions

A patient recovering from surgery for triple coronary artery bypass grafts is scheduled to begin a phase III
cardiac rehabilitation program. During the resistance training portion of the circuit training program, the
therapist instructs the patient to AVOID the Valsalva maneuver because:

Choices:
1. Heart rate (HR) and blood pressure are likely to be elevated.
2. Slowing of pulse and increased venous pressure are possible.
3. A cholinergic or vagal response can occur.
4. The decreased return of blood to the heart can lead to pitting edema.

Teaching Points

Correct Answer: 2
The Valsalva maneuver results from forcible exhalation with the glottis, nose, and mouth closed. It
increases intrathoracic pressures and causes slowing of the pulse, decreased return of blood to the heart,
and increased venous pressure. Although Valsalvas occur during normal daily activities (breath holding,
straining), they can be dangerous for patients with cardiovascular disease. On relaxation, blood rushes to
the heart and can overload the cardiac system, resulting in cardiac arrest.

Incorrect Choices:
HR is not elevated. A cholinergic or vagal response is the result of parasympathetic nervous system (PNS)
stimulation providing inhibitory control of the vagus nerve over HR and atrioventricular conduction. Pit-
ting edema is caused by long-term factors such as CHF.

Type of Reasoning: Evaluation


The test taker must determine the value and believability of the reasons presented to AVOID the Valsalva
maneuver, which encourages evaluative reasoning skill. Having knowledge of the Valsalva maneuver and
its effects on HR and venous pressure is important for arriving at the correct conclusion for this question.
If this question was answered incorrectly, review information on the Valsalva maneuver.

Examination C 769

Cardiovascular/Pulmonary and Lymphatic I Evaluation, Diagnosis

A patient experiences color changes in the skin during position changes of the foot. During elevation, pal-
lor develops. When the limb is then positioned in the seated hanging position, hyperemia develops. These
changes are indicative of:

Choices:
1. Chronic venous insufficiency.
2. Lymphedema.
3. Arterial insufficiency.
4. Deep vein thrombophlebitis.

Teaching Points

Correct Answer: 3
Arterial insufficiency can be determined by skin color changes during position changes of the foot
(termed rubor of dependency test).

Incorrect Choices:
Chronic venous insufficiency can be determined by the history, presence of aching calf pain with pro-
longed standing, a percussion test in standing, or Trendelenburg's test (retrograde filling test). With
chronic venous insufficiency, skin will be dark and cyanotic. Acute deep vein thrombophlebitis (DVT)
can be evident with aching calf pain, edema, and muscle tenderness. Lymphedema is evident with visual
inspection (i.e., swelling, decreased ROM) and volumetric measurements.

Type of Reasoning: Inference


One must infer the exact meaning of the symptoms presented and then draw conclusions about what the
diagnosis may be. Questions that inquire about a possible diagnosis for symptoms presented encourage
inferential reasoning skill. If this question was answered incorrectly, refer to information on arterial insuf-
ficiency of the lower extremity.

Neuromuscular I Examination

An elderly and frail adult is referred to physical therapy for an examination of balance. The patient has a
recent history offalls (two in the last 6 months). Based on knowledge of balance changes in the elderly and
scoring of standardized balance measures, the test data that BEST indicate increased fall risk are:

Choices:
1. Functional Reach of 7 inches.
2. Berg Balance score of 50.
3. Tinetti Performance Oriented Mobility Assessment (POMA) score of 27.
4. Timed Cet Up & Co (CUC) test result of 13 seconds.
770 Examination C

Teaching Points

Correct Answer: 1
All of these instruments can be used to examine functional balance and fall risk. A Functional Reach score
of < 10 is indicative of increased fall risk.

Incorrect Choices:
A POMA score of 27 out of a possible 28 is an excellent score. (Scores of < 19 indicate a high risk for falls,
whereas scores between 19 and 24 indicate moderate risk for falls). A Berg score of 50 out of a possible 56
points also indicates low fall risk. A Timed GUG score of < 20 seconds for the 3-meter walk and turn test
indicates low fall risk (scores > 30 second indicate increased risk).

Type of Reasoning: Deductive


This question requires one to recall factual information about the various tests described in the question
and then determine the meaning of the scores (which also encourages some analytical reasoning) . Factual
knowledge recall often encourages deductive reasoning skill, in which recall of protocols and guidelines
are pivotal to arriving at the correct conclusion. For this scenario, Functional Reach of 7 inches is the BEST
indicator of increased fall risk. If this question was answered incorrectly, review standardized balance tests .


Gast roi ntes t inal I Evaluation , Diagnosis

During an examination, a patient complains of right upper quadrant pain and tenderness. The PT percusses
over the costal margin at the point where the lateral border of the rectus muscle intersects with the costal
margin. The patient complains of acute pain and stops inspiratory effort. This is MOST likely indicative of:

Choices:
1. Irritation of the psoas muscle by an inflamed appendix.
2. Acute cholecystitis.
3. Peritoneal inflammation.
4. Hernia.

Teaching Points

Correct Answer: 2
Percussion for costovertebral tenderness that reveals a sharp increase in tenderness with a sudden stop in
inspiratory effort is a positive Murphy's sign and is indicative of acute cholecystitis.

Incorrect Choices:
An inflamed appendix results in pain in the right lower quadrant during left-sided pressure (positive Rov-
sing's sign) or right lower quadrant pain on quick withdrawal (referred rebound tenderness). A hernia pro-
duces a bulge in the abdominal wall (ventral hernias). Peritoneal inflammation presents with abdominal
pain on coughing or with light percussion. Rebound tenderness is also present.

Type of Reasoning: Inference


In order to arrive at a correct conclusion, one must have knowledge of the indications for percussion of
the costovertebral angle. This requires inferential reasoning skill, in which one draws conclusions based
upon evidence and facts . In this situation, percussion is indicated to reveal acute cholecystitis. If this ques-
tion was answered incorrectly, review indications for Murphy's percussion.

Exa mination C 771

Cardiovascular/Pulmonary and Lymphatic I Evaluation, Diagnosis

A 72-year-old patient is walking on a treadmill in the physical therapy department while vital signs and
pulse oximetry are being monitored. It is noted that the patient's arterial oxygen saturation (Sp02) drops
from 97% to 95%. In this case, it would be BEST to:

Choices:
°
1. Place a 100% 2 face mask on the patient for the remainder of the exercise session.
2. Place a 40% 0 2 face mask on the patient for the remainder of the exercise session.
°
3. Place 2 L of 2 by nasal cannula on the patient for the remainder of the exercise session.
4. Not use supplemental 0 2.

Teaching Points
Correct Answer: 4
A 72-year-old patient would likely h ave a resting Sp02 of 95% from the changes associated with aging
alone. There is no need to supplement oxygen in this case.

Incorrect Choices:
The guideline for using supplemental oxygen is a Sp02 of < 88% or a Pa0 2 of < 55 mm Hg. Therefore, the
use of oxygen in this scenario is not justified (all other choices). Supplemental 0 2 is by prescription only
unless it is an emergency.

Type of Reasoning: Inference


One must infer the exact meaning of the symptoms presented from this patient in order to make a deter-
mination of the BEST course of action. For this situation, the patient's response is within normal param-
eters and does not require supplemental oxygen. Knowledge of oxygen saturation guidelines is key to
arriving at the correct conclusion and should be reviewed if this question was answered incorrectly.

Nonsystem I Equipment, Devices, Therapeutic Modalities

The therapist is evaluating the needs of a young child who is diagnosed with myelodysplasia at the TIO
level. The therapist determines the MOST beneficial mobility device for this child to use in the school envi-
ronment is a:

Choices:
l. Bilateral hip-knee-ankle-foot orthosis (HKAFO).
2. Lightweight wheelchair.
3. Bilateral knee-ankle-foot orthosis (KAFO).
4. Parapodium.

Teaching Points

Correct Answer: 2
The lightweight wheelchair is the MOST beneficial choice for this child. It provides effective and efficient
mobility.
772 Examination C

Incorrect Choices:
Ambulation with orthotic devices at this level lesion requires too much energy and time to be functional.
The parapodium permits standing but does not allow for sufficient mobility for the entire school day.

Type of Reasoning: Inductive


One must utilize clinical judgment to determine the MOST beneficial mobility device for this child. In
order to arrive at the correct conclusion, the test taker must have a thorough understanding of the func-
tional abilities of a child with myelodysplasia at the no level. In this case, a lightweight wheelchair is
most appropriate because ambulation requires too much energy and time. If this question was answered
incorrectly, review information on thoracic myelodysplasia.

Musculoskeletal I Interventions

A patient has limited right rotation caused by left thoracic facet joint capsular tightness at T6-7 . The arthro-
kinematic glide that would MOST effectively improve right rotation in sitting is:

Choices:
1. Superior and anterior glide on the right T6 transverse process.
2. Superior and anterior glide on the left T6 transverse process.
3. Superior and anterior glide on the right T7 transverse process.
4. Superior and anterior glide on the left T7 transverse process.

Teaching Points

Correct Answer: 2
Because the left thoracic facet joint capsule is restricting movement, motion that would stretch the capsule
would facilitate improved right rotation. With right rotation, the left superior facets move upward (open-
ing the joint and stretching the capsule) and the right facets move downward (closing the joint and putting
the capsule on relative slack) .

Incorrect Choices:
Providing a superior and anterior glide on the right T6 transverse process would improve left rotation. Pro-
viding a superior and anterior glide on the left T7 transverse process would improve left rotation at T6-7.
Providing a superior and anterior glide on the right T7 transverse process would improve left rotation
between T7 and T8.

Type of Reasoning: Inductive


One must utilize clinical judgment to determine the MOST effective intervention approach for facilitating
improved right rotation. In order to arrive at the correct conclusion, the test taker must have a thorough
understanding of normal facet joint capsule motion with rotation. For this patient, a superior and anterior
glide to the left T6 transverse process will MOST effectively stretch the joint capsule and improve right
rotation.
Examination C 773

"
Musculoskeletal I Examination

Electromyogram (EMG) activity in the lower extremities during quiet standing is fairly continuous in the:

Choices:
1. Anterior tibialis and peroneals.
2. Posterior tibialis and intrinsic foot muscles.
3. Quadriceps femoris and anterior tibialis.
4. Soleus and gastrocnemius.

Teaching Points

Correct Answer: 4
The soleus and gastrocnemius muscles oppose the dorsiflexion moment that exists at the ankle as a result
of the line of gravity, which falls slightly anterior to the lateral malleolus. This fairly continuous activity is
crucial for maintaining balance during quiet standing.

Incorrect Choices:
The anterior tibialis, peroneals, and tibialis posterior muscles are inconsistently active and provide trans-
verse stability in the foot during postural sway. The quadriceps femoris and anterior tibialis are active
during posterior sway.

Type of Reasoning: Analysis


One must recall the muscles that are active in the lower extremity during erect standing in order to arrive
at the correct conclusion. This requires one to analyze the various muscles presented and make a determi-
nation of the correct solution, which encourages analytical reasoning skill. Knowledge of lower extremity
kinesiology is beneficial to choosing the best solution.

Nonsystem I Safety, Professional Responsibilities, Research

A patient falls while walking in the parallel bars. The therapist is required to fill out an incident report of
the event. Information in the report should include the names of those involved and:

Choices:
1. The cause of this fall and cross-references to others who have fallen in the parallel bars.
2. A description of the event, where the patient was injured, and the corrective actions to be taken.
3. What occurred, when and where it occurred, and witness statements.
4. Witness reports and therapist's opinion as to the cause.
774 Examination C

Teaching Points

Correct Answer: 3
The typical information included on an incident report are the names of those involved, inclusive of wit-
nesses, what occurred, when it occurred, and where it occurred.

Incorrect Choices:
An incident report should avoid interpretive information such as cause of the occurrence or corrective ac-
tions that were taken. There is no presumption that someone was injured.

Type of Reasoning: Evaluation


One must weigh the information presented and determine which statement is most representative of the
information that should appear in an incident report. This requires evaluative reasoning skill, in which
one uses judgment to determine the most likely correct solution. If this question was answered incorrectly,
review information on components of an incident report.

Cardiovascular/Pulmonary and Lymphatic I Interventions

A therapist is beginning manual lymphatic drainage for a patient recently diagnosed with secondary
lymphedema. Initially, proper bandaging for this condition requires a:

Choices:
l. Custom-made low-elastic garment.
2. Gauze wrap.
3. Long-stretch compression wrap (Ace wrap).
4. Short-stretch compression wrap (Comprilan®).

Teaching Points

Correct Answer: 4
A short-stretch wrap has a low resting pressure and high working pressure. This means it has enough pres-
sure to enhance lymphatic return at rest, improve the activity of the lymphangion (contractile unit of the
lymphatic system), and facilitate increased return during muscle pumping activities.

Incorrect Choices:
Long-stretch wraps have high resting pressures and low working pressure. The problem with long-stretch
wraps (Ace wraps) is they can become like a tourniquet at rest and do not provide enough support dur-
ing activities. Custom-made, low-elastic garments are not ordered until the limb reduction has reached a
plateau, which may take 4 to 6 months. Gauze wraps provide no support.

Type of Reasoning: Deductive


This question requires the test taker to recall guidelines for lymphedema management in order to arrive
at a correct conclusion. This is factual information, which is a deductive reasoning skill. For this scenario,
short-stretch compression wrap is required. Refer to manual lymphatic drainage guidelines if answered
incorrectly.
Examination C 775

Neuromuscular I Evaluation, Diagnosis

An elderly individual was found unconscious at home and was hospitalized with a diagnosis of cerebro-
vascular accident (CVA). Examination by the PI reveals normal sensation and movement on the right side
of the body with impaired sensation (touch, pressure, proprioception) and paralysis on the left side of the
body. The left side of the lower face and trunk are similarly impaired. The MOST LIKELY location of the
CVA is the:

Choices:
1. Left side of brainstem.
2. Left parietal lobe.
3. Spinal cord.
4. Right parietal lobe.

Teaching Points
Correct Answer: 4
This patient demonstrates involvement of the long tracts (sensory and motor) indicative of involvement
of the contralateral cerebral cortex, parietal lobe.

Incorrect Choices:
Tracts cross in the medulla so it is the right brain that is involved, not the left. The involvement of the face
indicates a lesion above the level of the midbrain. A lesion in the spinal cord would not affect the face. A
lesion in the brainstem would produce facial signs contralateral to the limb signs.

Type of Reasoning: Analysis


The test taker must determine the precise location of the lesion given the symptoms presented. Questions
that require one to draw conclusions based on symptoms often encourage analytical reasoning skill. For
this patient, the symptoms indicate right parietal lobe damage, which should be reviewed if this question
was answered incorrectly.


Neuromuscular I Evaluation , Diagnosis

The loss of sensory function in peripheral neuropathy is often among the first noticeable symptoms. If
more than one nerve is involved, the sensory loss typically appears as:

Choices:
l. Band-like dysesthesias and paresthesias in the hips and thighs.
2. Paresthesias affecting primarily the proximal limb segments and trunk.
3. Stocking and glove distribution of the lower and upper extremities.
4. Allodynia of the feet accompanied by pronounced dorsiflexor weakness.
776 Examination C

Teaching Points

Correct Answer: 3
Symmetrical involvement of sensory fibers, progressing from distal to proximaL is the hallmark of poly-
neuropathy. It is termed "stocking and glove distribution" and is the result of the dying back of the longest
fibers in all the nerves from distal to proximal. Sensory symptoms include decreased sensation and pain,
paresthesias, and dysesthesias (abnormal sensations such as numbness, tingling, or prickling).

Incorrect Choices:
Proximal involvement (hips) can occur, but only after long-standing disease and distal involvement first.
Involvement of the trunk is not typical. Allodynia refers to the perception of an ordinarily painless stimu-
lus as painfuL and is not characteristic of polyneuropathy.

Type of Reasoning: Inference


This question requires one to determine the likely symptoms for polyneuropathy. This necessitates one
to make inferences about the nature of polyneuropathy, which is an inferential reasoning skill . Through
knowledge of neuropathology, the test taker should determine that polyneuropathy characteristically ap-
pears as a stocking-and-glove distribution of the hands and feet .


Cardiovascular/Pulmonary and Lymphatic I Interventions

The PT is supervising a phase II cardiac rehabilitation class of 10 patients. One of the patients, who is being
monitored with radiotelemetry, is having difficulty. The PT decides to terminate the patient's exercise ses-
sion upon observing:

Choices:
l. An increase in HR 20 beats/ minute above resting.
2. An increase in systolic BP to 150 and diastolic BP to 90 .
3. A second-degree atrioventricular (AV) heart block.
4. I -mm ST segment depression, upsloping.

Teaching Points

Correct Answer: 3
Criteria for reducing exercise intensity or termination according to the American College of Sports Medi-
cine include (1) onset of angina and other symptoms of exertional intolerance; (2) systolic BP ;:::: 240
mm Hg, diastolic BP ;:::: llO mm Hg; (3) > I-mm ST segment depression, horizontal or downsloping; (4)
increased frequency of ventricular arrhythmias; and (5) second-degree or third-degree AV block or other
significant electrocardiogram (ECG) disturbances.

Incorrect Choices:
The other findings do not fall within the criteria listed. HR is expected to rise propoltionally to workload
intensity unless the patient is on beta-blockers. The rise in BP is not significant enough to stop exercise.
1-mm ST segment depression that is isoelectric or within 1 mm is within normal limits.

Type of Reasoning: Analysis


One must determine which symptoms warrant termination of the treatment session and which symptoms
merely require monitoring. Through analytical reasoning, the person must interpret the symptoms present-
ed and determine their significance. For this patient, second-degree AV heart block warrants termination
of the treatment session. If this question was answered incorrectly, review cardiac rehabilitation guidelines
for terminating exercise.
Examination C 777

Cardiovascular/Pulmonary and Lymphatic I Interventions

A patient presents with severe claudication that is evident when walking distances greater than 200 feet.
The patient also exhibits muscle fatigue and cramping of both calf muscles. Upon examination, the PT
finds the skin is pale and shiny with some trophic nail changes. The BEST choice for intervention is to:

Choices:
1. Begin with an interval walking program, exercising just to the point of pain.
2. Avoid any exercise stress until the patient has been on calcium channel blockers for at least 2 weeks.
3. Utilize a walking program of moderate intensity, instructing the patient that some pain is expected and
to be tolerated.
4. Utilize non-weight-bearing exercises such as cycle ergometry.

Teaching Points

Correct Answer: 1
This patient is exhibiting classic signs of peripheral artery disease (PAD). Rehabilitation guidelines for
arterial disease include using an intermittent walking program of moderate intensity and duration, 2 to 3
times/day, 3 to 5 days/wk. The patient should be instructed to exercise to the point of claudication pain
within 3 to 5 minutes, not beyond.

Incorrect Choices:
Exhaustive exercise and exercising with persistent pain are contraindicated. Calcium channel blockers
may be used in vasospastic disease; exercise is not contraindicated. A cycle ergometry program is less
desirable than a walking program (treadmill or track) to reduce claudication (ACSM Guidelines for Exercise
Testing and Prescription, 8th ed. Philadelphia: Lippincott Williams & Wilkins).

Type of Reasoning: Inductive


One must utilize clinical judgment to determine the BEST choice for intervention, which is an inductive
reasoning skill. The test taker must understand what the symptoms are indicative of in order to arrive
at the correct conclusion. If this question was answered incorrectly, refer to information on exercise for
patients with PAD .


Integumentary I Interventions

A patient is hospitalized with diabetes and a large stage II plantar ulcer located over the right heel. The pa-
tient has been non-weight-bearing for the past 2 weeks as a result of the ulcer. The PT determines the BEST
intervention is:

Choices:
1. Clean and bandage with a sterile gauze dressing.
2. A surgical consult because available wound dressings will not promote healing.
3. Clean and debride the wound and apply a hydrogel dressing.
4 . Wash the foot and apply skin lubricants followed by a transparent film dressing.
778 Examination C

Teaching Points

Correct Answer: 3
A stage II ulcer (deep ulcer) involves a partial-thickness skin loss with involvement of epidermis, dermis,
or both; it is reversible. Intervention should be directed toward improving perfusion and relieving local-
ized pressure. The wound should be cleaned with an antimicrobial agent, debrided of necrotic tissue, and
covered with a sterile dressing. Hydrogel dressings maintain moisture in the wound bed, soften necrotic
tissue, and support autolytic debridement. Pressure relief is also an important consideration. Techniques
of protective foot care should be taught.

Incorrect Choices:
Application of a dry, sterile gauze dressing is contraindicated, as is the application of skin lubricants. A
stage II ulcer has the potential to heal; a surgical consult is not needed at this time.

Type of Reasoning: Inference


One must utilize clinical judgment and knowledge of wound care approaches to determine the BEST
intervention approach for this patient, which requires inductive reasoning skill. For this patient, cleaning
and debriding of the wound with application of a hydrogel dressing is most appropriate for the type of
wound described. If this question was answered incorrectly, review information on wound care proce-
dures for stage II plantar ulcers .


Nonsystem I Equipment, Devices, Therapeutic Modalities

A PT is prescribing a wheelchair for a patient with left hemiplegia who is of average height (5 feet 7 inches) .
The MOST beneficial feature to include in this prescription is:

Choices:
1. Desk armrests.
2. A 17.5-inch seat height.
3. A 20-inch seat height.
4. Elevating leg rests.

Teaching Points
Correct Answer: 2
A hemi- or low-seat wheelchair has a seat height of 17.5 inches. The lower seat height permits the patient
to propel and steer the wheelchair using the sound right upper and lower extremities.

Incorrect Choices:
A wheelchair with a standard seat height (20 inches) is too high to permit efficient use of the sound lower
extremity for steering and propulsion. Elevating leg rests may be considered if the patient has problems
with edema (not indicated in this case). Shorter-length desk arm rests are a useful option to allow an indi-
vidual to get close to tables or work surfaces but they are not a priority in this example.

Type of Reasoning: Inference


This question requires one to determine the most beneficial feature in wheelchair prescription for a patient
with hemiplegia. This requires knowledge of various wheelchair features and which patient populations
derive the most benefit from these features.
Examination C 779

Nonsystem I Safety, Professional Responsibilities, Research

A patient who is participating in a cardiac rehabilitation program suddenly collapses and falls to the floor.
The PT is the lone rescuer on site. The therapist checks for a response and finds the patient unresponsive.
After activating the emergency response system (phone 911), the therapist, who is a trained health care
provider, should:

Choices:
1. Give two rescue breaths followed by 15 chest compressions, repeating the cycle for at least 2 minutes.
2. Give 100 chest compressions per minute.
3. Use the automated external defibrillator (AED) to shock the patient after 3 minutes of cardiopulmonary
resuscitation (CPR).
4. Begin CPR and attach and use the AED as soon as it is available.

Teaching Points

Correct Answer: 4
Guidelines from the American Heart Association (2010) concerning Basic Life Support and CPR specify
that the first responder call 911 for unresponsive adults, get an AED (if available), and return to the victim
to provide CPR and defibrillation, if needed. Trained HCPs can use ventilations (1 breath every to 8 sec-
onds) with chest compressions (at least 100/minute) .

Incorrect Choices:
The responder should use the AED as soon as possible after beginning CPR, and not wait 3 minutes. The
compression rate for adult CPR is about IOO/ minute with a recommended compression-to-ventilation
ratio of 30:2. The old ratio was 15:2. Untrained rescuers should use compressions only.

Type of Reasoning: Deductive


This question requires one to recall the proper and current Basic Life Support for health care provider
guidelines. Questions that ask one to recall knowledge of protocols and guidelines necessitates deductive
reasoning skill. If this question was answered incorrectly, refer to current guidelines for CPR.

Musculoskeletal I Evaluation, Diagnosis

A patient who was casted for 3 weeks after a grade III right ankle sprain has been referred to physical
therapy for mobility exercises. Examination shows a loss of 10 degrees of dorsiflexion. The patient will have
the MOST difficulty in:

Choices:
1. Ambulating barefoot.
2. Descending stairs.
3. Descending a ramp .
4. Ambulating over rough surfaces.
780 Examination C

Teaching Points

Correct Answer: 2
Loss of dorsiflexion will make descending stairs most difficult because the ankle must have dorsiflexion
during the single-limb support phase during descent.

Incorrect Choices:
Although the activity may be changed, full range in dorsiflexion is not needed for the other choices.

Type of Reasoning: Inductive


The test taker must analyze all of the activities and then utilize clinical judgment to determine which ac-
tivity requires the most ankle dorsiflexion range. This requires inductive reasoning skill, in which clinical
judgment is paramount to finding the correct solution. For this patient, descending stairs would be MOST
difficult because full-range dorsiflexion is required to complete the task successfully.

Musculoskeletal I Interventions

The torque output produced in the sitting position during isokinetic exercise involving the hamstrings is:

Choices:
1. Lower than the torque actually generated by the hamstrings.
2. Higher because of eccentric assistance of the quadriceps.
3. Higher than the torque actually generated by the contracting hamstrings.
4. Lower because of resistance of the quadriceps.

Teaching Points

Correct Answer: 3
Gravity-produced torque adds to the force generated by the hamstrings when they contract, giving a higher
torque output than is actually produced by the muscle (gravity-assisted exercise) . Testing values may be
misleading; software is available to correct for the effects of gravity.

Incorrect Choices:
Hamstring work is lower, not higher, during this activity. Eccentric assistance by the quadricps (effects of
reciprocal inhibition) does not lower or significantly raise the work of the hamstrings but rather serves to
smooth out contractions.

Type of Reasoning: Deductive


This question requires one to recall biomechanical guidelines of torque output in exercise. Recall of
factual information and guidelines is a deductive reasoning skill. To arrive at the correct conclusion, one
must recall that gravity-produced torque adds to the force generated by a contracting muscle, resulting in a
higher torque output than what is produced by the muscle itself. If this question was answered incorrectly,
review biomechanical guidelines of torque for isokinetic exercises.
Examination C 781

Musculoskeletal I Interventions

The manual therapy technique appropriate to correct a closing restriction ofT5 on TG is:

Choices:
1. Central posteroanterior (PA) pressure at a GO-degree angle on the spinous process ofTG while stabilizing
T5.
2. Central PA pressure at a 4S-degree angle on the spinous process ofTS while stabilizing TG.
3. Unilateral PA pressure at a 45-degree angle on the right transverse process ofTG while stabilizing TS.
4. Unilateral PA pressure at a GO-degree angle on the left transverse process ofTG while stabilizing TS.

Teaching Points

Correct Answer: 1
In a closing restriction, the inferior facets of the superior vertebrae will not inferiorly glide on the superior
facets of the inferior vertebra. Therefore, T5 inferior facets will not caudally glide on the superior facets
ofTG. Stabilizing TS and application of pressure to TG localizes the cephalad movement of the superior
facets TG on T5 bilaterally. The angle of the thoracic facets is GO degrees; therefore, the application of force
should be at the same plane.

Incorrect Choices:
Providing the force at 45 degrees does not match the anatomical orientation for the facets in that region,
so will not be as effective. The force should be a central PA glide because a unilateral glide will promote
rotation and/or side-bending rather than extension.

Type of Reasoning: Analysis


Knowledge of anatomy and biomechanics is important for choosing the correct solution for this question.
The test taker utilizes analytical reasoning skills, determining the meaning of the four choices and decid-
ing which choice most accurately represents the correct manual therapy technique for a closing restriction
ofT5 on TG. If this question was answered incorrectly, refer to information on manual therapy for closing
restriction of thoracic vertebrae.

Neuromuscular I Evaluation, Diagnosis

A patient presents with an acute onset of vertigo overnight. Symptoms worsen with rapid change in head
position. If the head is held still, symptoms subside usually within 30 to GO seconds. The MOST likely
cause of these signs and symptoms is:

Choices:
1. Benign paroxysmal positional vertigo (BPPV).
2. Bilateral vestibular neuritis.
3. Meniere's disease.
4. Acoustic neuroma.
782 Exam ination C

Teaching Points

Correct Answer: 1
BPPV is characterized by acute onset of vertigo and is positional, related to the provoking stimulus of head
movement.

Incorrect Choices:
Vestibular neuritis is an inflammation of the vestibular nerve caused by a virus and typically produces
symptoms of dysequilibrium, nystagmus, nausea, and severe vertigo. Meniere's disease is characterized by
a sensation of fullness in the ears associated with abnormal fluid buildup. Additional symptoms include
tinnitus, vertigo, nausea, and hearing loss. Acoustic neuroma (vestibular schwannoma) produces unilat-
eral sensorineural hearing loss along with vestibular symptoms.

Type of Reasoning: Analysis


One must analyze the symptoms and make a determination of the most likely cause for these symptoms,
which requires analytical reasoning skill. For this case, the patient's symptom's MOST likely cause is BPPV
owing to the nature of an acute onset and being related to changing position of the head. Questions that
inquire about a group of symptoms and whereby the test taker must determine the diagnosis often utilize
analytical reasoning skill .


Cardiovascular/Pulmonary and Lymphatic I Evaluation, Diagnosis

A patient with a significant history of coronary artery disease is currently taking atropine. Based on knowl-
edge of the effects of this medication, the therapist expects:

Choices:
1. Palpitations at rest and with exercise.
2. Orthostatic hypotension.
3. Increased HR and contractility at rest.
4 . Increased myocardial ischemia.

Teaching Points

Correct Answer: 3
Atropine is an anticholinergic agent (it blocks the action of acetylcholine at parasympathetic sites in
smooth muscle, secretory glands, and the central nervous system). It produces an increase in HR and con-
tractility and is used to treat symptomatic sinus bradycardia and exercise-induced bronchospasm.

Incorrect Choices:
The other choices are not expected effects of this medication but rather adverse cardiovascular reactions
(not asked for in this question) . These can include tachycardia, orthostatic hypotension, palpitations,
ventricular fibrillation, and increased ischemia in patients with MI.

Type of Reasoning: Inference


This question requires one to infer the expected effects of atropine, thereby necessitating inferential reason-
ing skill. For this agent, atropine, one should anticipate possible increased HR and contractility at rest.
Knowledge of anticholinergic agents and their effects are beneficial for arriving at the correct conclusion,
which should be reviewed if this question was answered incorrectly.

Examination C 783

Neuromuscular I Interventions

A patient recovering from stroke is taking warfarin (Coumadin) . During rehabilitation, it would be impor-
tant to watch for potential adverse reactions including:

Choices:
1. Edema and dermatitis.
2. Palpitations and edema.
3. Cellulitis and xeroderma.
4. Hematuria and ecchymosis.

Teaching Points

Correct Answer: 4
Warfarin sodium (Coumadin) is an anticoagulant indicated in the prophylaxis and treatment of venous
thrombosis, pulmonary embolism, and thromboembolic disorders. Potential adverse reactions include
hematuria and ecchymosis (skin discoloration and hemorrhaging) . Serious bleeding is possible with drug
toxicity.

Incorrect Choices:
Xeroderma (dry skin), cellulitis (inflammation of tissues), and palpitations (awareness of heart rhythm
abnormalities) are not seen as adverse reactions with warfarin.

Type of Reasoning: Inference


One must infer the possible effects of warfarin therapy in order to arrive at the correct conclusion. This
requires one to understand the common indications for the drug therapy and adverse side effects. For this
medication, one should be watchful for hematuria and ecchymosis because of warfarin's properties of
blood thinning. If this question was answered incorrectly, review information on warfarin therapy.


Metabolic/Endocrine I Evaluation, Diagnosis

The PT reviews the laboratory results of a patient admitted to the acute care hospital yesterday: Hematocrit
45%, fasting blood glucose 180 mg/dL, and cholesterol 180 mg/ dL. Based on these laboratory results, the
patient MOST likely has:

Choices:
1. Hyperlipidemia.
2. Anemia.
3. Diabetes mellitus.
4. Polycythemia vera.

Teaching Points

Correct Answer: 3
Fasting blood glucose of 180 mg/ dL is abnormal and indicative of diabetes.
784 Examination C

Incorrect Choices:
The hematocrit and cholesterol readings are within normal limits. Hyperlipidemia (excessive level of
lipids in the blood) and anemia (reduced circulating red blood cells [RBCsJ) are not indicated. An elevated
hematocrit could be indicative of polycythemia vera (proliferation or hyperplasia of all bone marrow cells
with an increase ofRBCs and hemoglobin concentration) .

Type of Reasoning: Analysis


This question requires one to determine the precise meaning of the laboratory values as it relates to a diag-
nosis for the patient. In this situation, the patient's laboratory values indicate diabetes mellitus. Drawing
conclusions based on a group of indicators or symptoms requires analytical reasoning skill. If this question
was answered incorrectly, review information on blood glucose laboratory values .


Neuromuscular I Interventions

A patient recovering from a stroke is having difficulty with stair climbing. During ascent, the patient is able
to position the more involved foot on the step above but is unable to transfer the weight up to the next stair
level. The BEST intervention to solve this problem is:

Choices:
1. Bridging, holding.
2 . Standing, partial wall squats.
3. Plantigrade, knee flexion with hip extension.
4. Standing, side steps.

Teaching Points

Correct Answer: 2
The quadriceps muscle is responsible for most of the energy generation needed to transfer up stairs to
the next level. Partial wall squats are the BEST choice to strengthen these muscles (closed-chain exercise).
During forward continuance (corresponding to mid stance), the ankle plantarflexors assist. Hip extensors
are also active concentrically, assisting these actions.

Incorrect Choices:
The other choices might be good lead-up activities for gait but would not optimally strengthen the key
muscles involved in ascending stairs.

Type of Reasoning: Analysis


This question requires one to analyze the described patient challenge and then determine the BEST in-
tervention. The test taker must determine what the cause is for the patient who has difficulty transferring
weight to ascend stairs, which necessitates analytical reasoning. After analyzing the situation, one should
conclude that the quadriceps muscle is weak and that partial wall squats in standing would BEST address
this issue.
Exa mination C 785

Cardiovascular/Pulmonary and Lymphatic I Evaluation, Diagnosis

An elderly patient has been hospitalized and is on complete bed rest for 10 days. A physical therapy referral
requests mobilization out of bed and ambulation. The patient complains of aching in the right calf. The
therapist's examination reveals calf tenderness with slight swelling and warmth. The therapist decides to:

Choices :
1. Ambulate the patient with support stockings on.
2. Postpone ambulation and report the findings immediately.
3. Begin with ankle pump exercises in bed and then ambulate.
4. Use only AROM exercises with the patient sitting at the edge of the bed.

Teaching Points

Correct Answer: 2
The patient is exhibiting early signs of acute deep vein thrombophlebitis (DVf) . These findings should be
reported immediately.

Incorrect Choices:
Exercise and ambulation are contraindicated during the acute phase. If DVf is present, the patient will be
given anticoagulation medication and will remain on bed rest with elevation of the involved leg until the
acute phase subsides.

Type of Reasoning: Evaluation


One must evaluate the patient's symptoms and determine the best course of action, given one's under-
standing of the symptoms. Here, the patient's symptoms are indicative of DVf, which necessitates im-
mediate notification and postponement of ambulation. If this question was answered incorrectly, refer to
information on DVf and appropriate actions.

Neuromuscular I Examination

A 9-year-old boy with Duchenne's muscular dystrophy is referred for home care. The PT should BEGIN the
examination by:

Choices:
1. Performing a complete motor examination.
2. Performing a functiona l examination using the weeFIM.
3. Asking the child and his parents to describe the boy's most serious functional limitations.
4. Asking the parents to outline the boy's past rehabilitation successes.
786 Examination C

Teaching Points
Correct Answer: 3
The child and his parents/caretakers play an important part in determining impairments, functionallimi-
tations, disability, and future intelVentions. Taking a thorough initial history is important in determining
what other components of the examination would be appropriate.

Incorrect Choices:
The other choices may indeed be appropriate; however, performing the intelView first helps decide which
examination tools are needed.

Type of Reasoning: Inductive


Questions such as these are difficult because one must determine what comes first in a process, especially
when all the potential choices can be correct in the evaluation of a patient. For this question, the other
choices are appropriate when conducting an examination, but the test taker must determine which one
most logically should come first. This requires clinical judgment, an inductive reasoning skill.


Musculoskeletal I Evaluation, Diagnosis

During an examination of an adolescent female who complains of anterior knee pain, the PT obselVes that
the lower extremity shows medial femoral torsion and toeing-in position of the feet. The lower extremity
position may be indicative of excessive hip :

Choices:
1. Anteversion.
2. Medial/internal rotation.
3. Retroversion.
4. Lateral/external rotation.

Teaching Points

Correct Answer: 1
The pathology commonly associated with medial femoral torsion and toeing-in is hip anteversion due
to an increase in the antetorsion angle (> 15 degrees) between the femoral condyles and the neck of the
femur.

Incorrect Choices:
Excessive internal or external rotation of the hip would not force the patient to stand with toeing-in. Retro-
version of the hips would cause the feet to toe-out.

Type of Reasoning: Analysis


This question provides the symptoms, and the test taker must determine the likely cause, which requires
analytical reasoning skill. Here, the symptoms are analyzed in order to make a determination for the cause
of this patient's medial femoral torsion and toeing-in position of the feet. The test taker should conclude
that excessive hip anteversion is the cause, which should be reviewed if this question was answered incor-
rectly.

Examination C 787

Musc u los k eletal I In te rve nti o n s

A patient with osteoporosis and no fractures complains of increased middle and lower back pain during
breathing and other functional activities. The MOST beneficial interventions for this patient include patient
education and:

Choices:
1. Trunk extension and abdominal stabilization exercises.
2. Trunk rotation and abdominal stabilization exercises.
3. Trunk flexion and extension exercises.
4. Trunk flexion and rotation exercises.

Teaching Points

Correct Answer: 1
It is important to strengthen from the core to the floor as well as train in proprioception and balance
enhancement techniques. Trunk extension and abdominal stabilization exercises are indicated.

Incorrect Choices:
Patients should avoid trunk flexion or rotation exercise because it can cause a compression fracture of the
spine.

Type of Reasoning: Inference


One must determine first what may be the cause of the patient's pain and then determine the interven-
tion approach that will BEST remedy the symptoms. If this question was answered incorrectly, refer to
intervention approaches for osteoporosis.

Nonsystem I Safety, P rofess io n al Responsibilities, Researc h

A single 22-year-old woman who is 3 months' pregnant arrives at a therapist's private practice complaining
of shoulder and leg pain. She has a black eye and some bruising at the wrists. The state in which the thera-
pist practices has direct access. The BEST course of action for the therapist is:

Choices:
1. Administer massage for bruising, TENS, and ice modalities for pain, as indicated by the examination
findings .
2. Direct the patient to the nearest ambulatory care center for physician evaluation.
3. Refuse to examine the patient and send her to the nearest emergency room.
4. Examine the patient, and if abuse is suspected, report the findings to the appropriate authorities.
788 Examination C

Teaching Points

Correct Answer: 4
According to the American Physical Therapy Association's (APTA's) Guidelines for Recognizing and
Providing Care for Victims of Domestic Violence, this patient falls into a category of high risk. Women
between the ages of 17 and 28 years and women who are single, separated, or divorced or who are plan-
ning a separation or divorce are at high risk. Battered women usually have more than one injury. Most
injuries occur in the head, face, neck, breasts, and abdomen. According to the American Medical Associa-
tion (AMA), battered women represent 23% of pregnant women who seek prenatal care. The victim may
not volunteer information about her situation, but more often than not when asked will reveal it. It is
important for the PT to examine the patient and, if abuse is suspected, report the findings to appropriate
authorities. The therapist should be familiar with resources available for victims of domestic violence and
their own state reporting laws.

Incorrect Choices:
In most state jurisdictions, a PT may be fined or indicted for failure to report (all other choices).

Type of Reasoning: Evaluation


This question requires one to make a judgment call for the BEST course of action, given the patient's
symptoms. Questions that necessitate professional judgment in ethical situations often utilize evaluative
reasoning skill, in which the merits of each potential choice are weighed. For this patient, it is important to
do a comprehensive examination and then, if abuse is suspected, report it to the appropriate authorities.

Nonsystem I Equipment, Devices, Therapeutic Modalities

An elderly patient presents with a stage III decubitus ulcer on the plantar surface of the right foot. After a
series of conservative interventions with limited success, the therapist chooses to apply electrical stimula-
tion for tissue repair. The BEST choice for electrical current in this case is:

Choices:
1. Low-volt biphasic pulsed current.
2. High-volt monophasic pulsed current.
3. Medium-frequency burst current.
4. Medium-frequency beat current.

Teaching Points

Correct Answer: 2
Because high-volt pulsed current is a monophasic, unidirectional current, the unidirectional current
would produce a therapeutic effect at the active (treatment) electrode. A negative charge (polarity) should
be applied for a bactericidal effect or a positive charge given to promote wound healing.

Incorrect Choices:
A biphasic current, which alternates the polarity, would tend to negate the treatment effects. Russian
(burst) and interferential (beat) are medium-frequency biphasic currents. Interrupted currents (> 0.5-
second interruption) are also not used for tissue healing.

Type of Reasoning: Deductive


One must recall the appropriate electrical stimulation parameters to treat the patient's condition, which re-
quires deductive reasoning skill. One must recall the differences between currents of differing low-, medium-,
and high-volt currents, as well as pulsed, burst, and beat current, and the therapeutic effects on wound heal-
ing. If this question was answered incorrectly, review guidelines for electrical stimulation for wound healing.
Examination C 789

Nonsystem I Equipment, Devices, Therapeutic Modalities

A patient presents with pain radiating down the posterior hip and thigh as a result of a herniated disc in
the lumbar spine. The therapist decides to apply mechanical traction. If the patient can tolerate it, the PRE-
FERRED patient position is:

Choices:
1. Supine with one knee flexed.
2. Supine with both knees flexed .
3. Prone with no pillow.
4. Prone with pillow under the abdomen.

Teaching Points

Correct Answer: 3
Placing the patient in the prone position would better align the spine so that the pull of the traction
would b e along the axis of the vertebral bodies.

Incorrect Choices:
If the prone position is intolerable, then a pillow may be placed under the abdomen. Flexing the spine
could exacerbate the disc herniation. A supine, knee-flexed position can be used for spinal stenosis.

Type of Reasoning: Inductive


The test taker must determine which patient position is PREFERRED for mechanical traction for a herni-
ated lumbar disc. This requires recall of traction guidelines, but also clinical judgment to determine the
correct solution, which necessitates inductive reasoning. Some other positions are acceptable but not
preferred. If this question was answered incorrectly, review guidelines for mechanical traction for lumbar
disk herniation .


Neuromuscular I Interventions

A patient recovering from stroke demonstrates hemiparesis of the right upper extremity with moderate
flexion and extension synergies (flexion stronger than extension) . The therapist's goal is to strengthen the
shoulder muscles first to promote elevation of the arm. The BEST method to achieve this goal is to promote:

Choices:
1. Shoulder horizontal adduction with elbow extension.
2. Shoulder horizontal adduction with elbow flexion.
3. Shoulder abduction with elbow flexion.
4. Shoulder flexion with elbow extension.

Teaching Points

Correct Answer: 4
Obligatory hemiplegic synergies are present and should not be reinforced. Shoulder flexion with elbow
extension is the correct choice. It is an out-of-synergy combination that strengthens the shoulder flexors
needed to stabilize the shoulder in an elevated functional position. In sitting, bending forward with elbow
straight and hand touching floor is a good example of an early activity to promote this.
Examination C

Incorrect Choices:
Shoulder abduction with elbow flexion is part of the flexion synergy, whereas adduction with elbow exten-
sion is part of the extension synergy. Adduction with elbow flexion is an out-of-synergy combination but
does not strengthen muscles needed to elevate and stabilize the upper extremity.

Type of Reasoning: Inference


One must evaluate the patient's symptoms and infer the BEST method for strengthening the shoulder
muscles, thereby requiring inferential reasoning. Given the diagnosis and current status, shoulder flexion
with elbow extension is BEST because hemiplegic synergies are discouraged using this approach. If this
question was answered incorrectly, review limb synergies with stroke and out-of-synergy movement pat-
terns .


Neuromuscular I Examination

A patient recovering from a middle cerebral artery stroke presents with gaze deviation of the eyes. In this
type of stroke, examination will reveal deviation of the involved eye:

Choices:
1. Toward the hemiplegic side.
2. Upward.
3. Toward the sound side.
4. Down and out.

Teaching Points

Correct Answer: 3
Unopposed action of the eye muscles causes the eye to deviate in the direction of the intact musculature.
Thus, patients with hemispheric lesions involving a large frontal or pari tal lobe lesion look away from the
hemiplegic side and toward the sound side.

Incorrect Choices:
Patients with brainstem pontine lesions cannot look toward the nonparalyzed side. Upward or vertical
gaze results from lesions in the upper brainstem. An eye that is deviated downward and out with ptosis is
characteristic of a lesion involving eN III.

Type of Reasoning: Inference


One must recall the nature of gaze deviation after middle cerebral artery stroke in order to arrive at the
correct conclusion. This requires one to utilize inferential reasoning, in which the test taker must draw
conclusions from the evidence presented. For this patient, the involved eye may deviate toward the sound
side because of the unopposed action of the eye muscles. If this question was answered incorrectly, review
information on gaze deviation of the eyes after stroke.

Examination C 791

Cardiovascular/Pulmonary and Lymphatic I Examination

A patient is referred for physical therapy after an exercise tolerance test. The physician reports the test was
positive and had to be terminated at 7 minutes. Based on the therapist's knowledge of this procedure, the
therapist expects the patient may have exhibited:

Choices:
1. ST segment depression from baseline of 3-mm horizontal or downsloping depression.
2. A hypertensive response with blood pressure of at least 170/ 95.
3. Mild angina and dyspnea with progressive increases in the treadmill speed and grade.
4. ECG changes from baseline of I-mm ST segment elevation.

Teaching Points

Correct Answer: 1
A positive exercise tolerance test (graded exercise test) indicates myocardial ischemia with increasing exer-
cise intensities. The optimal test duration is 8 to 12 minutes but can be terminated if symptoms of exer-
tional intolerance are evident. The American College of Sports Medicine (ACSM) indicates these include
ECG changes from baseline (> 2 mm horizontal or downsloping; ST segment depression, or > 2 mm ST
segment elevation).

Incorrect Choices:
Additional signs of exertional intolerance that indicate the test should be terminated include onset of
moderate to severe angina (some angina is expected with increasing work), a drop in systolic BP with
increasing workload, serious arrhythmias, signs of exertional intolerance (pallor, cyanosis, cold or clammy
skin), unusual or severe shortness of breath (some shortness of breath is expected) , CNS signs (ataxia,
vertigo, visual or gait problems, confusion), and a hypertensive response equal to or greater than 260/115
(a BP of 170/95 is not a reason for stopping the test (ACSM Guidelines for Exercise Testing and Prescription,
8th ed. Philadelphia: Lippincott Williams &. Wilkins) .

Type of Reasoning: Inference


This question requires one to infer information based on the evidence presented. In this case, the test taker
must infer what a positive exercise tolerance test indicates and early termination of the test after 7 minutes.
If this question was answered incorrectly, refer to information on exercise tolerance testing and ACSM
guidelines for terminating the test.
792 Examination C

Musculoskeletal I Evaluation, Diagnosis

A college soccer player sustained a hyperextension knee injury when kicking the ball with the other lower
extremity. The patient was taken to the emergency room of a local hospital and was diagnosed with "knee
sprain." The patient was sent to physical therapy the next day for rehabilitation. As part of the examination,
the therapist conducts the test shown in the figure. The test is positive. The type of exercise that is indicated
in the acute phase of treatment is:

Magee 0 (2002). Orthopedic Phys ical Assessment, 4th ed. Philadelphia, W. B. Saunders, Figure 12-27B,
page 700, with permission.

Choices:
1. Agility exercises.
2. Closed-chain terminal knee extension exercises.
3. Open-chain terminal knee extension exercises.
4. Plyometric functional exercises.

Teaching Points

Correct Answer: 2
The test that was conducted was a Lachman's test to determine integrity of the ACL. A positive test suggests
laxity of the ACL. Closed-chain terminal knee extension exercises are safe and effective secondary to the
dynamic stability inherent with this type of exercise.

Incorrect Choices:
Quick cutting/ lateral movements that occur in agility training and heavy joint loading that occurs with
plyometric exercise should be avoided until the muscular restraints that reduce excessive anterior transla-
tion of the affected tibiofemoral joint are strengthened. Open-chain knee extension may place excessive
load on the ACL.

Type of Reasoning: Analysis


Questions that include analysis of information depicted in pictures often necessitate analytical reasoning
skill. This question requires one to determine what type of test is being depicted in the picture and then
determine, upon positive result, the best exercise approach. This picture depicts a Lachman's test for
determining ACL integrity. The positive test indicates laxity of the ACL and that closed-chain terminal
knee extension exercises are best.
Examinatio n C 793

Integument ary I Interventions

A patient with a grade III diabetic ulcer is being treated with a calcium alginate wound dressing. This type
of dressing can be expected to:

Choices:
1. Absorb exudate and allow rapid moisture evaporation.
2 . Facilitate autolytic debridement and absorb exudate.
3. Provide semirigid support for the limb, while maintaining a sterile field.
4. Restrict bacteria from the wound, while supporting the tissues.

Teaching Points

Correct Answer: 2
Moisture-retentive occlusive wound dressings such as calcium alginate are recommended for use on
exudating wounds (grade III ulcer). They maintain a moist wound environment, absorb exudate, provide
autolytic debridement, reduce pain at the wound site, or promote faster healing (reepithelialization) .

Incorrect Ch oices:
Calcium alginate dressings do NOT allow rapid evaporation. A disadvantage is that the dressing is very
permeable to bacteria, urine, and so forth. Unna's boot is a semi-rigid dressing that provides limb support.

Type of Reasoning: Deductive


This question requires one to recall the properties of a calcium alginate wound dressing. Recall of proto-
cols and guidelines and other types of factual information often require deductive reasoning skill . For this
question, calcium alginate dressings can be expected to facilitate autolytic debridement and absorption
of exudate. If this question was answered incorrectly, review information on calcium alginate and other
wound dressings.

Nonsystem I Safety, Professional Respons ibili t ies, Research

The PT receives a referral to treat a hospitalized patient with adhesive capsulitis and a 5-year history of cir-
rhosis and hepatitis B. The therapist should:

Choices:
1. Use droplet transmission precautions.
2. Use contact precautions.
3. Wear personal protection equipment (PPE) when transporting the patient to therapy.
4. Ask the patient to wear gloves and avoid contact.
794 Examination C

Teaching Points

Correct Answer: 2
Hepatitis B is a viral infection that is transmitted by close contact with the infected patient's body fluids
(nasopharyngeal exudate, saliva, sweat, urine, feces, semen, vaginal secretions) and blood and blood prod-
ucts. Health care workers should be vaccinated against the possibility of infection because they are in a
high-risk category. Contact precautions should be observed to reduce the risk of microorganism transmis-
sion by direct or indirect contact (refer to Box 6-1, Standard Precautions, Chapter 6) .

Incorrect Choices:
Droplet precautions are used when microorganisms can be transmitted by the patient during coughing,
sneezing, or talking. The therapist should wear PPE, gloves, and gowns when in direct contact with the
patient. The patient does not wear the gloves.

Type of Reasoning: Dedu ctive


This question causes one to recall the guidelines for standard precautions when working with a patient
who has a history of cirrhosis and hepatitis B. The recall of guidelines is factual in nature and necessitates
deductive reasoning skill. One must recall how hepatitis B is transmitted to arrive at the correct conclusion.
For this patient, the therapist should avoid direct exposure to any blood or body fluids .


Musculoskeletal I Evaluat ion, Diagnosis

A patient with a transtibial amputation is learning to walk using a patellar tendon-bearing (PTB) prosthesis
and is having difficulty maintaining knee stability from heel-strike to foot-flat. The muscles that are MOST
likely weak are the:

Choices:
1. Knee extensors.
2. Back extensors.
3. Hip flexors.
4. Knee flexors.

Teaching Points

Correct Answer: 1
The knee extensors (quadriceps) are maximally active at heel-strike (initial contact) to stabilize the knee
and counteract the flexion moment.

Incorrect Choices:
Erector spinae, gluteus maximus, and hamstrings contribute to core stability (trunk and pelvis) and assist
in counteracting the flexion moment from heel-strike to foot-flat. Hip flexors contribute to initiate swing
(acceleration to midswing), whereas knee flexors (hamstrings) decelerate the momentum of the swinging
leg (midswing to deceleration).

Type of Reasoning: Inference


One must infer the reason for the difficulty in maintaining prosthetic stability in order to choose the cor-
rect solution. This requires inferential reasoning, where one must determine the muscles that are MOST
likely weak.
Examination C 795

Cardiovascular/Pulmonary and Lymphatic I Evaluation, Diagnosis

A patient with a history of coronary artery disease and recent myocardial infarction (MI) is exercising in an
inpatient cardiac rehabilitation program. Because the patient is new, continuous ECG telemetry monitoring
is being done. The therapist observes the following. The BEST course of action is to:

H ,,! I :t

From: Jones, S (2005) ECG Notes. Philadelphia, FA Davis, p.51, with permission.

Choices:
1. Have the patient sit down and send him/her back to the room after a brief rest period.
2. Activate the emergency medical response team.
3. Have the patient sit down, continue monitoring, and notify the physician immediately.
4. Have the patient sit down, rest, and then resume the exercise at a lower intensity.

Teaching Points

Correct Answer: 3
This tracing shows premature ventricular contractions (PVCs) that are multifocal (originating from dif-
ferent irritable ventricular focus) . These multiform PVCs pose a potential danger of deteriorating into
ventricular tachycardia and ventricular fibrillation (cardiac standstill). Because the heart is demonstrating
a high degree of irritability, the BEST course of action is to stop the exercise, have the patient sit down,
continue monitoring carefully, and notify medical staff (attending physician) immediately.

Incorrect Choices:
Failure to report this finding by allowing the patient to rest or return to the room can be life-threatening.
The patient has not arrested; therefore, the emergency medical response team should not be activated.

Type of Reasoning: Evaluation


This question actually couples two types of reasoning. First, analytical reasoning is used to evaluate the
information depicted in the picture. However, ultimately, evaluative reasoning skills are used to determine
the BEST course of action, given the information presented. For this patient, the information indicated
multifocal PVCs and necessitates termination of exercise with careful monitoring and notification of the
physician. If this question was answered incorrectly, refer to information on ECG interpretations and
pves.
796 Examination C

Neuromuscular I Interventions

A patient recovering from traumatic brain injury (TBI) is unable to bring the right foot up on the step dur-
ing stair climbing training. The BEST training activity is to:

Choices:
1. Practice stair climbing inside the parallel bars using a 3-inch step.
2. Practice marching in place.
3. Passively bring the foot up and place it on the 7 -inch step.
4. Strengthen the hip flexors using an isokinetic training device before attempting stair climbing.

Teaching Points

Correct Answer: 1
The most appropriate lead-up activity to promote the skill of stair climbing is practice using a 3-inch step
in the parallel bars.

Incorrect Choices:
Passive movements do not promote active learning. Marching in place and isokinetic training may im-
prove the strength of the hip flexors but do not promote the same synergistic patterns of muscle activity as
the desired skill.

Type of Reasoning: Inductive


One must determine through clinical judgment the BEST approach for promoting the skill of stair climb-
ing. This question requires inductive reasoning skill, in which the test taker must first determine the
problem and then judge which intervention approach leads up to improving stair climbing ability. If this
question was answered incorrectly, review information on exercises to promote stair negotiation .


System Interactions I Evaluation, Diagnosis

The PT is completing general activity recommendations for a group home of young adults with emotional
and behavioral issues. All patients are chemically controlled with either antipsychotic or antidepressant
medications. Full-time supervision is available for any activity recommended. Which exercise precautions
would be important for the therapist to include?

Choices:
1. Promote rhythmic movement to soothing music to avoid agitation.
2. Promote activities with sequential movements to improve memory.
3. Avoid aerobic exercises outdoors when temperature is over 90°F.
4. Avoid games with throwing activities to prevent injuries.

Teaching Points
Correct Answer: 3
Overheating is detrimental to individuals on antipsychotic or antidepressant medications.
Examination C 797

Incorrect Choices:
Assumptions of agitation, memory deficits, or violence in such populations are discriminatory and inac-
curate. Although an individual client may demonstrate these, it is inaccurate to suggest these for a group
without further individual information.

Type of Reasoning: Inference


This question requires one to have knowledge of the adverse/side effects of antipsychotic and antidepres-
sant medications in order to arrive at a correct conclusion. In this situation, the potential adverse effect is
overheating; therefore, avoiding aerobic exercise outdoors in temperatures over 90 F is the MOST ap-
0

propriate precaution. If this question was answered incorrectly, review information on adverse effects of
anti psychotics and antidepressants.

Neuromuscular I Examination

Following an initial functional examination using the Functional Independence Measure (FIM), a patient is
found to require minimal contact assistance in transferring from sit-to-stand and bed-to-wheelchair. These
results are BEST reported as:

Choices:
1. FIM level 3; completes activity with 50% or more but less than 75% of the effort.
2. FIM levelS; completes activity with cueing.
3. FIM level 4; completes activity with 75% or more of the effort.
4. FIM level 6; completes activity with extra time.

Teaching Points
Correct Answer: 3
This patient's performance is best categorized by FIM level 4 (completes activity with minimal contact as-
sistance and 75% or more of the effort) .

Incorrect Choices:
With FIM level 3 grades, the patient requires moderate assistance and completes activity with 50% or
more but less than 75% of the effort. FIM level 5 grade specifies completes activity with cueing or standby
assistance but no manual contact. FIM level 6 (modified independence) does not permit manual contact
and allows for equipment or extra time.

Type of Reasoning: Deductive


This question requires one to recall the Functional Independence Measure (FIM) scale and to match the
scale scores to the performance indicated with this patient. This is factua l recall of knowledge, which is
a deductive reasoning skill. In this case, the patient's performance indicates minimal assist (level 4). If
answered incorrectly, review the FIM.
798 Exam ination C

Neuromuscular I Interventions

The therapist is on a home visit, scheduled at lunchtime, visiting an I8-month-old child with moderate de-
velopmental delay. The therapist notices that the child and mother are experiencing difficulties with feeding.
The child is slumped down in the highchair and is unsuccessfully attempting to use a raking grasp to lift ce-
real pieces to the mouth. Both the child and the mother are frustrated. The FIRST intervention should be to:

Choices:
1. Work on desensitizing the gag reflex.
2. Recommend that the mother return to breastfeeding for a few more months.
3. Recommend that the mother feed the child baby food instead of cereal for a few more months.
4 . Reposition the child in a proper sitting position using postural supports.

Teaching Points

Correct Answer: 4
Feeding can be successful only if the child is positioned in a stable sitting posture: Head upright, trunk
erect with pelvis neutral and hips flexed to 90 degrees, and feet resting flat. Correct positioning in sitting
will facilitate upper extremity function (grasp and release) as well as swallowing.

Incorrect Choices:
The other choices fail to address the central problem in this case, lack of postural support. Changing the
child's diet (baby food or breast milk) will not improve the functional skill of feeding.

Type of Reasoning: Inductive


One must determine what should happen FIRST when providing interventions for this child. This re-
quires inductive reasoning skill, in which clinical judgment is utilized to determine a best course of ac-
tion. For this child, the FIRST intervention should be to reposition the child in proper sitting and utilize
postural supports.

"
Musculoskeletal I Interventions

A patient is sent to physical therapy with a diagnosis of "frozen shoulder. " The MOST effective mobilization
technique for restricted shoulder abduction is:

Choices:
1. Posterior glide at 10 degrees of abduction.
2. Inferior glide at 55 degrees of abduction.
3. Inferior glide at 95 degrees of abduction.
4. Lateral glide in neutral position.

Teaching Points

Correct Answer: 2
The convex-concave rule for mobilization applies. The MOST effective position to mobilize for improved
shoulder abduction is in the resting position (55 degrees). Because the convex humeral head is moving on
the concave glenoid, an inferior glide would be MOST appropriate to improve shoulder abduction.
Examination C 799

Incorrect Choices:
A posterior glide would improve external rotation. An inferior glide at 95 degrees would be outside the
normal glenohumeral joints resting position so this technique would not be optimal. A lateral glide (ana-
tomical distraction) is a generic technique that is nonspecific for anyone motion and is, therefore, not the
best choice.

Type of Reasoning: Analysis


One must have knowledge of biomechanics and kinesiology in order to arrive at the correct conclusion.
In addition, one should understand the diagnosis offrozen shoulder (adhesive capsulitis) and appropri-
ate joint mobilization techniques. This necessitates analytical reasoning skill, in which one must interpret
the information presented and determine its precise meaning. If this question was answered incorrectly,
review joint mobilization guidelines for treatment of adhesive capsulitis.

Musculoskeletal I Evaluation, Diagnosis

A patient presents with insidious onset of pain in the jaw that is referred to the head and neck regions. As
best as the patient can recall, it may be related to biting into something hard. Cervical ROM is limited in
flexion by 20 degrees, cervical lateral flexion limited to the left by 10 degrees. Mandibular depression is 10
mm with deviation to the left, protrusion is 4 mm, and lateral deviation is 15 mm to the right and 6 mm
to the left. Based on these findings, the diagnosis for this patient would be:

Choices:
1. Capsule-ligamentous pattern of temporomandibular joint (TMJ) on the left.
2. Weak lateral pterygoids on the left.
3. Weak lateral pterygoids on the right.
4. Cervical spine and TMJ capsular restrictions on the left.

Teaching Points

Correct Answer: 1
The capsule-ligamentous pattern of the TMJ is limitation on opening, lateral deviation greater to the
uninvolved side, and deviation on opening to the involved side. Normal parameters for TMJ measures are
25 to 35 mm functional and 35 to 50 mm normal; normal protrusion is 3 to 6 mm, and normal lateral
deviation is 10 to 15 mm.

Incorrect Choices:
Weakness of the lateral pterygoids presents as deviation on protrusion to the opposite side of the muscle
weakness. A capsular pattern of the cervical spine presents as side flexion and rotation, equally limited,
and extension.

Type of Reasoning: Analysis


Questions that inquire about a potential diagnosis given a patient's symptoms necessitate analytical
reasoning skill. This is because one must weigh the information and analyze the symptoms in order to
reach a conclusion that the diagnosis is most likely capsule-ligamentous pattern ofTMJ on the left. If this
question was answered incorrectly, review information on symptoms ofTMJ.
800 Exa mination C

Nonsystem I Safety, Professional Responsibilities, Research

A mother of three is being treated for a Colles' fracture. Her husband wants to look at her medical record.
The PT should:

Choices:
1. Allow him to look at the chart because he has a spousal right to view the information .
2. Refuse to let him look at the chart because he may misinterpret the documentation.
3. Let him look at the chart and be available to answer any questions.
4. Deny access to the chart unless written permission is granted by his wife.

Teaching Points

Correct Answer: 4
The issue here is patient confidentiality. Spouses do not have access to medical information unless they
have consent of their spouse or that of a proxy because of incompetence of the other spouse. In accordance
with APTA's Guide for Professional Conduct Principle 1: PTs respect the rights and dignity of all individu-
als; and 1.2 Confidential Information C: Information derived from the working relationship of PTs shall
be held confidential by all parties. HIPAA regulations also limit access to medical record information un-
less consent is granted.

Incorrect Choices:
The husband should not be allowed to look at the chart. Not letting him look at the chart because of fear
he will misinterpret the information is not a valid reason. However, permission must be granted first.

Type of Reasoning: Evaluation


This question provides an ethical situation that requires the test taker to determine the best response, rely-
ing on knowledge of the APTA's Guide for Professional Conduct. This fo llows HIPAA regulations. Ques-
tions requiring determination of ethical guidelines and actions often require evaluative reasoning skill.

"
Cardiovascular/Pulmonary and Lymphatic I Evaluation, Diagnosis

A patient with a recent history of rib fractures suddenly becomes short of breath during a bout of coughing.
The patient looks panicked and complains of sharp pain in the left chest. A quick screen shows a deviated
trachea to the right, among other signs and symptoms. The MOST LIKELY explanation for this is:

Choices:
1. Angina.
2. Pulmonary emboli.
3. Pneumothorax.
4. Mucous plugging of an airway.

Teaching Points

Correct Answer: 3
The deviation of the trachea toward the right with the chest pain on the left is a match of symptoms for the oc-
currence of a pneumothorax on the left. The history of a rib fracture makes pneumothorax all the more likely.
Examination C 801

Incorrect Choices:
Whereas all of the pathologies listed would cause panic on the part of the patient, mucous plugging of an
airway would not cause pain. The deviation of the trachea would not result from angina or pulmonary emboli,
but would happen with a pneumothorax and lung tissue collapse (which could result from mucous plugging).

Type of Reasoning: Analysis


This question provides symptoms, and the test taker must determine a cause for them. This necessitates
analytical reasoning skilL in which one must produce the correct diagnosis based on the patient's symp-
toms. If this question was answered incorrectly, review information of symptoms of pneumothorax.

Musculoske letal I Interventions

A patient presents with supraspinatus tendinitis. After the initial cryotherapy, the therapist decides to apply
US. To effectively treat the supraspinatus tendon, the therapist should place the shoulder joint in:

Choices:
1. Slight abduction and external rotation.
2. Slight abduction and internal rotation.
3. Adduction and internal rotation.
4. Adduction and external rotation.

Teaching Points

Correct Answer: 2
Abduction and internal rotation of the shoulder places the supraspinatus tendon in a good position to
apply US by exposing the tendon from under the acromion process.

Incorrect Choices:
The other choices fail to position the supraspinatus tendon in optimal position.

Type of Reasoning: Inductive


One must utilize diagnostic and clinical judgment to determine the best course of action when providing
US for supraspinatus tendinitis. Questions that require clinical and diagnostic reasoning utilize inductive
reasoning skill.

In t egumentary I Exam ination

A patient is referred for postmastectomy rehabilitation. During the initial examination, the therapist ob-
serves an irregular area of skin on the patient's shoulder about 7 mm in diameter. The patient reports that
there has always been a mole there but is more prominent lately and that the color has changed, now rang-
ing from black to red to blue. The therapist documents this finding as a:

Choices:
1. Benign nevus.
2. Atypical dysplastic nevus.
3. Papule.
4. Wheal.
802 Examination C

Teaching Points

Correct Answer: 2
A nevus is a common mole. A changing nevus (atypical dysplastic nevus) that presents with asymmetry
(A), irregular borders (B), variations in color (C), diameter > 6 mm (D), and elevation (E) is indicative of
malignant melanoma (the "ABCDEs" from the American Cancer Society).

Incorrect Choices:
A benign nevus does not present with changes and variations in color. A papule is an elevated nevus. A
wheal is an irregular, transient superficial area of localized skin edema (e.g., hive, mosquito bite).

Type of Reasoning: Analysis


Questions that provide an array of symptoms and require the test taker to determine the likely findings
often require analytical reasoning skills. For this scenario, the key words including "irregular area" and
"variations in color" should assist the test taker in concluding this finding as an atypical dysplastic ne-
vus. If this question was answered incorrectly, review signs and symptoms of skin malignancies from the
American Cancer Society.


Neu romuscular I Evaluatio n , Diagnosis

A patient is 2 days post-left CVA and has just been moved from the intensive care unit to a stroke unit.
When beginning the examination, the therapist finds the patient's speech slow and hesitant. The patient is
limited to one- and two-word productions, and expressions are awkward and arduous. However, the patient
demonstrates good comprehension. These difficulties are consistent with:

Choices:
1. Global aphasia.
2. Dysarthria.
3. Wernicke's aphasia.
4. Broca's aphasia.

Teaching Points

Correct Answer: 4
This patient is demonstrating classic signs of Broca's aphasia (also known as nonfluent, expressive, or
motor aphasia) . It is the result of a lesion involving the third frontal convolution of the left hemisphere.
Broca's aphasia is characterized by slow and hesitant speech with limited vocabulary and labored articula-
tion. There is relative preservation of auditory comprehension.

Incorrect Choices:
Wernicke's aphasia is characterized by impaired auditory comprehension and fluent speech. Global apha-
sia is a severe aphasia with marked dysfunction across all language modalities. Dysarthria is impairment
in the motor production of speech.

Type of Reasoning: Analysis


This question provides symptoms, and the test taker must determine the diagnosis. This requires analyti-
cal reasoning skill, in which one must determine the cause for the symptoms. If this question was an-
swered incorrectly, review symptoms of Broca's aphasia.
Examination C 803

Neuromuscular I Examination

A patient recovering from traumatic brain injury (TBI) is functioning at stage IV on the Rancho Los Ami-
gos Levels of Cognitive Functioning Scale. During the therapist's initial examination, the patient becomes
agitated and tries to bite the therapist. The BEST course of action is to:

Choices:
1. Postpone the examination until later in the day when the patient calms down.
2. Postpone the examination for 1 week and then try again .
3. Document the behaviors and engage in a calming activity.
4. Restructure the formal examination so the therapist can complete it in three very short sessions.

Teaching Points

Correct Answer: 3
Patients with TBI in level IV of recovery are confused and agitated. Behavior is bizarre and nonpurposeful
relative to the immediate environment. This patient is unable to cooperate directly with formal exami-
nation or treatment, lacking both selective attention and memory. The therapist needs to observe and
document the behaviors closely and engage the patient in a calming activity such as slow rocking. A quiet,
closed environment is critical.

Incorrect Choices:
Because the patient's symptoms are expected for a person in stage IV of Rancho Los Amigos Levels of
Cognitive Functioning Scale, it is not appropriate to defer treatment or restructure the examination. The
patient's immediate needs must be addressed.

Type of Reasoning: Evaluation


One must determine the BEST course of action based on evaluating the merits of the four possible
CHOICES presented. One must ask, "Are these symptoms expected for this stage of recovery, and if so,
what would be BEST?" If this question was answered incorrectly, review Rancho stages.

"
Cardiovascular/Pulmonary and Lymphatic I Evaluation, Diagnosis

A therapist is working on a cardiac care unit in an acute care facility. After exercising a patient recovering
from a ventricular infarct, the therapist notices fatigue and dyspnea after mild activity. Later that day, on a
return visit, the therapist notices the patient has a persistent spasmodic cough while lying in bed. Their heart
rate is rapid (140), and slight edema is evident in both ankles. The patient appears anxious and agitated.
The therapist suspects:

Choices:
1. Left ventricular failure.
2. Right ventricular failure.
3. Impending MI.
4. Developing pericarditis.
804 Examination C

Teaching Points

Correct Answer: 1
Typical clinical manifestations of left ventricular failure (CHF) include those described in the case exam-
ple along with an S3 heart gallop, paroxysmal nocturnal dyspnea, orthopnea, and signs and symptoms of
pulmonary edema (marked dyspnea, pallor, cyanosis, diaphoresis, tachypnea, anxiety, and agitation) .

Incorrect Choices:
Typical clinical manifestations of right ventricular failure include dependent edema of the ankles (usually
pitting edema), weight gain, fatigue, right upper quadrant pain, anorexia, nausea, bloating, right-sided
S3 or S4, cyanosis of nail beds, and decreased urine output. Impending MI may include anginal pain
or discomfort in the chest, neck, jaw, or arms; palpitations; tachycardia; or unusual fatigue or dyspnea.
Pericarditis produces substernal pain that may radiate to neck and upper back; difficulty swallowing; pain
aggravated by movement or coughing and relieved by leaning forward or sitting upright; and a history of
fever, chills, weakness, or heart disease.

Type of Reasoning: Analysis


This question provides symptoms of cardiac issues, and the test taker must determine the likely diagno-
sis. This necessitates analytical reasoning skills; one must determine the correct diagnosis based on the
patient's symptoms. If this question was answered incorrectly, review symptoms of coronary artery disease.

Musculoskeletal I Examination

A patient with anterior knee pain has increased adduction and internal rotation at the hip when performing
a squat. The potential cause of this compensatory movement is decreased strength of the:

Choices:
1. Knee flexors and extensors.
2. Hip and knee flexors.
3. Hip adductors and internal rotators.
4. Hip abductors and external rotators.

Teaching Points

Correct Answer: 4
Decreased strength of the hip abductors and external rotators are common findings in patients with ante-
rior knee pain. Weakness is often demonstrated during a squat with increased hip adduction and internal
rotation due to the poor eccentric control of these muscles.

Incorrect Choices:
Weakness of the knee flexors and extensors would not significantly affect hip mechanics during a squat.
Weakness of the hip and knee flexors would not affect the squat as the hip and knee flexion are controlled
eccentrically by the hip and knee extensors. Weakness of the hip adductors and internal rotators would
not create this compensatory movement; rather, the opposite may occur.

Type of Reasoning: Inductive


This question requires the test taker to draw from clinical knowledge of knee pain and compensatory
patterns of movement in order arrive at a correct conclusion. This is an inductive reasoning skill. For this
situation, a patient with anterior knee pain who demonstrates hip adduction and internal rotation during
a squat is most likely compensating for weak hip abductors and external rotators. Review knee pain and
compensatory movement strategies if answered incorrectly.

Examination C 805

Metabolic/Endocrine I Evaluation, Diagnosis

A patient has been taking corticosteroids (hydrocortisone) for management of adrenocortical insufficiency
and is referred to physical therapy for mobility training after a prolonged hospitalization. Potential adverse
effects that one can expect from prolonged use of this medication include:

Choices:
1. Confusion and depression.
2. Atrophy and osteoporosis.
3. Decreased appetite and weight loss.
4. Hypotension and myopathy.

Teaching Points

Correct Answer: 2
Prolonged use of corticosteroids may result in muscle weakness, osteoporosis, fractures, and joint pain.
Large doses are associated with cushingoid changes (e.g., moon face, central obesity, hypertension, my-
opathy, electrolyte and fluid imbalance). Common central nervous system changes include insomnia and
nervousness.

Incorrect Choices:
The other choices are not expected potential adverse effects of corticosteroids.

Type of Reasoning: Inference


One must infer or draw conclusions about the expected symptoms of prolonged use of corticosteroids in
order to arrive at the correct conclusion. For this patient, one could expect atrophy and osteoporosis to
occur. If this question was answered incorrectly, review information on side effects of long-term cortico-
steroid use .


Integumentary I Evaluation, Diagnosis

A PT is treating a patient with deep partial-thickness burns over 35% of the body (chest and arms). Wound
cultures reveal a bacterial count in excess of 105/g of tissue on the anterior left arm. The therapist can rea-
sonably expect that:

Choices:
1. The risk of hypertrophic and keloid scars is low because there is no viable tissue.
2. The burn area is pain free because all nerve endings in the dermal tissue were destroyed.
3. With antibiotics, spontaneous healing can be expected.
4. The infected wound can convert the area to a full-thickness burn.

Teaching Points

Correct Answer: 4
A deep partial-thickness burn will heal in about 3 to 5 weeks if it does not become infected. An infection
typically results in conversion of the wound to a full-thickness burn.
806 Examination C

Incorrect Choices:
Full-thickness burns (not partial-thickness burns) are without sensation because the nerve endings are
destroyed. However, the area is not pain free, because adjacent areas of partial-thickness burns have intact
nerve endings and can be painful. The risk of hypertrophic and keloid scars is high ( not low) with deep
partial-thickness or full-thickness burns. With wound conversion, grafting will be necessary because all
epithelial cells are destroyed with a full-thickness burn. With an infected wound, spontaneous healing is
not expected.

Type of Reasoning: Inference


One must determine what a bacterial count of 105/g indicates in order to choose the correct solution. This
requires one to draw conclusions about the information presented, which, when inferred correctly, indi-
cates that the wound is infected and could convert the burned area to a full-thickness burn. If this question
was answered incorrectly, refer to burn care guidelines, especially infected wounds.

Nonsystem I Equipment, Devices, Therapeutic Modalities

Recently, a lO-year-old patient has begun walking with supination of the right foot. With the shoe off, the
therapist finds a new callus on the lateral side of the metatarsal head of the fifth toe. The BEST choice for
orthotic prescription is:

Choices:
1. Viscoelastic shoe insert with a forefoot medial wedge.
2. A Scaphoid pad.
3. A Thomas' heel.
4. A Viscoelastic shoe inselt with forefoot lateral wedge.

Teaching Points
Correct Answer: 4
Supination of the foot (pes cavus) is accompanied by supination of the talocalcaneonavicular (TCN), sub-
talar, and transversal tarsal joints. It is characterized by an abnormally high arch. The flexible cavus foot
generally responds well to orthotic foot control, especially in a child. The best choice is a viscoelastic shoe
insert with forefoot lateral wedge.

Incorrect Choices:
The other choices are used to control flexible pes valgus.

Type of Reasoning: Inductive


One must utilize clinical judgment in order to determine the BEST choice for orthotic prescription. This
requires one to utilize inductive reasoning skill, which also includes diagnostic thinking, in order to
determine a best course of action. If this question was answered incorrectly, review orthotic prescription
approaches for children with foot supination.
Examination C 807

Musculoskeletal I Interventions

A patient is recovering from a right total hip replacement (posterolateral incision, cementless fixation). The
MOST appropriate type of bed-to-wheelchair transfer to teach is to have the patient use a:

Choices:
1. Stand-pivot transfer to the surgical side.
2. Lateral slide transfer to the surgical side using a transfer board.
3. Stand-pivot transfer to the sound side.
4. Squat-pivot transfer to the surgical side.

Teaching Points

Correct Answer: 3
During initial healing, it is important to protect the hip from dislocation or subluxation of the prosthesis.
With a posterolateral incision, excessive hip flexion and adduction past neutral are contraindicated. This
is minimized by transferring to the sound side.

Incorrect Choices:
All other choices emphasize transfer to the surgical side, which can move the hip into adduction. In addi-
tion, the stand-pivot transfer with some hip extension is a better choice than transferring with the hip in
full flexion .

Type of Reasoning: Deductive


This question requires one to recall the appropriate guidelines for bed-to-wheelchair transfers of patients
with total hip replacements. The recall of guidelines necessitates deductive reasoning, in which factual
recall of knowledge is expected. For patients with hip replacements, transfers toward the sound side helps
to preserve hip precaution guidelines. If this question was answered incorrectly, review guidelines for
transferring patients with hip replacements .


Neuromuscular I Examination

During an initial interview and history, a patient with a right OIA seems unconcerned about obvious pa-
ralysis of the left arm and leg. When the therapist asks the patient to describe what happened, the patient
says "I must have slept wrong and my arm and leg fell asleep." The patient further tells the therapist, "My
family put me in this place so they could go on vacation." The therapist correctly identifies these responses
as evidence of:

Choices:
1. Somatoagnosia.
2. Spatial relations disorder.
3. Prosopagnosia.
4. Anosognosia.
808 Examination C

Teaching Points

Correct Answer: 4
Anosognoisa is a perceptual disorder that is characterized by denial, neglect, and lack of awareness of the
presence or severity of one's paralysis.

Incorrect Choices:
Somatoagnosia is a perceptual disorder characterized by an impairment in body scheme (a lack of aware-
ness of body structure and the relationship of body parts of oneself or of others) . Spatial relation disor-
ders encompass a constellation of impairments characterized by difficulty in perceiving the relationship
between self and two or more objects. Prosopagnosia is a perceptual disorder characterized by an inability
to recognize faces (face blindness) .

Type of Reasoning: Analysis


This question provides symptoms, and the test taker must determine the likely cause for such symptoms.
Questions of this nature often require analytical reasoning skill, where pieces of information are analyzed
to draw an appropriate conclusion. For this situation, the symptoms indicate anosognosia. Review symp-
toms of anosognosia if answered incorrectly.


Neuromuscular I Interventions

The therapist is treating a child with mild developmental delay secondary to 7 weeks prematurity at birth.
The child is now 8 months old and is just learning to sit. The BEST choice for training activity is:

Choices:
1. Standing tilting reactions.
2. Sideward protective extension in sitting.
3. Prone tilting reactions.
4. Supine tilting reactions.

Teaching Points

Correct Answer: 2
Sideward protective extension in sitting is a functional, protective reaction that normally occurs at about
the same time as sitting begins.

Incorrect Choices:
The child who is starting to sit should already have prone and supine tilting reactions. It is too early to
begin standing tilting reactions.

Type of Reasoning: Inference


One must recall the developmental milestones of infants and infer the BEST choice for training in order
to choose the correct solution. If this question was answered incorrectly, review the motor developmental
milestones of infants.
Examination C 809

Cardiovascular/Pulmonary and Lymphatic I Examination

The picture depicts a patient who is learning to take her own pulse. What is the patient doing incorrectly?

Choices:
1. Palpating the ulnar artery.
2. Assessing the pulse with the thumb.
3. Taking the pulse with the forearm supinated.
4. Palpating the right arm as opposed to the more accurate left arm.

Teaching Points
Correct Answer: 2
The thumb is pulsatile and cannot be used to assess a pulse.

Incorrect Choices:
A pulse can be assessed on any artery that can be palpated. While in some patients a pronated forearm
can facilitate palpation of the pulse, any arm position is acceptable. The circulatory system is a closed
system and therefore, in the absence of pathology, the pulse is the same in either upper limb.

Type of Reasoning: Inductive


This question requires the test taker to draw from clinical knowledge of taking a pulse in order to arrive at
a correct conclusion. This is an inductive reasoning skill. If this question was answered incorrectly, review
guidelines for taking pulse.

C101
Gastrointestinal I Interventions

A patient with complete spinal cord injury at the level ofTll is on a bowel program. The PRIMARY meth-
odology of bowel training in this case is use of:

Choices:
1. Medications such as laxatives for passive elimination.
2. Diet and medications to manage a flaccid bowel.
3. Digital stimulation of intact defecation reflexes.
4. Manual removal of stool from the rectum.
810 Examination C

Teaching Points

Correct Answer: 3
A SCI injury at the level ofT11 produces an UMN or spastic bowel with intact spinal defecation reflexes.
Bowel and anal sphincters respond to rectal/anal stimulation, enabling a planned bowel elimination
program.

Incorrect Choices:
Medications such as laxatives and stool softeners can be used to assist the patient in manually stimulated
elimination; however, the primary methodology is digital stimulation. A LMN or flaccid bowel occurs with
lesions at TI2 or below with loss of spinal defecation reflexes. Response to medications is less effective,
and manual removal of stool may be required.

Type of Reasoning: Inductive


This question requires clinical judgment in order to determine a best course of action for a patient with a
complete spinal cord injury. This necessitates inductive reasoning skills, where determining a best course
of action is often used. For this case, the patient with a TIl injury would primarily use digital stimulation
of intact defecation reflexes for bowel retraining. Refer to bowel retraining guidelines for patients with
spinal cord injury, especially upper motor neuron injury, if answered incorrectly.

C102

Musculoskeletal I Examination

When performing scoliosis screening in a school setting, the optimal age for girls to be screened is:

Choices:
l. 6-8 .
2. 12-14.
3. 15-17.
4.9-11.

Teaching Points

Correct Answer: 4
The most effective age to screen girls for scoliosis is just before the pubescent growth spurt between 9 and
11 years, when the scoliotic curve can increase dramatically. Boys should be screened between 11 and 13
years of age because of differences in the age of onset of puberty between girls and boys.

Incorrect Choices:
Screening can occur at any age, but routine screening should be performed before the pubescent growth
spurt. Large changes in abnormal spinal curves can occur during growth spurts.

Type of Reasoning: Analysis


One must analyze the merits of performing scoliosis screenings with each of the identified age groups in
girls in order to determine the age at which it is optimal to complete the screening. This requires analytical
reasoning skills, in which one interprets the various ages and how these ages are relevant to normal devel-
opment and puberty. If this question was answered incorrectly, review information on scoliosis screening.

Examination C 811

Musculoskeletal I Interventions

A patient with a confirmed left C6 nerve root compression due to foraminal encroachment complains of
pain in the left thumb and index finger. The MOST effective cervical position to alleviate this radicular pain
in weight-bearing is:

Choices:
1. Lower cervical flexion .
2. Left side bending.
3. Lower cervical extension.
4. Right rotation.

Teaching Points

Correct Answer: 1
Flexion increases the space at the intervertebral foramen, allowing the C6 nerve root to decompress and
reduce or alleviate radicular pain.

Incorrect Choices:
Left side-bending, cervical extension, and/ or right rotation all close down the foramen on the left hand
side, which will increase the compression on the nerve.

Type of Reasoning: Inference


This requires the test taker to infer how various cervical positions will result in less pain from C6 nerve
root compression. This requires knowledge of anatomy and the cervical position that will increase space
at the intervertebral foramen to alleviate pain. Evaluative reasoning skills are utilized whenever one must
make a judgment about a best course of action.

Musculoskeletal I Examination

A patient is standing with excessive subtalar pronation. The therapist examines for possible related
motions of:

Choices:
1. Tibial and femoral external rotation, with pelvic internal rotation.
2. Tibial, femoral, and pelvic internal rotation.
3. Tibial, femoral, and pelvic external rotation.
4. Tibial and femoral internal rotation with pelvic external rotation.
812 Examination C

Teaching Points

Correct Answer: 2
With the patient standing, the calcaneus is fixed to the ground. Subtalar joint pronation will occur as the
talus plantar flexes, adducts, and inverts. In response to subtalar joint pronation, obligatory internal rota-
tion of the tibia, femur, and pelvis occurs.

Incorrect Choices:
External rotation of the tibia, femur, and pelvis would be associated with excessive subtalar supination.
The pelvis must follow the rotation present in the lower limb (specifically the femur) .

Type of Reasoning: Analysis


One must analyze these motions and postures in order to determine which solution is most likely correlat-
ed with the excessive subtalar pronation. Through analytical reasoning skilL one should determine that the
tibiaL femoraL and pelvic internal rotations are the possible obligatory motions as a result of the excessive
subtalar pronation. If this question was answered incorrectly, refer to information on postures related to
excessive subtalar pronation.

Nonsyst e m I Safety, Professiona l Responsi b ili t ies, Resea rch

A patient receiving outpatient physical therapy as a result of a rotator cuff tear has achieved all goals and
returned to baseline function. The patient enjoys coming to physical therapy and participates well in treat-
ment. The patient requests to continue with therapy to maintain all achievements because their insurance
policy has unlimited physical therapy coverage. The IT should:

Choices:
l. Continue to see the patient only if the patient agrees to pay out-of-pocket.
2. Continue to see the patient because physical therapy will be reimbursed by the insurance company.
3. Decrease the frequency of treatment to one time per week.
4. Discharge the patient from physical therapy and do not see the patient under any circumstances.

Teaching Points

Correct Answer: 1
ITs are obligated to provide therapy services that are indicated according to patient presentation and the ex-
amination findings. The Standards of Practice indicate that once the patient's goals are met and the patient
has returned to baseline, he/she should be discharged from skilled therapy. The fact that there is unlimited
physical therapy coverage should not be a factor in this clinical decision. As long as the IT explains that the
therapy is not medically necessary, the patient may still decide that therapy is beneficial to him/her and that
he/she must pay out-of-pocket. The Code of Ethics obligates ITs to be responsible stewards of health care
resources and avoid both underutilization and overutilization of services. It would not be considered
overutilization of services if the patient was informed of the nature and need of the continued services.

Incorrect Choices:
Even though the insurance would cover continued therapy, it would be unethical for the IT to provide ser-
vices because this would be overutilization of resources paid by the insurance company. The therapist could
continue to see the patient in an ethical manner as long as it was explained that the therapy was not neces-
sary, although the patient chooses to continue treatment. By decreasing the therapy to one time per week, it
would still be overutilization because it would be presented to the insurance company as needed services.
Examination C 813

Type of Reasoning: Evaluation


The question requires one to recognize the issues of overutilization of services and the unethical behavior
in doing so, which is an evaluative reasoning skill. In order to answer the question correctly, the test taker
must first identify that skilled physical therapy is no longer indicated and then must know the ethical op-
tions for honoring the patient's request for continued therapy. If this question was answered incorrectly,
review the Code of Ethics Principle 8 (C) .

Nonsystem I Equipment, Devices, Therapeutic Modalities

A therapist is applying a symmetrical biphasic pulsed current to the vastus medialis tQ improve patellar
tracking during knee extension. The patient complains that the current is uncomfortable. To make the
current more tolerable to the patient, yet maintain a good therapeutic effect, the therapist should consider
adjusting the:

Choices:
l. Pulse rate.
2. Current intensity.
3. Pulse duration.
4 . Current polarity.

Teaching Points
Correct Answer: 3
Decreasing the pulse duration reduces the electrical charge of each pulse, making the current more com-
fortable by decreasing the total current applied while maintaining the full therapeutic effect.

Incorrect Choices:
The only other parameter that would have a direct effect on comfort would be the intensity. For motor lev-
el stimulation, decreasing the intensity would decrease the therapeutic effect by decreasing the quality of
the contraction. For sensory level stimulation, decreasing the intensity would decrease the level of sensory
input needed for the treatment of pain. The pulse rates used for pain management are typically modu-
lated. For motor level stimulation, decreasing the pulse rate would decrease the quality of the contraction,
and increasing the pulse rate could make it more uncomfortable or contribute to muscle fatigue. Changing
the polarity would have no effect because a symmetrical biphasic waveform has no net polarity.

Type of Reasoning: Deductive


This question requires one to factually recall the correct parameters for high-volt pulsed current and how
adjustments to the pulse or current results in alterations in therapeutic effect. Factual recall or protocols
or guidelines is a deductive reasoning skill. If this question was answered incorrectly, review information
on guidelines for high-volt pulsed current.
814 Exa mination C

Gastrointestinal I Evaluation, Diagnosis

Which of the following gastrointestinal sources of pain can refer to the shoulder?

Choices:
1. Esophageal pain.
2. Spleen or diaphragmatic pain.
3. Colon or appendix pain.
4. Gallbladder pain.

Teaching Points

Correct Answer: 2
Spleen or diaphragmatic pain can refer to the shoulder.

Incorrect Choices:
Esophageal pain can refer to the mid-back, head, or neck. Colon or appendix pain can refer to the lower
back, pelvis, or sacrum. Gallbladder pain can refer to the mid-back and scapular regions.

Type of Reasoning: Deductive


One must recall the typical referral patterns for gastrointestinal pain in order to determine which refers to
the shoulder. This requires deductive reasoning skills, because factual recall of guidelines is used to reach a
conclusion. If this question was answered incorrectly, review gastrointestinal pain referral patterns .

C108

Neuromuscular I Evaluation, Diagnosis

A patient presents with symptoms of uncoordinated eye movements, profound gait and trunk ataxia, and
difficulty with postural orientation to vertical. Balance deficits are pronounced in standing with eyes open
and eyes closed. Examination of the extremities reveals little change in tone or coordination. The therapist
suspects involvement of the:

Choices:
1. Spinocerebellum.
2. Basal ganglia.
3. Premotor cortex.
4. Vestibulocerebellum.

Teaching Points
Correct Answer: 4
The symptoms are suggestive of cerebellar dysfunction. The vestibulocerebellum (archicerebellum) is
concerned with adjustment of muscle tone in response to vestibular stimuli. It coordinates muscle actions
to maintain postural coordination and balance control along with eye muscle control (all impaired in this
example).
Examination C 815

Incorrect Choices:
The spinocerebellum (paleocerebellum) controls muscle tone and synergistic movements of the extremi-
ties on the same side of the body. The basal ganglia's functions are complex. It contributes to motor
planning and sequencing of voluntary movements. Deficits typically include rigidity, bradykinesia, and
tremor along with postural instability. The premotor cortex (precentral area, frontal lobe) stores programs
of motor activity assembled as the result of past experience and programs the motor cortex for voluntary
movements. It assists the basal ganglia in control of coarse postural movements.

Type of Reasoning: Analysis


This question provides the symptoms, and the test taker must determine, through analytical reasoning, the
cortical region responsible for the symptoms. If this question was answered incorrectly, review informa-
tion on cerebellar dysfunction .


System Interactions I Evaluatio n, Diagnosis

A patient with a lO-year history of diabetes complains of cramping, pain, and fatigue of the right buttock
after walking 400 feet or climbing stairs. When the patient stops exercising, the pain goes away immediately.
The skin of the involved leg is cool and pale. The therapist checks the record and finds no mention of this
problem. The therapist suspects:

Choices:
1. Spinal root impingement.
2. Peripheral nerve injury.
3. Peripheral arterial disease (PAD) .
4. Raynaud's phenomenon.

Teaching Points
Correct Answer: 3
Intermittent claudication, often the earliest indication of PAD, is manifested by cramping, pain, or fatigue
in the muscles during exercise that is typically relieved by rest. The calf muscle is most commonly affected,
but discomfort may also occur in the thigh, hip, or buttock. Cessation of pain immediately upon stopping
the exercise is characteristic of intermittent claudication, not other spinal problems. With severe disease,
however, pain may be present even at rest.

Incorrect Ch oices:
The pain associated with spinal root impingement (nerve pain) is often acute and becomes worse or
aggravated by extension, side flexion, rotation, standing, walking and exercise in general. It is relieved
by lying down. Peripheral nerve injury presents with sensory and motor loss. Raynaud's phenomenon is
an intermittent attack of pallor or cyanosis of the small arteries and arterioles of the fingers as a result of
inadequate blood flow.

Type of Reasoning: Analysis


This is a question that provides the symptoms and one must determine the likely diagnosis. This requires
analytical reasoning skill, in which the test taker utilizes knowledge of anatomy, physiology, and pathol-
ogy to determine the cause. If this question was answered incorrectly, review information on symptoms of
PAD.
816 Exa mination C

C110
Cardiovascular/Pulmonary and Lymphatic I Evaluation, Diagnosis

A patient with lower back pain has marked elevation of BP and complains of mild to severe mid-abdominal
pain that increases upon exertion. Palpation reveals a pulsing mass in the lower abdomen. The therapist
sh ould:

Choices:
1. Instruct in relaxation exercises because a pulsating mass is not unusual with hypertension.
2. Discontinue treatment and notify the patient's physician immediately.
3. Instruct the patient to contact his/her physician at the conclusion of therapy.
4. Provide hot packs to the abdomen to help relieve the muscle spasm.

Teaching Points

Correct Answer: 2
This patient is demonstrating signs and symptoms of aortic aneurysm. Pain is intermittent or constant and
can be felt in the mid-abdominal or lower back regions. The pulsating mass is highly significant, and the
level of hypertension dramatically increases risk of rupture. This is a serious medical condition; the thera-
pist should notify the physician immediately.

Incorrect Choices:
The therapist should not rely on the patient to contact the physician. All physical therapy intervention
sh ould cease.

Type of Reasoning: Evaluation


One must evaluate the significance of the information presented, determine the root cause of the symp-
toms, and then determine the best course of action. This requires evaluative reasoning skilL in which one
evaluates the merits of the fo ur possible choices and determines which approach is most appropriate
and safe for the patient. In this case, it is important to discontinue treatment and immediately notify the
physician .


Musculoskeletal I Examination

A patient presents for the initial examination with an acute and painful shoulder impingement. During the
examination, the PT finds significantly increased muscle guarding around the shoulder girdle with difficulty
in accurately assessing joint mobility. Which manual therapy technique is the BEST option to use to assist
in performing a proper assessment?

Choices:
1. Maitland grade III inferior glide to the glenohumeral joint.
2. Maitland grade II oscillation to the glenohumeral joint.
3. Maitland grade IV inferior glide to the glenohumeral joint.
4. Maitland grade III posterior glide to the glenohumeral joint.
Examination C 817

Teaching Points

Correct Answer: 2
Maitland grades I and II are used to improve joint lubrication/nutrition as well as to decrease pain and
muscle guarding. A grade II oscillation will promote muscle relaxation, allowing a proper assessment of
the patient's joint mobility.

Incorrect Choices:
Decreasing the muscle guarding is the first priority before more vigorous interventions are employed. Mait-
land grades III and IV are used to stretch tight muscles, capsule, and ligaments in order to improve motion.

Type of Reasoning: Inference


One needs to be aware of the indication for each of the five grades of Maitland's oscillatory techniques and
determine the technique that will have the best outcome in order to arrive at a correct conclusion. This
requires one to determine what is likely to be true of a situation, which is an inferential reasoning skill .


System Interactions I Evaluation, Diagnosis

A frail, elderly wheelchair-dependent resident of a community nursing home has a diagnosis of organic
brain syndrome, Alzheimer's type, stage 2. During the therapist's initial interview, the patient demonstrates
limited interaction and mild agitation and keeps trying to wheel the chair down the hall. Because it is late
in the day, the therapist decides to resume the examination the next morning. The patient is most likely
exhibiting:

Choices:
1. Frustration because of an inability to communicate.
2. Disorientation to time and date.
3. Sundowning behavior.
4. Inattention as a result of short-term memory loss.

Teaching Points

Correct Answer: 3
A patient with stage 2 Alzheimer's disease can be expected to exhibit impaired cognition and abstract
thinking, sundowning (defined as extreme restlessness, agitation, and wandering that typically occurs
in the late afternoon), inability to carry out activities of daily living, impaired judgment, inappropriate
social behavior, lack of insight, repetitive behavior, and a voracious appetite.

Incorrect Ch oices:
Inability to communicate is characteristic of stage 3. Short-term memory loss and disorientation to time
and date are early and persistent signs of the disease.

Type of Reasoning: Analysis


This question provides the symptoms, and the test taker must determine, through analytical reasoning,
the most likely cause for the patient's behavior. In this situation, the symptoms suggest that the patient
with Alzheimer's disease is exhibiting sundowning behavior. If this question was answered incorrectly,
review information on Alzheimer's disease and sundowning behavior.

818 Examination C

C113
Musculoskeletal I Interve ntions

To increase the step length of a patient with a right transfemoral amputation who is taking an inadequate
step with the limb, the therapist should:

Choices:
1. Provide posterior-directed resistance to the right ASIS during stance.
2. Facilitate the gluteals with tapping over the muscle belly.
3. Provide posterior-directed resistance to the left ASIS during swing.
4. Provide anterior-directed resistance to the right PSIS during swing.

Teaching Points

Correct Answer: 1
Light resistance and stretch applied to the pelvis (right ASIS) in a posterior direction during mid-stance to
late stance will facilitate forward pelvic rotation on that side and enhance forward movement of the limb
during swing.

Incorrect Choices:
Anterior-directed resistance functions to pull the hip forward but does little to facilitate active forward
limb movement. The gluteals function to stabilize the limb during stance (not advance the limb forward).
Manual resistance applied to the pelvis during swing may interfere with stepping.

Type of Reasoning: Inference


One must infer the best approach to providing directed resistance for the patient with a transfemoral am-
putation. This requires knowledge of kinesiology and therapeutic exercise in order to arrive at the correct
conclusion .


G a s t roi n tes ti nal I Evaluation , Diagnosis

An elderly, frail resident of an extended care facility has intractable constipation. During a scheduled visit
from the PT, the patient complains of abdominal pain and tenderness. The therapist recognizes the patient
may experience pain in the:

Choices:
1. Low back and front of the thigh to the knee.
2. Buttock, thigh, and posterior leg.
3. Medial thigh and leg.
4. Anterior hip, groin, or thigh region.

Teaching Points

Correct Answer: 4
Intractable constipation (obstipation) can cause partial or complete bowel impaction, pain, and tender-
ness in the lower abdomen. Referred pain is to the anterior hip, groin, or thigh region.

Incorrect Choices:
Pain in the back and front of thigh to knee is characteristic of L2 nerve root compression. Pain in the but-
tock, thigh, and posterior leg is characteristic of S1 nerve root compression. Pain in the bladder can refer
to the medial thigh and leg.
Examinatio n C

Type of Reasoning: Inference


One must infer or draw a reasonable conclusion about the symptoms a client is likely to experience given
the diagnosis provided. Inferential reasoning skills are often utilized in cases in which one must deter-
mine what may be true of a patient. If this question was answered incorrectly, review signs and symptoms
of intractable constipation and pain referral patterns from the viscera .


Musculoskeletal I Int erventions

An adult patient is diagnosed with thoracic outlet syndrome. The patient presents with guarding in the up-
per trapezius and scalene muscles. Given this situation, which technique would be the MOST EFFECTIVE
way to decrease the muscle guarding and provide pain relief?

Choices:
1. Maitland grade II mobilization of the atlanto-axial joint.
2. Maitland grade III mobilization of the C6-C7 joint.
3. Maitland grade V manipulation of the first rib.
4. Maitland grade V manipulation of the C6-C7 joint.

Teaching Points

Correct Answer: 3
One needs to be aware that the muscle guarding pattern present with this condition is increased tone of
the scalene muscles. These muscles (anterior and middle portions) attach to the first rib. Grade V manipu-
lation is used to decrease muscle guarding and pain in the target area.

Incorrect Choices:
The atlanto-axial and C6-C7 joints are not the target joints of most importance nor is the technique the
most effective based on the pattern presented.

Type of Reasoning: Inference


One needs to be aware of the indication for each of the five grades of Maitland's oscillatory techniques in
order to determine the correct conclusion. Questions that require one to infer what will result in a best
clinical outcome often require inferential reasoning skill. For this scenario, Maitland grade V manipula-
tion of the first rib is best. Review Maitland's oscillatory techniques if answered incorrectly.

Musculoskeletal I Interventions

A patient presents with decreased motion at the occipitoatlantal joint (OA). The PTwants to use the princi-
ples of coupled motions that occur in that area of the spine during manual therapy techniques. In order to
improve OA mobility, when the occiput is side bent to the right, the therapist should mobilize C1 into:

Choices:
1. Flexion.
2. Extension.
3. Rotation to the right.
4. Rotation to the left.
820 Examination C

Teaching Points

Correct Answer: 4
Given the rules of coupled movement in the upper cervical spine, when the occiput is sidebent into one
direction, Cl rotates into the opposite direction. Sidebending and rotation occur in the same direction
from C2 to C7 regardless if the spine is in flexion or extension.

Incorrect Choices:
The other choices do not represent opposite directions or coupled movement from a right side bend of
the occiput at Cl.

Type of Reasoning: Inductive


One must understand the coupled movements of the cervical spine in order to arrive at a correct conclu-
sion. Questions of this nature often require inductive reasoning skill, as one must use clinical judgment to
determine a best course of action. For this situation, the therapist should mobilize into rotation to the left
when the occiput is side bent to the right. Review biomechanics and coupled movements of the cervical
spine if answered incorrectly.

C117
Cardiovascular/Pulmonary and Lymphatic I Examination

An apparently healthy individual has several risk factors for coronary artery disease. The client is interested in
improving overall fitness and cardiac health. After an exercise tolerance test, which was asymptomatic, the
client is referred for an exercise class. The MOST accurate measure of exercise intensity to monitor during the
FIRST exercise session is:

Choices:
1. Rating of perceived exertion (RPE).
2. MET level.
3. Respiratory rate.
4 . Heart rate (HR) .

Teaching Points
Correct Answer: 4
An exercise tolerance test should be performed before commencing an exercise program for all high-risk
individuals. The best measurement of exercise intensity in a newly tested and exercising individual is HR.

Incorrect Choices:
RPE will become a valuable measurement tool once the patient becomes adept at using it, but it would
not be reliable for the first exercise session. MET level is more of a measurement of workload, not an accu-
rate measurement of an individual's response to exercise. Respiratory rate is not used to prescribe exercise
intensity.

Type of Reasoning: Inductive


This question requires the test taker to recall the various measures of exercise intensity and determine,
through clinical judgment, which measure will MOST accurately monitor the patient's exercise intensity,
given his/her diagnosis and symptoms on the FIRST visit. Questions that require clinical judgment and
diagnostic reasoning require inductive reasoning skill. If this question was answered incorrectly, review
measures to monitor exercise intensity for at-risk individuals.
Examination C 821

Neuromuscular I Examination

A mother brings her 8-week-old infant to be examined at early intervention because she noticed that the
infant was taking steps in supported standing at 2 weeks, but is not able to do it now. The therapist should:

Choices:
1. Recommend that a full developmental examination be performed by the early intervention team.
2. Explain that this is normal and that the stepping was a newborn reflex that has gone away.
3. Recommend that the mother bring the infant to a pediatric neurologist.
4. Explain this was due to a stepping reflex that will re-emerge around 10 months.

Teaching Points
Correct Answer: 2
The mother probably saw the neonatal stepping reflex, which is normal in a newborn but is not exhibited
in the older infant probably because of anthropomorphic factors and neural maturation. The age that this
reflex typically disappears is 2-3 months of age.

Incorrect Choices:
In most infants, pull-to-stand emerges at 8-9 months, whereas unassisted standing and walking occurs
at 10 to 15 months. Stepping at this age is not the result of a reflex (does not reemerge later on) . A full
developmental examination or referral to a neurologist is not indicated, and should be performed only
after the child is older and is not walking. It is important to remember that these norms are averages, and
that children may be more advanced or slower in reaching these milestones.

Type of Reasoning: Evaluation


One must have firm knowledge of the developmental milestones of infants in order to arrive at the cor-
rect conclusion. Using evaluative reasoning, one must evaluate the merits of the four possible choices
and determine which solution seems most reasonable, given the infant's symptoms and behaviors. In this
case, the behaviors are normal. If this question was answered incorrectly, review the neonatal stepping
reflex and developmental milestones for walking .


Musculoskeletal I Examination

A patient complains of pain with mouth opening that makes it difficult to eat foods that require chewing.
Examination revealed active mouth opening to be within normal limits of:

Choices:
l. 15-24 mm.
2.35-44 mm.
3.50-6 4 mm.
4.65-74 mm.

Teaching Points

Correct Answer: 2
Average AROM is approximately 35-50 mm. However, only 25-35 mm of opening between the teeth is
required for normal everyday activity.
822 Examination C

Incorrect Choices:
The TMJs are considered hypo mobile if < 25 mm of opening is achieved. Hypermobility would include
values> 50 mm.

Type of Reasoning: Analysis


One must recall normal AROM of the TMJ as well as functional AROM ranges in order to arrive at the
correct conclusion. Through analytical reasoning, one must review each of the AROM parameters and
determine which range seems most reasonable, given one's knowledge of normal ROM guidelines for the
TMJ. If this question was answered incorrectly, review AROM guidelines for the TMJ .


Musculoskeletal/Examination

The physical therapist is examining the muscle length of the patient's left hip and knee. The muscle length
test pictured below is the:

Choices:
l. Ober test.
2. Thomas test.
3. Noble test.
4. FABER test.

Teaching Points

Correct Answer: 2
The Thomas test is utilized to test for hip flexor length and to distinguish between one joint and two joint
hip flexor tightness. With low back and sacrum flat on the table, a normal one joint hip flexor length
would be with thigh flat on the table. Normal two joint hip flexor length would be 80 degrees of knee
flexion .

Incorrect Choices:
The Ober test is performed for tightness of the tensor fascia latae and iliotibial band. The Noble test is
performed for iliotibial band friction syndrome. The FABER test is hip flexion, abduction, and external
rotation and is to screen hip and SI pathology.

Type of Reasoning: Analysis


One must understand the tests described in the question in order to choose the correct one. Through ana-
lytical reasoning, the test taker must determine how each test is performed and for what purpose. If this
question was answered incorrectly, review information on special tests.

Exa mination C 823

Musculoskeletal I Evaluation, Diagnosis

A patient presents with complaints of tingling and paresthesias in the median nerve distribution of the right
forearm and hand. The fo llowing tests were found negative bilaterally: Adson's, hyperabduction, costocla-
vicular, Phalen's, and the ulnar nerve Tinel's sign. Based on this information, the diagnosis that is likely is:

Choices:
l. Thoracic outlet syndrome (TOS).
2. Pronator teres syndrome.
3. Carpal tunnel syndrome.
4. Ulnar nerve entrapment.

Teaching Points

Correct Answer: 2
All of these special tests are used to determine neurological compromise of the lower trunk and brachial
plexus. Special tests to rule out pronator teres syndrome are (1) passive supination to elongate the prona-
tor, which is tight (this would compress the nerve at that level); and (2) active resistance of pronation,
which would compress the nerve as it courses through the pronator muscle belly.

Incorrect Choices:
A negative Adson's test, hyperabduction test, and costoclavicular test will rule out TOS. A negative Pha-
len's test will rule out carpal tunnel syndrome. A negative ulnar nerve Tinel's sign will rule out ulnar nerve
entrapment. By process of elimination, pronator teres syndrome is the only diagnosis remaining.

Type of Reasoning: Analysis


One must understand the tests described in the question in order to choose the correct solution. Through
analytical reasoning, the test taker must determine what each test indicates (by a negative result) in order
to determine the diagnosis that is likely. If this question was answered incorrectly, review information on
provocative testing for pronator teres syndrome .

C122

Musculoskeletal I Evaluation, Diagnosis

A young adult patient is referred to outpatient physical therapy for an insidious onset of thoracic spine stiff-
ness and mild pain. The patient reports a great deal of difficulty moving the mid-back region in the morn-
ing and states that this has become progressively worse over the past 6 months. The neurological exam is
negative. Based on this clinical scenario, the patient is exhibiting symptoms of:

Choices:
1. Lyme disease.
2 . Pneumothorax.
3. Thoracic compression fracture.
4. Ankylosing spondylitis.
824 Examination C

Teaching Points

Correct Answer: 4
Ankylosing spondylitis is associated with a traumatic, insidious onset of thoracic pain and/or pain with
an increased duration of morning stiffness being the primary symptom.

Incorrect Choices:
Lyme disease may be associated with flu-like and/or neurological signs and symptoms in early stages. Fur-
thermore, there was no report of the common "red or bull's eye rash" frequently associated with Lyme dis-
ease. Pneumothorax is unlikely based on lack of reporting recent sickness, infection, or trauma, and onset
was not insidious. Thoracic compression fracture is unlikely based on age of patient and lack of trauma.

Type of Reasoning: Analysis


This question requires the tester to critically evaluate key findings from the patient interview and deter-
mine the most likely differential diagnosis based on the information provided. Questions of this nature
often require analytical reasoning skilL where pieces of information are analyzed to draw reasonable con-
clusions. For this case, the symptoms are indicative of ankylosing spondylitis. Review signs and symptoms
of ankylosing spondylitis if answered incorrectly.

C123
Cardiovascular/Pulmonary and Lymphatic I Interventions

When prescribing exercise for a young patient with primary lymphedema of the lower extremity, the thera-
pist should initially have the patient begin with:

Choices:
1. Brisk walking.
2. Step aerobics.
3. Running no more than 3 miles at a time.
4. Soccer or other sports that the patient enjoys.

Teaching Points

Correct Answer: 1
Initially, the patient should begin with beneficial low risk-activities (e.g., lymphedema exercise, walking,
easy biking, swimming, water aerobics, or Tai Chi) . Exercise should always be performed with a compres-
sion garment or compression bandages.

Incorrect Choices:
The patient should avoid medium-risk activities (e.g., jogging, running, or stair climbing machines) or
sport activities that involve high risk of injury (e.g., soccer, tennis, golf volleyball, or karate). Eventually,
patients can progress to certain higher risk activities provided there is no exacerbation of their lymph-
edema.

Type of Reasoning: Inductive


This question requires one to use clinical judgment in order to draw a reasonable conclusion for a patient
with primary lymphedema. For this case, it is best for the patient to begin exercise with a low-risk activ-
ity such as brisk walking. If answered incorrectly, review exercise guidelines for patients with primary
lymphedema.

Examination C 825

Neuromuscular I Interventions

An elderly patient is recovering from a right 01A and demonstrates strong spasticity in the left upper ex-
tremity. The therapist wants to reduce the expected negative effects of spasticity in the left upper extremity
while the patient is working on sitting control. The BEST position for the upper extremity is:

Choices:
1. Affected upper extremity extended and internally rotated, with the hand at the side.
2 . Left elbow flexed with arm resting on supporting pillow, positioned on the patient's lap.
3. Left shoulder adducted and internal rotation, with arm extended and hand resting on the thigh.
4. Left shoulder abducted and externally rotated, with elbow extended and weight supported on the palm
of the hand.

Teaching Points

Correct Answer: 4
In the upper extremity, spasticity is typically strong in scapular retractors, shoulder adductors, depres-
sors, and internal rotators; elbow flexors and forearm pronators; and wrist and finger flexors. The patient
should be positioned opposite the expected pattern.

Incorrect Choices:
The other choices all emphasize one or more of the expected spastic muscles/ pattern (i.e., internal rota-
tion, elbow flexion, shoulder adduction, and internal rotation).

Type of Reasoning: Inference


One must recall the patterns of spasticity in the body after CVA in order to arrive at the correct solution.
This requires inferential reasoning skill, in which the test taker must draw conclusions from the evidence
presented and determil)e the possible outcomes from implementing the above positions for this patient.
If this question was answered incorrectly, review spasticity after CVA and positioning strategies .


Neuromuscular I Examination

If the subject's vision is blocked, either by having the subject close the eyes or by placing a barrier between
the part being tested and the subject's eyes, the therapist can effectively examine:

Choices:
1. Somatosensory integrity.
2. Vestibular/visual/somatosensory integration.
3. Discriminative touch and fast pain, but not proprioception.
4. Conscious proprioception, but not discriminative touch.

Teaching Points

Correct Answer: 1
The term "somatosensation" refers to conscious relay pathways for discriminative touch, conscious pro-
prioception, fast pain, and discriminative temperature. Sensory examination must rule out vision in order
to establish the reliability of sensory testing.
826 Examination C

Incorrect Choices:
Vestibular/visual/somatosensory integration can be established only by a series of tests that include both
eyes open and eyes closed and by using flat and compliant (foam) or moving surfaces. The other choices
exclude one of the sensory sensations that should be tested with the eyes closed.

Type of Reasoning: Inference


One must infer why blocking a subject's vision is important for certain testing protocols in order to arrive
at the correct conclusion. Of all the named functions, somatosensory integrity requires the blocking of vi-
sion in order to effectively determine that the pathway is intact. Inferential reasoning is utilized because the
test taker must infer why absence of vision is important to determining the functioning of each pathway.

C126

Musculoskeletal I Examination

An adolescent felt a "clunk" in the lumbar spine 2 weeks ago while lifting weights. There was immediate
right lumbar pain and spasm. Posteroanterior and bilateral radiographic views of the lumbar spine were
normal except L4 was shifted approximately 1 mm anterior to L5 on the lateral views. Which of the follow-
ing imaging techniques would give the PT the best information regarding a diagnosis and formulating a
plan of care for this individual?

Choices:
1. Posteroanterior T1 magnetic resonance imaging (MRI).
2. Posteroanterior computed tomography (CT) scan.
3. Right oblique radiograph.
4. Bilateral oblique radiographs.

Teaching Points
Correct Answer: 4
The clinician should suspect a spondylolisthesis. A spondylolisthesis is a forward slippage of a vertebra
due to a bilateral defect in the pars interarticularis. Causes include congenital, acute fracture, or degenera-
tive conditions. The degree of forward slippage is graded on a 1-4 scale (4 being the most severe or a
75%-100% slippage) from the lateral view. It is unclear from the description whether there is an actual
spondylolisthesis. Bilateral oblique views with a radiograph are needed to see whether there is a fracture
at the pars interarticularis bilaterally. This is known as the "Scottie dog" defect (see appendix in Chapter 1
for example).

Incorrect Choices:
A unilateral view would provide information only about one pars interarticularis. Although a CT scan
and MRI might add some benefit, the posteroanterior view with these expensive techniques has no added
benefit. Depending upon the pathology or presentation of a client, a PT should be able to recommend
appropriate medical imaging to ensure accurate and effective patient/client treatment and management
decisions.

Type of Reasoning: Inference


This question requires one to infer or draw a reasonable conclusion about imaging techniques that would
provide the best information about a diagnosis and developing a plan of care. This requires inferential
reasoning skill. For this scenario, the best imaging technique would be bilateral oblique radiographs for
suspected spondylolisthesis. If this question was answered incorrectly, review symptoms of spondylolis-
thesis and radiographic imaging.
Exa mination C 827

Cardiovascular/Pulmonary and Lymphatic I Examination

A 28-year-old professional mountain bike rider presents to cardiac rehabilitation after suffering a myocar-
dial infarction 8 weeks ago . Which exercise test would be MOST beneficial in order to assess aerobic capac-
ity in this patient in order to create an aerobic conditioning program?

Choices:
1. Bruce protocol.
2. Lower extremity ergometry.
3. Six Minute Walk test.
4. Step test.

Teaching Points

Correct Answer: 2
The patient is well-trained on a mountain bike and will benefit from LE ergometry testing and a bike
training program to facilitate return to his role.

Incorrect Choices:
The Bruce Protocol and step test would provide an appropriate challenge for this patient, however would
not appropriately assess his training effects. A Six Minute Walk test will not allow the patient to exercise at
an intensity appropriate enough to assess his aerobic capacity.

Type of Reasoning: Analysis


One must understand the exercise tests described in the question in order to choose the correct one.
Through analytical reasoning, the test taker must determine how each test is performed and for what
purpose. If this question was answered incorrectly, review information on modes and purposes of exercise
tests .

C128

Neuromuscular I Evaluation, Diagnosis

A patient recovering from stroke has been using a bilateral exerciser (UBE) to strength en muscles in the
affected right upper extremity. The patient is now experiencing burning pain in the shoulder that worsens
when the limb is touched or moved. Paresthesias and pitting edema in the dorsum of the hand are also
present along with painful and diminished ROM of the wrist and fingers . The therapist's BEST course of
action is to:

Choices:
1. Discontinu e exercise and use ice for pain relief.
2. Switch to interval exercise and lower the resistance on the UBE.
3. Discontinue UBE exercise and splint the hand and wrist until pain and swelling disappear.
4. Discontinue UBE exercise and initiate elevation, massage, and active assistive ROM .
828 Examination C

Teaching Points
Correct Answer: 4
This patient presents with the classic signs and symptoms of early-stage complex regional pain syndrome
(CRPS), type I (reflex sympathetic dystrophy) . Interventions are chosen to quiet the sympathetic nervous
system and promote pain relief. Edema can be successfully managed with a combination of elevation,
massage, and compression bandaging. Restoring ROM (AROM) is also important in the treatment of
CRPS.

Incorrect Choices:
Immobilization through splinting is contraindicated because the joints will stiffen with lack of ROM. Ther-
motherapy is more beneficial than cryotherapy for pain relief because there is no accompanying sympa-
thetic nervous system stimulation as there is with cold. Altering the exercise prescription while continuing
with resistive exercise can worsen the problem.

Type of Reasoning: Inference


The test taker must first infer the reason for the patient's symptoms and then determine what the BEST
course of action is to effectively manage those symptoms. If this question was answered incorrectly, review
intervention guidelines for CRPS .


Metabolic/Endocrine I Interventions

A patient with hypothyroidism and poor drug compliance is referred to physical therapy following a fall .
During exercise, the therapist should be alert for exercise-induced:

Choices:
1. Sinus tachycardia and arrhythmias.
2. Myalgia and weakness.
3. Elevated cardiac output.
4. Paresthesias of the lower limbs.

Teaching Points
Correct Answer: 2
Hypothyroidism results in a decreased metabolic rate and is likely to produce exercised-induced myalgia
and weakness (rhabdomyolysis).

Incorrect Choices:
Sinus bradycardia (not tachycardia) and decreased cardiac output (not increased) can occur. Sensory
changes are not found.

Type of Reasoning: Inference


This question provides a diagnosis, and the test taker must determine the symptoms that could occur dur-
ing exercise based on this diagnosis. This is an inferential reasoning skill. For this situation, the therapist
should watch for signs of myalgia and weakness during exercise. If answered incorrectly, review exercise
guidelines for patients with hypothyroidism.
Exa min ati o n C 829

Nonsystem I Safety, P r ofess ional Respons ibil ities, Research

A group of researchers (the Ottawa Panel) utilized meta-analysis to identify the evidence for aerobic fitness
exercises in the management of fibromyalgia. Thirteen randomized, controlled trials (RCfs) and three con-
trolled clinical trials (cohort studies and case control studies) were selected. The main difference between
the two types of trials is the:

Choices:
1. Length of the studies.
2. Use of multiple centers versus single center trials.
3. Duration of the studies.
4. Use of randomization of subjects.

Teaching Points

Correct Answer: 4
The main difference between the two types of trials is randomization of subjects into experimental and
control groups (RCf) .

Incorrect Choices:
Meta-analysis involves the combining of a series of independent, previously published studies of similar
purpose to yield a larger target population. RCfs are used and can be either single-center or multiple-
center trials. A cohort study is a prospective study involving a group of participants with a similar condi-
tion. Comparison is made with a matched group that does not have the condition. Duration and length
of studies are not distinguishing factors between the two types of studies.

Type of Reasoning: Deductive


One must recall the guidelines for both RCfs and controlled clinical trials in order to arrive at a correct
conclusion. Recalling such guidelines is factual information, which requires deductive reasoning skills. If
this question was answered incorrectly, review types of research designs, especially RCfs and controlled
clinical trials.

C131
Neuromuscular I Interventions

A patient with a spinal cord injury at the level ofTl (ASIA A) is in the community phase of mobility train-
ing. In order for the patient to navigate a 4-inch-height curb with the wheelchair, the therapist tells the
patient to:

Choices:
1. Descend backward with the trunk upright and arms hooked around the push handles.
2. Ascend backward with the large wheels first.
3. Lift the front casters and ascend in a wheelie position.
4. Place the front casters down first during descent.
830 Examination C

Teaching Points

Correct Answer: 3
Curbs are ascended in the wheelie position (front casters lifted and moving first). Individuals must learn
to use momentum and a strong push to elevate the front casters of the chair (wheelie position) and propel
the wheelchair up the curb.

Incorrect Choices:
Patients can descend backward but must use a tucked forward lean position in order to prevent falling
backward or they can descend forward using a wheelie position, large wheels landing first. Curbs are not
ascended backward or descended with the front casters touching first .

Type of Reasoning: Evaluation


Evaluative reasoning is important to arriving at the correct conclusion for this question, because the test
taker must evaluate the merits of the four choices and then determine what is safe and appropriate for
this patient. If this question was answered incorrectly, review curb negotiation guidelines for patients with
paraplegia .


Cardiovascular/Pulmonary and Lymphatic I Interventions

A patient with a long history of systemic steroid use for asthma control has a contraindication for percus-
sion if there is evidence of:

Choices:
1. Decreased bone density.
2 . Intercostal muscle wasting.
3. BP > 140/9 0.
4. Barrel chest.

Teaching Points

Correct Answer: 1
The only one that is a contraindication to percussion would be decreased bone density, because a rib frac-
ture might be a possible result.

Incorrect Choices:
Although the other choices are sequelae to long-term systemic steroid use, they are not a contraindication
for percussion. An increased BP higher than that reported in the scenario might be a contraindication to
postural drainage. It is not a contraindication to percussion. Barrel chest is seen in patients with emphy-
sema, not asthma.

Type of Reasoning: Inference


One must determine, given an understanding of the nature of long-term systemic steroid use for asthma,
which condition would contraindicate percussion. It is beneficial to understand how percussion is per-
formed, because it helps one to reason why decreased bone density would contraindicate the therapy.
Inferential reasoning skill is used because one must infer the effects of long-term steroid use coupled with
the nature of percussion therapy when used with patients who have asthma.
Examination C 831

"
Musculoskeletal/Examination

The physical therapist is examining the patient's left ankle. The special test pictured below is primarily test-
ing the integrity of the:

Choices:
1. Calcaneofibular ligament.
2. Anterior talofibular ligament.
3. Tibiotalar ligament.
4. Anterior tibiofibular ligament.

Teaching Points

Correct Answer: 2
The special test being performed is the anterior drawer test of the ankle. This test is designed primarily
to test for injuries to the anterior talofibular ligament. This is the most frequently injured ligament in the
ankle. When the foot is in slight plantar-flexion position the anterior talofibular ligament is perpendicu-
lar to the long axis of the tibia. A positive anterior drawer test may be obtained with a tear of only the
anterior talofibular ligament, but anterior translation is greater if the calcaneofibular ligament is also torn.
Ideally, the knee should be placed in 90 degrees of flexion to alleviate tension on the Achilles tendon.

Incorrect Choices:
The primary test for the calcaneofibular ligament is talar tilt. The tibiotalar and anterior tibiofibular liga-
ment would not be stressed with the anterior drawer test.

Type of Reasoning: Analysis


One must understand the test described in the question in order to choose the correct solution. Through
analytical reasoning, the test taker must determine how each ligament is tested. If this question was an-
swered incorrectly, review information on provocative testing for ankle ligaments .


Musculoskeletal I Evaluation, Diagnosis

A patient with possible ligamentous injury of the knee presents with excessive tibial external rotation.
Which ligament is MOST LIKELY to be injured?

Choices:
1. Medial collateral.
2. Anterior cruciate.
3. Posterior cruciate.
4. Medial patello-femoral.
832 Examination C

Teaching Points

Correct Answer: 1
The medial collateral ligament prevents external rotation and provides stability to the knee. Excessive ER
would represent injury to either the MCL or the LCL.

Incorrect Choices:
The ACL and PCL prevent internal rotation, and the medial patello-femoralligament does not playa role
in the prevention of tibial rotation.

Type of Reasoning: Inference


One needs to be aware of the anatomical attachment of both the cruciate and collateral ligaments in order
to arrive at a correct conclusion. For this question, one must infer what is likely to be true of a case, which
is an inferential reasoning skill. In this case, the medial collateral ligament is most likely injured. Review
collateral and cruciate ligaments of the knee and stability if answered incorrectly.

Musculoskeletal I Interventions

A patient has suffered a meniscus injury and had to undergo meniscal repair surgery. The patient wants to
know when walking can commence after the surgery. The earliest the patient would be allowed to weight
bear after the surgery is:

Choices:
1. Immediately.
2. At the end of postsurgical week 1.
3. At the end of postsurgical week 2.
4. At the end of postsurgical week 3.

Teaching Points

Correct Answer: 4
After a meniscal repair, patients should be nonweight bearing for 3 to 6 weeks.

Incorrect Choices:
As the meniscus is still going through the healing process, it can be damaged if weight bearing occurs
before 3 weeks.

Type of Reasoning: Deductive


One needs to know the difference in weight-bearing restrictions between a partial meniscectomy, which
allows partial weight bearing as tolerated when full knee extension is obtained, and meniscus repair sur-
gery, which requires 3 to 6 weeks of nonweight bearing for proper healing. This factual recall of guidelines
is a deductive reasoning skill. For this situation, weight bearing can occur at the end of postsurgical week
3. Review postsurgical restrictions for meniscal repair if answered incorrectly.

Examination C 833

Musculoskeletal I Interventions

A therapist is treating a patient with a diagnosis of right shoulder rotator cuff tendinitis. The findings of a
work site ergonomic assessment indicate that the worker is required to perform repetitive reaching activities
above shoulder height. The most appropriate work site modification would be to:

Choices:
1. Allow the worker to take more frequent rests to avoid overuse.
2. Reposition the height of the shelf and items to below shoulder height.
3. Provide the worker with a taller, sit-stand chair.
4. Provide the worker with a standing desk for daily activities.

Teaching Points

Correct Answer: 2
Work stations should be designed to accommodate the persons who actually work on the job. Work sta-
tions should be easily adjustable and designed to be comfortable fo r the worker. In this case, lowering the
height of the shelf for frequent use is best.

Incorrect Choices:
Taking more frequent rests or providing a different ch air does not eliminate the essential problem of
repetitive overhead reach that is causing the shoulder tendinitis. Using a standing desk would eliminate
overhead reach, but is not as practical as lowering the shelf. In the workplace, individuals cannot be ex-
pected to stand all day long.

Type of Reasoning: Inference


One must infer the best solution to alleviate the patient's right shoulder rotator cuff tendinitis, given the
nature of the patient's work environment. In this situation, one should infer that the overhead repetitive
reaching is the cause for the diagnosis; therefore, reaching below shoulder height would help to alleviate
the symptoms. If this question was answered incorrectly, refer to information on ergonomic workstation
assessment.

C137

Musculoskeletal I Interventions

The patient has a fifth rib that is "stuck" in the position of maximal inspiration. Which technique is BEST
to improve the rib mobility and assist it in returning to its resting position?

Choices:
1. Maitland grade IV mobilization of the head of the rib at the costovertebral joint in the inferior direction.
2. Maitland grade II mobilization of the head of the rib at the costovertebral joint in the superior direction.
3. Maitland grade IV mobilization of the head of the rib at the costovertebral joint in the superior direction.
4. Maitland grade II mobilization of the head of the rib at the costovertebral joint in the inferior direction.
Teaching Points
834 Examination C

Correct Answer: 3
With inspiration, the lateral portion of the ribs moves up and the head moves down; to bring it back to a
neutral position, the head needs to glide superiorly, allowing the lateral part of the rib to lower with expi-
ration. Grade N mobilizations are used to improve joint mobility.

Incorrect Choices:
All other choices have either the incorrect technique to improve joint mobility or incorrect direction.

Type of Reasoning: Inference


One needs to be knowledgeable of the different utilization of mobilization grades and of the biomechan-
ics of the costovertebral joints during respiration in order to determine a correct conclusion. This neces-
sitates inferring a best course of action for a successful clinical outcome, which is an inferential reasoning
skill. For this situation, Maitland grade N mobilization of the head of the rib at the costovertebral joint in
the superior direction is best. Review Maitland's oscillatory techniques for the rib cage if answered incor-
rectly.


Neuromuscular I Examination

A patient with a fibular fracture complains of weakness in the RLE following cast removal. Examination re-
veals measurable loss of muscle bulk (2-inch girth difference between the right and left legs) . The therapist
next examines tone. The MOST likely finding will be:

Choices:
1. Hypotonia.
2. Hypertonia.
3. Dystonia.
4. Normal tone.

Teaching Points

Correct Answer: 1
Hypotonia is one of a number of signs and symptoms of LMN injury that also include neurogenic atrophy
and weakness (as in this case), along with decreased or absent reflexes and fasciculations.

Incorrect Choices:
Hypertonia is seen with UMN lesion (corticospinal lesions) . Dystonia is a CNS disorder characterized by
prolonged involuntary muscular contractions and increased muscle tone. Tone should not be expected to
be normal given the other findings .

Type of Reasoning: Inference


One must draw an appropriate conclusion of what is likely to be true in order to answer this question
correctly. Questions that require one to determine what is likely to be true of a situation necessitate infer-
ential reasoning. In this case, the most likely finding is hypotonia. Review hypotonia and typical features
after cast removal if answered incorrectly.

Exam ination C 835

Cardiovascular/Pulmo n ary a n d Ly mphati c I Ex a mi nati o n

A patient has a lO-year history of peripheral vascular disease (PVD) affecting the right lower extremity. Dur-
ing auscultation of the popliteal artery, the therapist would expect to find :

Choices:
l. A bruit.
2. A positive Homan's sign.
3. 4+ pulses.
4. Intense pain and cramping.

Teaching Points

Correct Answer: 1
A bruit is a swishing sound that occurs in the presence of narrowing of an artery. It is a characteristic find-
ing of PVD present on auscultation.

Incorrect Choices:
All other choices are not revealed on auscultation. Pulse palpation of the extremities will likely reveal a 0
(no pulse) or 1+ (diminished, barely perceptible), not 4+ (bounding, very strong). Homan's sign is pain
in the calf when the foot is passively dorsiflexed. It is no longer considered an accurate test to detect DVT.
Severe or intense pain with cramping is a likely response when the patient subjectively grades ambulatory
ischemic pain. It is not auscultation.

Type of Reasoning: Inference


This question requires one to infer what a therapist should find with a patient who has a history of PVD.
One must have a solid understanding of the nature of PVD and examination measures in order to choose
the correct solution. If this question was answered incorrectly, review PVD examination .

C140

Musculoskeletal I Examination

When performing the Thomas test, the patient's thigh does not touch the table, indicating limited hip ex-
tension. The amount of limited hip extension does not change when the ipsilateral knee is extended. What
is the range-limiting muscle?

Choices:
1. Tensor fascia lata.
2. Biceps femoris .
3. Iliopsoas.
4. Rectus femoris .

Teaching Points

Correct Answer: 3
The iliopsoas is a one joint hip flexor muscle and will limit hip extension regardless of the amount of
knee flexion.
836 Examination C

Incorrect Choices:
The tensor fascia lata and rectus femoris are two joint muscles that flex the hip and extend the knee. Pas-
sive knee extension would allow for further hip extension in both of these muscles. The biceps femoris, a
hamstring muscle, is a hip extensor and will not limit hip extension.

Type of Reasoning: Inference


One must infer the mostly likely muscle that is causing the limited hip extension, regardless of the
amount of knee flexion present. This necessitates one to determine what is most likely to be true of a situ-
ation, which is an inferential reasoning skill. For this situation, the iliopsoas muscle is limiting the range.
Review kinesiology of the hip and knee if answered incorrectly.

C141
Musculoskeletal I Evaluation, Diagnosis

A teenager presents to the clinic with vague left hip and groin pain that worsens with weight bearing. The
PT's examination reveals limited and painful hip internal rotation, antalgic gait, and a weak gluteus medius.
Based upon this clinical presentation, the MOST LIKELY diagnosis is:

Choices:
1. Slipped capital femoral epiphysis (SCFE).
2. Legg-Calve-Perthes disease.
3 . Gluteus medius muscle strain.
4. Oligoarticular juvenile rheumatoid arthritis (JRA).

Teaching Points

Correct Answer: 1
The SCFE age range is 10 to 16 years of age, and the male to female ratio is 3: 1. The incidence of left hip to
right hip is 2: I, with 30% bilateral. The best examination findings include pain that worsens with weight
bearing, limited and painful hip internal rotation, and weak hip abductors.

Incorrect Choices:
Legg-Calve Perthes disease is avascular necrosis of the femoral head with insidious onset between 3-12
years of age. The male to female ratio is 4:1. The clinical presentation is an antalgic gait, disuse atrophy of
the hip and thigh muscles, and painful limitation of abduction and internal rotation. A gluteus medius
muscle strain would present with more specific focal pain and pain with muscular contraction. The associ-
ated weakness in this case is secondary to an alteration of the length-tension relationship. Pauciarticular
or oligoarticular JRA onset occurs in 40% of children with juvenile rheumatoid arthritis. In some com-
bination, the knees, ankles, wrists, or elbows are the most frequently affected joints. The hips are usually
spared.

Type of Reasoning: Inductive


This question provides a group of symptoms, and the test taker must determine the most likely diagnosis
based on this information. Questions of this nature often require inductive reasoning skills. For this case,
the symptoms most likely indicate slipped capital femoral epiphysis. If answered incorrectly, review signs
and symptoms of SCFE.
Examination C 837

Nonsystem I Equ ipme nt, D evices , T h e rapeutic M o dalitie s

A patient with a transfemoral amputation and an above-knee prosthesis demonstrates knee instability
while standing. The patient's knee buckles easily when performing weight shifts. The therapist suspects the
cause of his problem is a:

Choices:
1. Prosthetic knee set too far posterior to the trochanter-knee-ankle (TKA) line.
2. Prosthetic knee set too far anterior to the TKA line.
3. Weak gluteus medius.
4. Tight extension aid.

Teaching Points

Correct Answer: 2
In order to increase stability of the knee, the prosthetic knee is normally aligned posterior to a line ex-
tending from the trochanter to the ankle (TKA line). A knee set anterior to the TKA line will buckle easily.

Incorrect Choices:
A prosthetic knee set too far posterior to the TKA would result in excessive knee stability and difficulty
flexing the knee. The gluteus medius contributes to stability during stance, primarily lateral stability. Weak
abductors can result in trunk lateral bend during stance. An extension aid assists knee extension during
the latter part of swing phase. A tight extension aid will result in terminal swing impact during late swing.

Type of Reasoning: Inductive


Clinical judgment and diagnostic reasoning are utilized to determine the likely cause for the patient's
knee instability while standing. Through inductive reasoning, the test taker should determine that the
prosthetic knee is set too far anterior to the TKA line. If this question was answered incorrectly, review
information on prosthetic adjustment/alignment for above-knee prostheses.

C143
Neurom usc u lar I Ex amin ati o n

An older adult at risk for falls has undergone a structured home-based exercise program that consisted of
standing balance training and strengthening exercises. The measure that can BEST show improvements is:

Choices:
1. 6-Minute Walk Test.
2. Berg Balance Test.
3. Timed Up &. Go Test.
4. Performance-Oriented Mobility Assessment (Tinetti) .

Teaching Points
Correct Answer: 2
The Berg Balance Test is a 14-item test of static and dynamic balance in sitting and standing. It also exam-
ines sit-to-stand and stand-to-sit transitions. It does not include items examining gait. Gait was not part of
this person's training program.
838 Examination C

Incorrect Choices:
The TUG and Tinetti examine gait along with along with balance. The 6-MWf is a measure of walking
endurance.

Type of Reasoning: Inductive


This question requires clinical judgment to determine the BEST measurement tool for improvement in
balance. This is an inductive reasoning skill, where clinical knowledge is applied to therapeutic situations
to determine a best course of action. For this situation, the Berg Balance Test is best. If answered incor-
rectly, review balance tests, especially the Berg Balance Test.

C144
Musculoskeletal I Evaluation, Diagnosis

A middle-aged patient complains of "throbbing pain" in the lumbar region with activities upon exertion,
such as walking up a flight of steps or playing tennis. The patient expresses no complaints of pain with
bending, twisting, sitting, standing, or walking any distance. Active movements of the lumbar spine are full
and pain free. Provocation testing is negative. Neurological signs are unremarkable. There is no significant
tenderness to palpation. The MOST LIKELY diagnosis for this patient would be:

Choices:
1. Quadratus lumborum muscle strain.
2. Lumbar disc herniation.
3. Sacroiliac joint sprain.
4. Aortic aneurysm.

Teaching Points

Correct Answer: 4
Throbbing low back pain that occurs only with activities that increase the heart rate is a red flag for aortic
aneurysm. Medical referral is indicated.

Incorrect Choices:
The fact that the patient reports no pain with activities such as bending, twisting, sitting, and walking
reduces the odds that the problem is mechanical. In addition, the fact that the symptoms could not be re-
produced during the physical therapy examination reduces the odds that it is a musculoskeletal condition.

Type of Reasoning: Analysis


This question provides a group of symptoms that the test taker must analyze to determine the most likely
diagnosis. Questions of this nature often require analytical reasoning skill. For this situation, the most
likely diagnosis is aortic aneurysm, given the symptoms presented. Review signs and symptoms of aortic
aneurysm if answered incorrectly.
Exam ination C 839

Neuromuscular I Interventions

A patient who is undergoing spinal cord rehabilitation is viewed as uncooperative by staff. The patient
refuses to complete the training activities outlined to promote independent functional mobility. A review
of history reveals that previously the patient was the director of a company with a staff of 20 . The MOST
appropriate strategy the therapist can adopt is to:

Choices:
1. Carefully structure the activities and slow down the pace of training.
2. Have the patient work with a supervisor who is a person in authority.
3. Refer the patient to a support group before resuming rehabilitation.
4. Involve the patient in goal setting and structuring the training session.

Teaching Points

Correct Answer: 4
An andragogical approach is best. The patient is an adult learner who should be allowed to share in the
responsibility for planning the learning experience and goal setting. The therapist should help clarify the
problem, structure the learning environment, and provide necessary resources.

Incorrect Choices:
Passing the buck or transferring responsibility to a supervisor is not appropriate. Requiring participation
in a support group before continuing rehabilitation or slowing down the pace of training would nega-
tively impact rehabilitation and result in increased length of stay.

Type of Reasoning: Evaluation


One must evaluate the merits of each statement in order to make a best course determination for this
patient. This question requires a judgment call and a value judgment, which necessitates evaluative rea-
soning skill. For this patient, it is MOST appropriate to involve the patient in goal setting and h ave him
participate in structuring the training session .


Nonsystem I Safety, Professional Responsibilities, Research

A group of 10 patients is recruited into a study investigating the effects of relaxation training on blood pres-
sure (BP). One group of patients is scheduled to participate in a supervised cardiac rehabilitation program
that includes relaxation training three times a week for 12 weeks. The other group of patients is instructed
to perform activities as usual. At the conclusion of the study, there was no significant difference between the
groups; BP decreased significantly in both groups. The investigator can reasonably conclude:

Choices:
1. The activities of the nonrehabilitation group were not properly monitored and may account for these
results.
2. Cardiac rehabilitation is not effective in reducing BP.
3. Both groups had BPs initially so high that reductions should have been expected.
4. The rehabilitation group was not properly monitored.
840 Examination C

Teaching Points

Correct Answer: 1
To ensure adequate control, the researcher should attempt to remove the influence of any variable other
than the independent variable in order to evaluate its effect on the dependent variable. In this study, the
investigator did not adequately investigate the usual activities of the control group . The small number of
subjects may also have contributed to lack of significance.

Incorrect Choices:
Accepting any statements concerning initial BP or changes in BP is not warranted because data on specific
BPs were not provided. The rehabilitation group was closely supervised.

Type of Reasoning: Inference


Whereas research-related questions are typically factual in nature, requiring deductive reasoning skill,
this question specifically asks for one to draw conclusions based on evidence and infer the reason for the
research outcomes, which is an inferential reasoning skill.

"
Neuromuscular I Examination

A therapist is examining a patient newly diagnosed with multiple sclerosis. The patient experiences an in-
tense shock-like pain throughout the body when the neck is passively flexed. The therapist recognizes these
findings as:

Choices:
1. Paresthesias.
2. Dysesthesias.
3. Lhermitte's sign.
4. Uthoff's sign.

Teaching Points

Correct Answer: 3
Lhermitte's sign is a sign of posterior column damage in the spinal cord; flexion of the neck produces an
electric shock-like painful sensation running down the spine and into the LEs. It can be seen in patients
with MS.

Incorrect Choices:
Sensory disturbances are common in MS, although not described in this case. Paresthesias are abnormal
sensations such as numbness, prickling, or tingling. Dyesthesias are abnormal and unpleasant sensations
such as burning or pins and needles. Uthoff's sign is an adverse reaction to heat, commonly seen in pa-
tients with MS.

Type of Reasoning: Analysis


One must analyze the presenting symptoms and determine the most likely cause for those symptoms.
This necessitates analytical reasoning skill, where a cluster of symptoms must be weighed to draw a con-
clusion. For this case, the symptoms indicate the patient is demonstrating a Lhermitte's sign. If answered
incorrectly, review common features of multiple sclerosis, especially Lhermitte's sign.

Examination C 841

Cardiovascular/Pulmonary and Lymphatic I Examination

Which cluster of examination findings would indicate that a patient is in decompensated heart failure?

Choices:
1. S2 heart sound, wheezes on lung auscultation, increased jugular vein distension.
2. S2 heart sound, crackles on lung auscultation, decreased jugular vein distension.
3. S4 heart sound, crackles on lung auscultation, increased jugular vein distension.
4. S4 heart sound, wheezes on lung auscultation, decreased jugular vein distension.

Teaching Points
Correct Answer: 3
S4 indicates a pathology in the ventricle, as is present in a patient with heart failure. Crackles on auscul-
tation are often heard in a patient with heart failure because they have compressive atelectasis due to
pulmonary edema. When cued to take a deep breath, they can create enough pressure to open the alveoli,
which causes the crackling sound. It is possible that a patient in heart failure will also have wheezes if
there is significant pulmonary edema causing the alveoli to fill with fluid . With decompensated heart fail-
ure, there will be a back-up of fluid throughout the cardiac and pulmonary system, which can extend into
the peripheral circulatory system. This will cause increased distension of the jugular veins.

Incorrect Choices:
An S2 heart sound is a normal sound, indicating the closure of the aortic and pulmonic valves. Decreased
jugular vein distension occurs only when a patient has decreased volume or is dehydrated.

Type of Reasoning: Analysis


For this question, the test taker must utilize clinical judgment to determine the findings that best indi-
cate decompensated heart failure. This is an inductive reasoning skill. For this situation, S4 heart sound,
crackles on lung auscultation, and increased jugular vein distension are all indicative of decompensated
heart failure. Review heart failure guidelines and signs and symptoms of decompensated heart failure if
answered incorrectly.

C149

Neuromuscular I Evaluation, Diagnosis

A therapist is examining a 24-month-old child and observes that the child can sit independently, creep in
quadruped, pull-to-stand, and cruise sideways, but not walk without support. The therapist concludes that
this child is exhibiting:

Choices:
1. Normal cephalocaudal motor development.
2. Delay in achieving developmental milestones.
3. Normal gross motor development.
4. Slow maturation that is within normal limits.
842 Examination C

Teaching Points

Correct Answer: 2
The 12- to IS-month-old child should be ambulating. At 24 months, lack of ambulation is indicative of
developmental delay. Other developmental milestones include sitting at 6 months and creeping, pull-to-
stand, and cruising at 8 to 9 months.

Incorrect Choices:
These findings are indicative of abnormal motor milestones (normal cephalocaudal motor development,
normal gross motor development, slow maturation within normal limits) . In fact, 24 months represents
considerable delay in ambulation.

Type of Reasoning: Analysis


One must recall the normal developmental milestones of children in order to determine through analyti-
cal reasoning that a 24-month-old child who is not ambulating is delayed. Analytical reasoning is utilized
whenever one must determine the precise meaning of information and make interpretations from that
information. If this question was answered incorrectly, review the developmental milestones of children
through the first 2 years .


Musculoskeletal I Interventions

Therapist hand/finger placements for posteroanterior mobilization techniques to improve down-gliding/


closure of the T? -8 facet joints should be located at the:

Choices:
1. Transverse processes ofT8.
2 . Spinous process ofT8.
3. Transverse processes ofT?
4. Spinous process ofT6 .

Teaching Points

Correct Answer: 1
The axis of motion for the mid-thoracic vertebrae is above the spinous processes and below the transverse
processes. Therefore, if down-gliding/ closure ofT?-8 vertebral segment is required, the therapist's hand
placement should be at the transverse process ofT8 or the spinous process ofT?

Incorrect Choices:
PA pressures on the spinous process in this region of the thoracic spine will cause the spinous process to
glide into and compress the spinous process of the segment below, so no arthrokinematic glide will occur.
A PA central glide to the transverse process ofT7 will increase extension between T6 and T7 .

Type of Reasoning: Analysis


One must determine the correct mobilization techniques to improve down-gliding/closure of the T7-8
facet joints in order to arrive at the correct conclusion. This requires knowledge of appropriate mobilization
techniques of the thoracic spine, including how to mobilize facet joints. Through analytical reasoning,
one should conclude that the correct hand/finger placements should be located at the transverse processes
ofT8.
Examination C 843

Neuromuscular I Evaluation , Diagnosis

A patient complains of waking up several times at night from severe "pins and needles" in the right hand.
On awakening, the hand feels numb for half an hour and fine hand movements are impaired. The thera-
pist's examination reveals sensory loss and paresthesias in the thumb, index, middle, and lateral half of the
ring finger and reduced grip and pinch strength. Some thenar atrophy is present. Based on these examina-
tion findings, the MOST appropriate diagnosis is:

Choices:
1. Thoracic outlet syndrome (TOS) .
2. Ulnar nerve entrapment.
3. Carpal tunnel syndrome.
4. Pronator teres syndrome.

Teaching Points

Correct Answer: 3
Carpal tunnel syndrome is the result of compression of the median nerve under the flexor retinaculum at
the wrist. It is characterized by thenar atrophy and sensory loss. Symptoms are worse at night and include
burning, tingling, pins and needles, and numbness into the median nerve sensory distribution (palmar
and dorsal thumb, index, middle, and lateral half ofthe ring finger) .

Incorrect Choices:
TOS is a neurovascular compression syndrome caused by compression of nerves and/or vessels in the
neck. It is characterized by brachial neuritis with or without vascular and vasomotor disturbances in the
upper extremity. Electrophysiological studies indicate proximal and not distal compression. Ulnar nerve
entrapment typically presents with nocturnal numbness and sensory loss in the little finger and half of
the ring finger. The palm is typically not affected. Pronator teres syndrome arises from compression of
the median nerve at the elbow with radiation into the radial aspect of the hand. Pronation is possible but
weak (not evident in this case) . Numbness extends into the median nerve distribution. With passage of
time, atrophy of the thenar muscles will occur.

Type of Reasoning: Analysis


One must analyze the symptoms presented in order to make a determination of the likely diagnosis.
This requires analytical reasoning skills as one must determine the meaning of the symptoms in order to
determine a diagnosis for the patient. In this case, the symptoms indicate carpal tunnel syndrome, which
should be reviewed if this question was answered incorrectly.


Musculoskeletal I Examination

A patient is referred for physical therapy with a diagnosis of degenerative joint disease (DJD) affecting C2
and C3 . The patient complains of pain and stiffness in the cervical region and transient dizziness with some
cervical motions. The BEST initial examination procedure is:

Choices:
l. Lhermitte's test.
2. Vertebral artery test.
3. Oppenheim's test.
4. Adson's maneuver.
844 Exam in ation C

Teaching Points

Correct Answer: 2
The vertebral artery test checks the integrity of the blood flow through the artery in the cervical region.
Because the patient is experiencing symptoms of circulatory disturbance and a unilateral pull could com-
press the left cervical structures, the vertebral artery test is an appropriate screening test. The test consists
of passively placing the patient's head in extension and side flexion. Then the head and neck are slowly
rotated to the laterally flexed side and held for 30 seconds. Some of the positive signs may be syncope,
lightheadedness, nystagmus, or visual disturbances.

Incorrect Choices:
Lhermitte's sign is pain down the spine and into the upper or lower limbs with passive flexion of the neck.
It is used to identify dysfunction of the spinal cord associated with upper motor neuron lesions, and is
typically positive in MS. Adson's maneuver is a test for TOS. Oppenheim's test involves running a finger-
nail along the crest of the tibia; a positive test is the same as a positive Babinski.

Type of Reasoning: Inference


One must first determine the meaning of each of the four possible tests and then infer which one is the
BEST initial test, given the patient's diagnosis and symptoms. If this question was answered incorrectly,
review the vertebral artery test.


Neuro mu scu la r I Evaluation , Diagnosis

A young, otherwise healthy, adult is recovering from a complete spinal cord injury (ASIA A) at the level of
L4. Functional expectations for this patient include:

Choices:
l. Ambulation using bilateral AFOs and canes.
2. Ambulation using bilateral KAFOs and a reciprocating walker.
3. Ambulation using bilateral KAFOs, crutches, and a swing-through gait.
4. Ambulation using reciprocating gait orthoses and a reciprocating walker.

Teaching Points

Correct Answer: 1
A spinal cord lesion at the level of L4 is considered an LMN injury (cauda equina injury) . Intact move-
ments include hip flexion, hip adduction, and knee extension. The quadriceps becomes FULLY innervated
at L4. This patient can be expected to be a functional ambulator using bilateral AFOs and crutches or
canes.

Incorrect Choices:
All other choices include orthotic devices that offer control of joints that is not needed (orthotic bracing
of knees and hips) . Also crutches or walkers should not be needed.

Type of Reasoning: Analysis


One must recall past knowledge of neuroscience related to spinal cord injuries in order to determine the
functional expectations for the patient with a complete L4 injury. In order to arrive at the correct conclu-
sion, one must analyze the four possible choices and determine which choice is most aligned with one's
knowledge of injury at this level. If this question was answered incorrectly, review functional expectations
for patients with lumbar spinal cord injuries.
Examinat ion C 845

Neuromuscular I Examination

To examine a patient with a suspected deficit in graphesthesia, the therapist should ask the patient to shut
his/ her eyes and identify:

Choices:
1. Differently weighted, identically shaped cylinders placed in the hand.
2. The vibrations of a tuning fork when placed on a bony prominence.
3. A series of letters traced on the hand.
4. Different objects placed in the hand and manipulated.

Teaching Points

Correct Answer: 3
Graphesthesia is the ability to recognize numbers, letters, or symbols traced on the skin.

Incorrect Choices:
Barognosis is the ability to recognize different weights placed in the hand using identically shaped
objects. Pallesthesia is the ability to recognize vibratory stimuli (i.e., a vibrating tuning fork placed on a
bony prominence). Stereognosis is the ability to recognize different objects placed in the hand and ma-
nipulated. During testing, vision is occluded.

Type of Reasoning: Deductive


This question requires factual recall of the definition of graphesthesia, which is a deductive reasoning
skill. Using past knowledge of neuroscience is beneficial to arriving at the correct solution. If this question
was answered incorrectly, review sensory testing procedures.

C155
Neuromuscula r I Interventions

A patient recovering from stroke is ambulatory without an assistive device and demonstrates a consistent
problem with an elevated and retracted pelvis on the affected side. The BEST therapeutic exercise strategy is
to manually apply:

Choices:
l. Anterior-directed pressure during swing.
2. Light resistance to forward pelvic rotation during swing.
3. Downward compression during stance.
4. Light resistance to posterior pelvic elevation during swing.

Teaching Points

Correct Answer: 2
An elevated and retracted pelvis is a common problem during gait for many patients recovering from a
stroke. Providing light resistance to forward pelvic rotation actively engages those muscles and reciprocally
inhibits the spastic retractors.
846 Examination C

Incorrect Choices:
Providing anterior-directed pressure during swing or resistance to pelvic elevation may serve only to
increase abnormal tone in those muscles. The problem is a swing phase deficit; downward compression
during stance is inappropriate for this problem.

Type of Reasoning: Inductive


One must utilize clinical judgment in order to determine which therapeutic exercise strategy would BEST
address the patient's problem. Using inductive reasoning skills, the test taker must analyze the four possi-
ble choices and then rely on knowledge of mobility challenges in a stroke in order to determine the BEST
course of action for this patient.

"
Neuromuscular I Examination

A therapist suspects lower brainstem involvement in a patient with amyotrophic lateral sclerosis (ALS).
Examination findings reveal motor impairments of the tongue with ipsilateral wasting and deviation on
protrusion. These findings confirm involvement of cranial nerve:

Choices:
1. IX.
2 . XII .
3. XI.
4. X.

Teaching Points

Correct Answer: 2
The hypoglossal (CN XII) controls the movements of the tongue. Ipsilateral wasting and the deviation to
the ipsilateral side on protrusion are indicative of damage.

Incorrect Choices:
Involvement of the glossopharyngeal (CN IX) results in slight dysphagia, loss of taste in the posterior
third of the tongue, and loss of gag reflex. Involvement of the spinal accessory nerve (CN XI) results in
minor problems in deglutition and phonation along with weakness in ipsilateral shoulder shrugging.
Involvement of the vagus (CN X) results in dysphagia, hoarseness, and paralysis of the soft palate.

Type of Reasoning: Analysis


This question requires one to recall the functions of the 12 cranial nerves and then determine which nerve
is most likely impaired in this situation. Questions that involve determining an impairment based on a
group of symptoms require analytical reasoning skill. If this question was answered incorrectly, review
examination of the 12 cranial nerves.
Exa min at ion C 847

Musculoskeletal I Evaluation , Diagnosis

A baseball pitcher reports insidious onset of symptoms characteristic of impingement, including catching
and popping in the throwing arm. Examination reveals th at glenohumeral passive internal rotation is pain-
ful and limited to 30 degrees. External rotation is less symptomatic and has 130 degrees of passive range.
The PT should first:

Choices:
l. Begin elastic resistance exercises for impingement.
2. Mobilize the glenohumeral joint to increase internal rotation ROM.
3. Recommend an anteroposterior radiograph.
4. Recommend an MRI.

Teaching Points

Correct Answer: 4
The therapist should suspect a labral tear. This is a common finding among pitchers and athletes who do
a lot of throwing (especially those who present with abnormal ROM findings and instability symptoms
of popping and catching). Although this athlete may present with impingement, an MRI is warranted to
fully diagnose the condition and to develop an appropriate treatment plan.

Incorrect Ch oices:
Beginning treatment without definitive testing is contraindicated. Plain radiographs are inappropriate for
suspected soft tissue lesions.

Type of Reasoning: Inductive


This question requires clinical judgment to determine a best course of action for a patient with painful
passive internal rotation with limitations in range. Questions that require one to weigh a group of symp-
toms and determine a best course of action often necessitate inductive reasoning skills. If this question
was answered incorrectly, review signs and symptoms of labral tear and diagnostic testing.

System Interactions I Evaluation, Diagnosis

A woman is hospitalized in the intensive care unit with multiple closed and open fractures after a motor
vehicle accident. A review of her medical record reveals the following laboratory values: hematocrit 28%,
hemoglobin 10 g/100 mL, and serum white blood cell (WBe) count 12,000/mm3 . The MOST accurate con-
clusion the PT can reach in this case is that:

Choices:
1. Hematocrit and hemoglobin values are abnormal; WBC is normal.
2. Only hematocrit values are abnormal.
3. Only serum WBC is abnormal.
4. All values are abnormal.
848 Examination C

Teaching Points
Correct Answer: 4
The hematocrit value is abnormally low; women average 42% with a normal range from 37% to 47%. The
hemoglobin value is also abnormally low; women average 12 to 16 g/100 mL of blood. The low values are
most likely due to blood loss. The serum WBC (leukocytes) is abnormally high; normal values are 5,000
to 1O,000/ mm 3 • The elevated count (> 10,000) indicates acute infection.

Incorrect Choices:
The other choices present findings in which only some, not all, of the values are abnormal.

Type of Reasoning: Deductive


One must recall the normal laboratory values for female adults in order to arrive at the correct conclusion.
This necessitates deductive reasoning, in which recall of factual knowledge and protocols is utilized to
determine the correct solution. In this situation, all of the laboratory values are abnormal. If this question
was answered incorrectly, review the parameters for these adult laboratory values.

C159
Integumentary I Evaluation, Diagnosis

A patient with a transfemoral amputation is unable to wear a total contact prosthesis for the past 4 days.
Examination of the residual limb reveals erythema and edema extending over most of the lower anterior
limb. The patient tells the therapist that the limb is very itchy and painful. The MOST LIKELY cause of these
symptoms is:

Choices:
1. Cellulitis.
2. Dermatitis.
3. Impetigo.
4. Herpes zoster.

Teaching Points

Correct Answer: 2
This patient is exhibiting symptoms of contact dermatitis. Primary treatment is removal of the offending
agent (in this case, the total contact prosthesis) and treatment of the involved skin with lubricants, topical
anesthetics, and/or steroids. The patient may require a thin sock ifthe problem does not resolve.

Incorrect Choices:
Cellulitis is a suppurative inflammation of the dermis and subcutaneous tissues frequently accompanied
by infection. Impetigo is a staphylococcal infection with small macules (unraised spots) or vesicles (small
blisters) . Herpes zoster is a viral infection with red papules along the course of a nerve or dermatome.

Type of Reasoning: Analysis


This question requires the test taker to determine the MOST LIKELY cause for the patient's symptoms,
which is an analytical reasoning skill. In order to arrive at the correct conclusion, one must analyze the
four possible choices and determine which choice is most aligned with one's knowledge of skin irrita-
tions, inflammations, and infections. If this question was answered incorrectly, review symptoms and
treatment approaches for dermatitis.
Examination C 849

Musculoskeletal I Examination

A PT is treating a patient who lacks wrist extension. The cause of the impairment is a problem at the ra-
diocarpal joint with a lack of arthrokinematic motion necessary for proper wrist extension. The proximal
aspect of the scaphoid/lunate should glide:

Choices:
1. Radially.
2. Ulnarly.
3. In a dorsal direction relative to the radius.
4 . In a palmar direction relative to the radius.

Teaching Points

Correct Answer: 4
The correct description of the arthrokinematic motion occurring at the radiocarpal joint lacking in this
patient is the palmar glide. Due to the anatomical structure of the bones forming this joint, the convex
scaphoid and lunate articulate with the concave radius, resulting in opposite roll and glide during exten-
sion.

Incorrect Choices:
Other choices describe movements occurring during ulnar and radial wrist deviation or of the glide occur-
ring during flexion .

Type of Reasoning: Deductive


One needs to understand the convex-concave rules of joint movement to arrive at a correct conclusion.
This requires recall of the anatomical structures of the wrist and proper rolls and glides, which is factual
information and therefore a deductive reasoning skill. For this case, the therapist should glide the lunate/
scaphoid palmarly relative to the radius. Review concave-convex rules of the wrist and mobilization tech-
niques if answered incorrectly.


Musculoskeletal I Examination

Following a hip fracture that is now healed, a patient presents with weak hip flexors (2/5 ). All other mus-
cles are within functional limits. During gait, the therapist expects that the patient may walk with:

Choices:
1. Forward trunk lean.
2. A circumducted gait.
3. Excessive hip flexion.
4. Backward trunk lean.
850 Examination C

Teaching Points

Correct Answer: 2
Circumduction is a compensation for weak hip flexors or an inability to shorten the leg (weak knee flexors
and ankle dorsiflexors). Hip hiking can also compensate for an abnormally long leg (lack of knee flexion
and dorsiflexion) .

Incorrect Choices:
Excessive hip flexion is a compensation for footdrop . Forward trunk lean and backward trunk lean are
stance phase deviations that compensate for quadriceps weakness and gluteus maximus weakness, respec-
tively.

Type of Reasoning: Inference


This question requires the test taker to draw conclusions based on evidence presented, which is an in-
ferential reasoning skill. Questions of this nature often provide a diagnosis, and the test taker must infer
the likely symptoms. In this case, the test taker must determine that a patient with weak hip flexors will
display a circumducted gait pattern during ambulation. If this question was answered incorrectly, review
causes for a circumducted gait pattern.

Musculoskeletal I Interventions

An elderly patient with a left transfemoral amputation complains that when sitting, the left foot feels
cramped and twisted. The therapist's BEST choice of intervention is:

Choices:
1. Hot packs and continuous US to the residual limb.
2. Iontophoresis to the distal residual limb using hyaluronidase.
3. Appropriate bed positioning with the residual limb in extension.
4. Icing and massage to the residual limb.

Teaching Points

Correct Answer: 4
This patient is experiencing phantom pain, a common occurrence seen in as many as 70% of patients with
lower limb amputations. Treatment interventions can include icing, pulsed US, TENS, or massage. Medical
interventions include injections and surgical procedures (rhizotomy, neurectomy).

Incorrect Choices:
Prolonged inactivity and bed rest are contraindicated. Hyaluronidase is indicated for edema reduction.
Continuous US is used to achieve thermal effects; heating is not indicated in this case.

Type of Reasoning: Inductive


This question asks the test taker to first consider what the cause for the symptoms is and then determine
the BEST choice for intervention with this patient. This is an inductive reasoning skill, in which diagnostic
reasoning is used to determine best approaches to clinical situations. For this patient, the symptoms clear-
ly indicate phantom pain, and icing and massage to the residual limb are BEST to treat the symptoms.

Exa mination C 851

Nonsystem I Safety, Professional Responsibilities, Research

A researcher uses a group of volunteers (healthy college students) to study the effects of therapy ball exer-
cises on ankle ROM and balance scores. Twenty volunteers participated in the 20-minute ball exercise class
three times a week for 6 weeks. Measurements were taken at the beginning and end of the sessions. Signifi-
cant differences were found in both sets of scores and reported at the local physical therapy meeting. Based
on this research design, the therapist concludes:

Choices:
1. Therapy ball exercises are an effective intervention to improve ankle stability after chronic ankle sprain.
2. The validity of the study was threatened with the introduction of sampling bias.
3. The reliability of the study was threatened with the introduction of systematic error of measurement.
4. The Hawthorne effect may have influenced the outcomes of the study.

Teaching Points

Correct Answer: 2
The investigator used a sample of convenience and therefore introduced systematic sampling error (a
threat to validity). Random selection of subjects would improve the validity of this study.

Incorrect Choices:
Generalization to a group of patients with chronic ankle sprain cannot be made. There was no system-
atic error in measurement in this example. The Hawthorne effect refers to the influence that the subject's
knowledge of participation in the experiment had on the results of the study.

Type of Reasoning: Inference


The test taker must determine what the most likely conclusion should be to this research study, given the
protocol implemented. Questions that require one to draw conclusions based on evidence necessitate in-
ferential reasoning skills to come to the correct conclusion. In this study, the validity has been threatened
by convenience sampling. If this question was answered incorrectly, review guidelines to improve validity
in research .

C164

Cardiovascular/Pulmonary and Lymphatic I Interventions

A patient has a very large right-sided bacterial pneumonia. Oxygen level is dangerously low. The body posi-
tion that would MOST likely improve the patient's arterial oxygen pressure (Pa0 2 ) is:

Choices:
1. Right side-lying with the head of the bed in the flat position.
2. Prone-lying with the head of the bed in the Trendelenburg position.
3. Supine-lying with the head of the bed in the Trendelenburg position.
4. Left side-lying with the head of the bed in the flat position.
852 Examination C

Teaching Points

Correct Answer: 4
In order to match perfusion and ventilation, the therapist needs to place the unaffected side in a gravity-
dependent position, or that of left side-lying.

Incorrect Choices:
Although prone and supine with the head of the bed in the flat position might be helpful, these positions
would not be as likely to show as much improvement as left side-lying. The use of the Trendelenburg
position (head lower than legs) is inappropriate.

Type of Reasoning: Inference


The test taker must infer the best body position in order to improve the patient's Pa0 2 levei. Knowledge of
pulmonary rehabilitation guidelines is important in order to arrive at the correct conclusion. If this ques-
tion was answered incorrectly, refer to pulmonary rehabilitation guidelines for patients with low Pa0 2,
especially body positions in bed.

C165

"
Musculoskeletal I Evaluation, Diagnosis

An examination of a patient reveals drooping of the shoulder, rotary winging of the scapula, an inability to
shrug the shoulder, and complaints of aching in the shoulder. Based on these findings, the cause of these
symptoms would MOST likely be due to:

Choices:
1. Muscle imbalance.
2 . A lesion of the long thoracic nerve.
3. A lesion of the spinal accessory nerve.
4. Strain of the serratus anterior.

Teaching Points

Correct Answer: 3
Rotary winging occurs when the inferior angle of one scapula is rotated farther from the spine than the
inferior angle of the other scapula. The shoulder drooping and inability to shrug the shoulder are second-
ary to a lesion of the spinal accessory nerve (CN XI), which innervates the trapezius muscle.

Incorrect Choices:
Although this type of winging could be found with all of these answers, the findings of shoulder drooping
and inability to shrug the shoulder clearly point to a lesion of eN XI.

Type of Reasoning: Analysis


This question requires the test taker to determine the diagnosis based on symptoms presented, which is an
analytical reasoning skill. Knowledge of neuroscience and nerve innervations of the cervical spine is ben-
eficial to arriving at the correct conclusion. If this question was answered incorrectly, refer to innervations
of the spinal accessory nerve and symptoms of lesions associated with this nerve.
Examination C 853

Nonsystem I Equipment, Devices, Therapeutic Modalities

A patient with spastic hemiplegia is referred to the therapist for ambulation training. The patient is hav-
ing difficulty with standing up from a seated position as the result of co-contraction of the quadriceps and
hamstrings during the knee and hip extension phase. The therapist wishes to use biofeedback beginning
with simple knee extension exercise in the seated position. The plan is to progress to sit-to-stand training.
The initial biofeedback protocol should consist of:

Choices:
1. High-detection sensitivity with recording electrodes placed closely together.
2 . Low-detection sensitivity with recording electrodes placed closely together.
3. Low-detection sensitivity with recording electrodes placed far apart.
4. High-detection sensitivity with recording electrodes placed far apart.

Teaching Points
Correct Answer: 2
By initially placing the electrodes close together, the therapist decreases the likelihood of detecting unde-
sired motor units from adjacent active muscles (crosstalk). By setting the biofeedback sensitivity (gain)
low, the therapist would decrease the amplitude of the signals generated by the hypertonic muscles and
keep the EMG output from exceeding a visual and/or auditory range (scale).

Incorrect Choices:
The other choices fail to use optimal placing of electrodes (electrodes placed far apart) or sensitivity (high-
detection) to optimize outcomes. The wider the spacing of electrodes, the more volume of the muscle is
monitored. Thus, when targeting a specific muscle, a narrow spacing should be used. When the focus is
not on a specific muscle but rather to encourage a general motion such as shoulder elevation, then a wider
spacing of electrodes can be used. In addition, when working with weakness of a muscle in which there is
a decreased ability to recruit motor units or there is a decrease in the size and number of motor units, then
a wider spacing and a high sensitivity would be used in order to create an adequate visible signal.

Type of Reasoning: Inductive


One must utilize clinical judgment and diagnostic reasoning in order to determine the most appropri-
ate initial biofeedback protocol. This requires knowledge of biofeedback guidelines and benefits of using
high- versus low-detection sensitivity and electrodes placed either closely together or far apart. If this
question was answered incorrectly, review biofeedback guidelines of the lower extremity.

C167
Neuromuscular I Examination

Examination of a patient recovering from stroke reveals a loss of pain and temperature sensation on the left
side of the face along with loss of pain and temperature sensation on the right side of the body. All other
sensations are normal. The therapist suspects a lesion in the:

Choices:
1. Midbrain.
2. Right cerebral cortex or internal capsule.
3. Left posterolateral medulla.
4. Left cerebral cortex or internal capsule.
854 Examination C

Teaching Points

Correct Answer: 3
A lesion in the posterolateral medulla cause mixed sensory loss (described in this case). Pain and tempera-
ture are affected, whereas discriminative touch and proprioception are not (the medial lemniscus is not
involved).

Incorrect Choices:
Sensory loss will be completely contralateral (not mixed) only after the discriminative sensory tracts
(fasciculus gracilis and fasciculus cuneatus) cross in the upper medulla. Patients with lesions above the
medulla (midbrain, cortex, or internal capsule) will present with contralateral sensory loss.

Type of Reasoning: Analysis


This question requires the test taker to determine the location of a lesion based on symptoms presented,
which is an analytical reasoning skill. Knowledge of neuroscience and the discriminative sensory tracts is
beneficial to arriving at the correct conclusion. If this question was answered incorrectly, refer to informa-
tion regarding the discriminative sensory tracts and lesions.

C168
Neuromuscular I Evaluation, Diagnosis

A patient is taking the drug baclofen to control spasticity after spinal cord injuries. This medication can be
expected to decrease muscle tone and pain. Adverse reactions of concern to the PT can include:

Choices:
1. Hypertension and palpitations.
2. Drowsiness and muscle weakness.
3. Headache with visual auras.
4. Urinary retention and discomfort.

Teaching Points

Correct Answer: 2
Baclofen, used in the management of spasticity, can produce CNS depression (drowsiness, fatigue, weak-
ness, confusion, vertigo, dizziness, and insomnia), occurring in less than 10% of patients. Additional
adverse effects can include hypotension and palpitations and urinary frequency. Vomiting, seizures, and
coma are signs of overdosage.

Incorrect Choices:
Adverse effects include hypotension not hypertension. Headaches with visual auras are characteristic of
migraines. Urinary frequency not retention is also an adverse effect.

Type of Reasoning: Deductive


One must recall the possible adverse effects of taking baclofen in order to arrive at the correct conclusion.
This requires knowledge of guidelines and factual information, which is a deductive reasoning skill. If this
question was answered incorrectly, refer to therapeutic uses of baclofen, including adverse effects.
Exa min atio n C 855

Muscu loskeletal/Examination

A teen-aged female distance runner presents a history of stress fractures and general leg pain. Her parents
think it may be due to overtraining. Based on this subjective information, what should the physical thera-
pist question the patient about next?

Choices:
1. Menses and eating habits.
2. Recent growth spurts.
3. Participation in other sports.
4. Type of running shoe.

Teaching Points

Correct Answer: 1
The female athlete triad is a syndrome common in females 14-20 years of age that consists of three relat-
ed conditions: disordered eating habits, irregular or absent menstrual periods, and osteopenia (thinning
of the bone density). It is imperative that the therapist inquire about menses and eating habits as they are
two of the three hallmark signs of the female athlete triad. It would also be helpful for the physical thera-
pist to inquire more about muscle pain, medications, and details of training regimen.

Incorrect Choices:
Recent growth spurts, participation in other sports, and type of shoes will not rule in/out the possibility
of the runner having this serious condition.

Type of Reasoning: Evaluation


In this question, the therapist must determine the BEST course of questioning to determine the patient's
likelihood of female athlete triad. This requires evaluative reasoning skill. If answered incorrectly, review
symptoms and examination of female athlete triad .


Cardiovascular/Pulmonary and Lymphatic I Interventions

A patient is exercising in a phase 3 outpatient cardiac rehabilitation program that utilizes circuit training.
One of the stations utilizes weights. The patient lifts a 5-lb weight, holds it for 20 seconds, and then low-
ers it slowly. The therapist corrects the activity and tells the patient to reduce the length of the static hold .
Resistance exercise with static holding can be expected to produce:

Choices:
1. Abnormal oxygen uptake.
2. Lower heart rate (HR) and arterial blood pressure (BP).
3. Higher HR and arterial BP.
4. Reduced normal venous return to the heart and elevated BP.
856 Examination C

Teaching Points

Correct Answer: 3
Dynamic exercise facilitates circulation, while isometric (static) exercise hinders blood flow, producing
higher HRs and arterial BPs. Rise in BP is related to degree of intensity (effort).

Incorrect Choices:
Resistance exercise does little to increase oxygen uptake. HR and BP are higher (not lower) . The Valsalva
maneuver (forced expiration against a closed glottis) that accompanies breath holding produces increased
intrathoracic pressure, which in tum hinders normal venous return to the heart. Breath holding is more
likely with isometric exercise, but is not always present.

Type of Reasoning: Inference


One must infer the likely outcomes of static exercise on BP and HR in order to arrive at the correct conclu-
sion. This requires one to draw conclusions based on the information presented, which is an inferential
reasoning skill. In this case, one should expect an elevated HR and arterial BP as a result of isometric exer-
cise, which should be reviewed if this question was answered incorrectly.


Musculoskeletal I Evaluation, Diagnosis

A patient complains of pain in the right lower aspect of the lateral rib cage. The chief complaint is intense
pain, which also occurs at night. The pain lasts for approximately 15 minutes and then subsides. There was
no mechanism of injury and no pain with activity during the day. Active and passive motion of the thoracic
spine and costal cage is normal and pain free. Mild tenderness is present inferior to the right lateral ribcage.
The MOST likely diagnosis for this patient would be:

Choices:
1. Costochondritis.
2. Intercostal muscle strain.
3. Abdominal muscle strain.
4. Systemic disease.

Teaching Points
Correct Answer: 4
This history is an insidious onset, with no injury and no pain with activity. There is no reproduction of
symptoms with the physical exam. This is typically a red flag for a systemic condition that should be re-
ferred to a medical doctor.

Incorrect Choices:
Most musculoskeletal conditions have a mechanism of injury or repetitive stress that initially causes the
symptoms. They are also activity related, so certain positions, activities, or movements aggravate or relieve
the symptoms.

Type of Reasoning: Analysis


This question requires the test taker to analyze symptoms in order to determine the most likely cause for
those symptoms. This requires analytical reasoning skill. For this case, the symptoms most likely indicate
systemic disease, and the patient should be referred to a medical doctor. Review assessment of rib cage
pain if answered incorrectly.
Examination C 857

System Interactions I Evaluation, Diagnos is

An elderly male patient recovering from a fractured hip repaired with ORIF has recently been discharged
home. During a home visit, his wife tells the therapist that he woke up yesterday morning and told her he
couldn't remember much. Upon examination, the therapist finds some mild motor loss in his right hand
and anomia. The therapist affirms the presence of short-term memory loss. The BEST course of action is to:

Choices:
1. Refer him to his physician, because the therapist suspects Alzheimer's dementia.
2. Refer him to his physician immediately, because the therapist suspects a stroke.
3. Advise the family to document and record any new problems that they notice over the next week and
then report back to the therapist.
4. Ignore the findings, because they are expected after surgical anesthesia.

Teaching Points

Correct Answer: 2
The presence offocal signs (incoordination, anomia) with cognitive signs (memory loss) is indicative of
impaired brain function and may be the result of small strokes. This is the most likely choice given the
spotty symptoms he presents with as well as their sudden onset.

Incorrect Choices:
Senile dementia, Alzheimer's type, can include some of the same symptoms but the onset is gradual and
the course is typically slowly progressive. The reporting of these findings to the primary physician should
not be delayed. Further diagnostic workup is indicated. Hospital admission and anesthesia can cause tem-
porary cognitive difficulties (delirium), but these should not persist with discharge home.

Type of Reasoning: Evaluation


In this question, the test taker must determine the BEST course of action, given the nature of the patient's
new symptoms. This requires evaluative reasoning skill, in which one evaluates the merits of each possi-
ble course of action in order to arrive at the correct conclusion. If this question was answered incorrectly,
review symptoms of transient ischemic attack and stroke.

Metabolic/Endocrine I Interventions

An adolescent with a 4-year history of type 2 diabetes is insulin dependent and wants to participate in
cross-country running. The PT working with the school team advises the athlete to measure plasma glucose
concentrations before and after running. In addition, the student should:

Choices:
1. Consume a carbohydrate after practice to avoid hyperglycemia.
2. Increase insulin dosage immediately before running.
3. Consume a carbohydrate before or during practice to avoid hypoglycemia.
4. Avoid carbohydrate-rich snacks within 12 hours of a race.
858 Examination C

Teaching Points

Correct Answer: 3
During exercise of increasing intensity and duration, plasma concentrations of insulin progressively
decrease. Exercise-induced hypoglycemia (abnormally low levels of glucose in the blood) is common for
exercising athletes with diabetes. Hypoglycemia associated with exercise can occur up to 48 hours after
exercise. To counteract these effects, the individual may need to reduce his insulin dosage or increase
carbohydrate intake before or after running. Consuming a carbohydrate product before or during the race
will have a preventive modulating effect on hypoglycemia.

Incorrect Choices:
Hyperglycemia (abnormally high levels of glucose in the blood) is more a risk for individuals with un-
controlled type 1 diabetes. These patients should demonstrate glycemic control before starting an exercise
program. Consuming carbohydrates will not lessen the likelihood of hyperglycemia.

Type of Reasoning: Evaluation


One must evaluate the merits of each of the four possible courses of action in order to arrive at the correct
solution. This requires knowledge of the effects of exercise on glucose concentrations in athletes with dia-
betes. Through evaluative reasoning, one should conclude that the athlete should consume a carbohydrate
snack before or during practice or a race to avoid hypoglycemia. If this question was answered incorrectly,
review guidelines for exercise with athletes with diabetes and carbohydrate consumption.

Genitourinary I Evaluation, Diagnosis

A male patient is referred to outpatient physical therapy for lower back pain. During the patient interview,
he describes a recent increased difficulty with urinating that does not affect his lower back pain symptoms.
Neurological screening examination is normal and Murphy's sign is negative. Based on this clinical scenar-
io, the patient's lower back pain symptoms may be associated with the:

Choices:
1. Kidney.
2. Lower urinary tract.
3. Sacro-iliac joint.
4. First lumbar nerve root.

Teaching Points

Correct Answer: 2
The lower urinary tract can refer symptoms to the lumbar spine region, and the recent report of increased
difficulty with urination is a key factor.

Incorrect Choices:
A negative Murphy's sign (percussion in the costovertebral area) decreases the likelihood of kidney symp-
toms. There was no information provided to specifically indicate the sacro-iliac joint as a source of current
symptoms; moreover, the recent reports of increased difficulty with urination may indicate a potential
systemic problem that may require immediate medical attention. Negative neurological findings would
rule out the first lumbar nerve root as a source of symptoms.
Examination C 859

Type of Reasoning: Analysis


This question requires the tester to critically evaluate key findings from the patient interview and deter-
mine the most likely differential diagnosis based on the information provided. This requires analytical
reasoning skill, where symptoms are analyzed to draw reasonable conclusions about the diagnosis. For
this scenario, the symptoms are consistent with the lower urinary tract. Review signs and symptoms of
lower urinary tract problems if answered incorrectly.

Neuro m uscula r I In terven t io n s

A PTA is ambulating a patient with a stroke using a walker. The patient is unsteady and fearful of falling.
The patient does not appear to understand the correct gait sequence. The supervising therapist's BEST strat-
egy is to:

Choices:
1. Instruct the PTA to have the patient sit down and utilize mental practice of the task.
2. Tell the PTA and patient to stop the ambulation and work on dynamic balance activities instead.
3. Instruct the PTA to use a distributed practice schedule to ensure patient success.
4. Intervene and teach the correct sequence because the PTA is apparently unable to deal with this special
situation.

Teaching Points
Correct Answer: 1
Mental rehearsal (mental practice) is the best strategy to have the patient learn the correct sequence. In
the non-weight-bearing position, the patient's anxiety is lessened, leaving the patient free to concentrate
on the task at hand.

Incorrect Choices:
The PT should provide appropriate guidance to the PTA but not necessarily take over care. Lack of under-
standing about the gait sequence rather than balance difficulties seems to be the major problem. Distrib-
uted practice with long rest times does not address the main difficulty.

Type of Reasoning: Evaluation


This question requires one to evaluate the value of each of the four possible solutions and then determine
which solution is the BEST strategy, given the patient's challenges. Evaluative reasoning questions often
require the test taker to make value judgments, which can be challenging for many test takers.

Musc u loskelet al I Ex am inat ion

A therapist is reviewing x-rays from a patient with a trimalleolar fracture. The BEST radiographic views to
visualize this bony anomaly are:

Choices:
1. PA and lateral.
2. Lateral and coronal.
3. Anteroposterior and lateral.
4. Oblique and lateral.
860 Examination C

Teaching Points

Correct Answer: 3
A trimalleolar fracture includes fracture of both malleoli and the posterior rim of the tibia. The anteropos-
terior view of the ankle demonstrates the distal tibia and fibula, including the medial and lateral malleoli
and the head of the talus. The fractures of both malleoli will be visible with this view. The lateral view
provides evidence of the fracture at the posterior rim of the distal tibia.

Incorrect Choices:
An oblique view of the foot demonstrates the phalanges, the metatarsals, and the intermetatarsal joints.
PA view is not routine for the ankle. A coronal view is also not indicated for this type of fracture.

Type of Reasoning: Analysis


One must recall the nature of a trimalleolar fracture in order to make a determination of the BEST radio-
graphic view for observation of this anomaly. This necessitates analytical reasoning skill, in which one
must interpret the information presented in order to make a determination of a BEST course of action.
If this question was answered incorrectly, review radiographic examination guidelines for trimalleolar
fractures.

C177
Nonsystem I Equipment, Devices, Therapeutic Modalities

A patient presents with pain and muscle spasm of the upper back (C7 - T8) extending to the lateral border
of the scapula. This encompasses a 10 x lO-cm area on both sides of the spine. If the US unit only has a
5-cm 2 sound head, the therapist should treat:

Choices:
1. The entire area in 5 minutes.
2. The entire area in 10 minutes.
3. Each side, allotting 2.5 minutes for each section.
4. Each side, allotting 5 minutes for each section.

Teaching Points
Correct Answer: 4
The total treatment area is too large for the 5-cm2 sound head to produce adequate tissue heating. Sonat-
ing the two areas independently will allow more time for the tissue temperature to rise during the treat-
ment time in each area.

Incorrect Choices:
Moving the transducer too fast to cover both sides adequately in the allotted time does not allow sufficient
time for the acoustic energy to produce heat because the head is not in a given area long enough. Increas-
ing the treatment time will not affect the rate of heat production. Two minutes is too brief to produce
sufficient tissue heating.

Type of Reasoning: Inductive


This question requires clinical judgment and diagnostic reasoning in order to determine the best approach
when providing US treatment for this patient. This requires recall of proper US treatment guidelines, in-
cluding consideration of the size of the sound head and size of the area to be treated. If this question was
answered incorrectly, review US treatment guidelines for larger treatment areas.

Examination C 861

Musculoskeletal I Examination

The physical therapist is examining the muscle length of the patient's right hip. The muscle length test
pictured below is the:

Choices:
l. Ober test.
2. Thomas test.
3. Noble test.
4. FABER test.

Teaching Points

Correct Answer: 1
The Ober test is for tightness of the tensor fascia latae and iliotibial band. With the pelvis stabilized and
the hip kept in neutral rotation, the Ober test would be considered normal if the thigh drops slightly
below horizontal.

Incorrect Choices:
The Thomas test is utilized to test for hip flexor length and to distinguish between one joint and two joint
hip flexor tightness. The Noble test is for iliotibial band friction syndrome. The FABER test is hip flexion,
abduction, and external rotation and is to screen hip and SI pathology.

Type of Reasoning: Analysis


One must understand the tests described in the question in order to choose the correct one. Through ana-
lytical reasoning, the test taker must determine how each test is performed and for what purpose. If this
question was answered incorrectly, review information on special tests .

C179

N e uro muscular I Examination

In posturography testing using the Clinical Test for Sensory Integration in Balance (CfSIB), patients who
sway more or fall under conditions with the eyes closed and platform moving (condition 5) or with the
visual surround moving and platform moving (condition 6) are likely to demonstrate:

Choices:
1. Problems with sensory selection.
2. Vestibular deficiency.
3. Somatosensory dependency.
4. Visual dependency.
862 Examination C

Teaching Points

Correct Answer: 2
The crSIB is positive for vestibular deficiency with loss of balance on conditions 5 and 6.

Incorrect Choices:
Patients who are surface dependent (somatosensory) have difficulties with conditions 4,5, and 6. Patients
who are visually dependent have difficulties with conditions 2, 3, and 6. Sensory selection problems are
evident with loss of balance on conditions 3-6.

Type of Reasoning: Inference


One must first determine what conditions 5 and 6 are indicative of in crSIB testing in order to arrive at
the correct conclusion. This necessitates inferential reasoning, in which one must draw conclusions from
the information presented in order to determine what the patient is likely to demonstrate. If this question
was answered incorrectly, refer to posturography testing.

"
Neuromuscular I Examination

A patient presents with severe, frequent seizures originating in the medial temporal lobes. After bilateral
surgical removal of these areas, the patient is unable to remember any new information just prior to the
surgery to the present. The patient cannot recall text read minutes ago or remember people previously met.
These outcomes are indicative of:

Choices:
1. Loss of the hippocampus and declarative memory function .
2. Loss of procedural memory and integration with frontal cortex.
3. A primary deficit from the loss of the amygdala.
4. Loss of integration of the temporal lobe with the basal ganglia and frontal cortex.

Teaching Points
Correct Answer: 1
Declarative memory refers to conscious, explicit, or cognitive memory. It is a function of the cerebral cor-
tex and the hippocampus.

Incorrect Choices:
Procedural memory (unconscious memory or implicit memory) refers to the recall of skills and habits
and emotional responses. It is the result of integrated action of the frontal cortex (neocortex), thalamus,
and striatum of the basal ganglia. The amygdala is a collection of nuclei in the anteromedial temporal
lobe, forming the core of the limbic circuits. It is important for triggering feelings and drive-related behav-
iors.

Type of Reasoning: Analysis


This question requires the test taker to determine the diagnosis based on symptoms presented, which is
an analytical reasoning skill. In this situation, the symptoms are indicative of loss of the hippocampus
and declarative memory function . Knowledge of neuroscience and the cortical structures responsible for
declarative memory function are beneficial to arriving at the correct conclusion. If this question was an-
swered incorrectly, refer to memory functions and structures.
Examination C 863

Metabolic/Endocrine I Evaluation, Diagnosis

A 14-year-old with a body mass index of 33 kg/ m2 and a history oflimited participation in physical activi-
ties is referred for exercise training. The nutritionist has prescribed a diet limiting his caloric intake. The
BEST initial exercise prescription is:

Choices:
l. Three weekly sessions of 50 minutes at 70-85% V0 2max .
2. Three weekly sessions of 30 minutes at 60-70% V0 2max .
3. Three weekly sessions of 60 minutes at 50% V0 2max '
4. Two daily sessions of 30 minutes at 40- 70% V0 2Il1 ax .

Teaching Points

Correct Answer: 4
This individual is obese (body mass index:2: 30 kg/m2) and will benefit from exercise to increase energy
expenditure and diet to reduce caloric intake. The initial exercise prescription should utilize low-intensity
with longer duration exercise. Splitting the training into two sessions each day is a good choice. The goal
is to work toward bringing the target HR into a suitable range. Obese individuals are at increased risk of
orthopedic injuries and require close monitoring.

Incorrect Choices:
Weekly sessions at high intensities (70-85% V0 2max) and long duration (50 min) are contraindicated.
Weekly sessions (two to three times/wk) are not as beneficial initially as are daily sessions at shorter dura-
tions and moderate intensities.

Type of Reasoning: Inductive


One must utilize clinical judgment and diagnostic reasoning to determine the BEST initial exercise
prescription for this individual. This necessitates inductive reasoning skill, in which one must refer to
knowledge of exercise for individuals with obesity to arrive at the correct conclusion. If this question was
answered incorrectly, review exercise prescription guidelines for the obese .

C182

Metabolic/Endocrine I Evaluation, Diagnosis

A patient is admitted to a hospital after a fall . A review of the patient's medical chart reveals a blood pres-
sure (BP) of 160/85, a triglyceride level of 160 mg/dL, and a fasting blood glucose level of ll5 mg/ dL.
Weight is 310 lb. Examination of the patient reveals a rotund man with a 54-inch waistline. These findings
are indicative of:

Choices:
1. Chronic heart disease.
2. Type 1 diabetes.
3. Metabolic syndrome.
4. Cushing's syndrome.
864 Examination C

Teaching Points

Correct Answer: 3
This patient is exhibiting four of the risk factors of metabolic syndrome (diagnosis is made if three or
more are present). Risk factors include (1) abdominal obesity: waist circumference> 40 inches in men or
> 35 inches in women; (2) elevated triglycerides: triglyceride level of 150 mg/dL or higher; (3) low high-
density lipoprotein (HDL) cholesterol or being on medicine to treat low HDL: HDL level < 40 mg/dL
in men or 50 mg/dL in women; (4) elevated BP: systolic BP;::: 130 mm Hg and/or diastolic BP = 85 mm
Hg; and (5) fasting plasma glucose level > 100 mg/dL. The therapist's plan of care should be reflective of
the patient's increased risk for heart disease, stroke, and diabetes and should assist the patient in lifestyle
changes that reduce these risk factors.

Incorrect Choices:
No mention is made of absolute insulin deficiency (type 1 diabetes). Although these are risk factors for
heart disease, they do not specifically define or characterize chronic heart disease. Cushing's syndrome
(glucocorticoid hormone excess) refers to the manifestations of hypercortisolism from any cause. Patients
typically exhibit a round "moon face" with a protruding abdomen or "buffalo hump" on the back along
with muscle weakness and wasting.

Type of Reasoning: Analysis


This question requires one to analyze all of the signs and symptoms presented and then draw a conclu-
sion about a potential diagnosis. Questions of this nature require analytical reasoning skills. If this ques-
tion was answered incorrectly, refer to signs and symptoms of metabolic syndrome.

C183
System Interactions I Evaluation, Diagnosis

A patient in an exercise class develops muscle weakness and fatigue. Examination reveals leg cramps and
hyporeflexia. The patient also experiences frequent episodes of postural hypotension and dizziness. Abnor-
malities on the ECC include a flat T wave, prolonged QT interval, and depressed ST segment. These findings
are suggestive of:

Choices:
1. Hyperkalemia.
2. Hypocalcemia.
3. Hyponatremia.
4. Hypokalemia.

Teaching Points

Correct Answer: 4
Hypokalemia, decreased potassium in the blood, is characterized by these signs and symptoms. Other
possible symptoms include respiratory distress, irritability, confusion or depression, and gastrointestinal
disturbances.

Incorrect Choices:
Hyperkalemia is excess potassium in the blood. Hyponatremia is decreased sodium in the blood, and hy-
pocalcemia is decreased calcium in the blood. These conditions cannot produce this battery of symptoms.
Exam ination C 865

Type of Reasoning: Analysis


This question requires the test taker to determine the diagnosis based on the symptoms described, which
is an analytical reasoning skill. The test taker must analyze the symptoms in order to determine the most
likely cause for them. If this question was answered incorrectly, refer to signs and symptoms of the condi-
tions listed (Chapter 6) .


Cardiovascular/Pu lmonary and Lymphatic I Examination

A patient has experienced swelling in both lower legs (below the knees) since the age of 16 (lO-year his-
tory). The referring diagnosis is bilateral lymphedema. The therapist's initial examination should include:

Choices:
1. Systolic blood pressure before and after exercise.
2. Rubor of Dependency test.
3. Ankle-brachial index (ABI).
4. Homan's sign of venous insufficiency.

Teaching Points
Correct Answer: 3
The ABI is helpful to determine the arterial patency or sufficiency. A value of 1.0 indicates normal arterial
flow; values between 0.5 and 0.8 indicates moderate compromise while values below 0.5 indicates severe
compromise of arterial flow. Adequcy of adequate arterial blood flow is essential in considering applica-
tion of compression for lymphedema.

Incorrect Choices:
Two tests (Rubor of Dependency test, systolic BP) are used in the examination of periperal artery disease.
Homan's sign was used in the examination of deep vein thrombophlebitis (DVT) though is no longer
considered a reliable indicator of DVT.

Type of Reasoning: Analysis


One must understand the tests described in the question in order to choose the correct one. Through ana-
lytical reasoning, the test taker must determine how each test is performed and for what purpose. If this
question was answered incorrectly, review information on special tests .


Genitou r inary I Interventions

A patient is referred to a woman's health specialist PT with a diagnosis of pelvic pain and uterine prolapse.
Physical therapy intervention should focus on:

Choices:
1. Protective splinting of abdominal musculature.
2. External stabilization with a support belt.
3. Gentle abdominal exercises with incisional support.
4. Kegel's exercises to strengthen pubococcygeal muscles.
866 Examination C

Teaching Points
Correct Answer: 4
Intervention should focus on pelvic floor rehabilitation (Kegel's exercises to strengthen pubococcygeal
muscles) along with postural education and muscle reeducation.

Incorrect Choices:
Diastasis recti abdominis requires protective splinting of abdominal musculature during initial separation
(> 2 cm split of the rectus abdominis after pregnancy) . External stabilization with a support belt may be
required for sacroiliac dysfunction. Postcesarean interventions include gentle abdominal exercises with
incisional support. There was no mention of pregnancy or postpregnancy complications.

Type of Reasoning: Inference


One must infer or draw a reasonable conclusion about the best intervention approach for a patient with
pelvic pain and uterine prolapse. This requires inferential reasoning skills because one must determine
which therapeutic approach will have the most effective outcome of improving function . If this question
was answered incorrectly, review exercise guidelines for patients with uterine prolapse.


Neuromuscular I Interventions

A therapist is working with a patient recovering from traumatic brain injury, Rancho Los Amigos Levels of
Cognitive Functioning (LOCF), level IV. This is the patient's first time in the physical therapy gym. While
the therapist attempts to work with the patient on sitting control, the patient becomes agitated and combat-
ive. Which strategy is the BEST choice in this situation?

Choices:
1. Enlist the help of a PTA to support the patient while slowly instructing the patient again.
2. Remove the patient to a quiet environment and provide support and calming stimuli.
3. Return the patient to his/ her room and try the training activity later in the day.
4. Give the patient a 5-minute rest and resume the training activity.

Teaching Points
Correct Answer: 2
The best choice is to remove the patient to a quiet room and provide calming stimuli and support. At
level IV LOCF, the therapist can expect the patient to be confused, inappropriate, and highly distractable.
The environment should be changed. Memory is severely impaired and the patient is unable to learn new
information.

Incorrect Choices:
Enlisting the PTA or resuming training after a 5-minute rest does not change the environment nor calm
the patient. Rescheduling for the afternoon similarly does not address the central problem of the patient's
level IV behaviors.
Examination C 867

Type of Reasoning: Inference


Questions that inquire about a BEST approach to a therapeutic situation often necessitate inferential rea-
soning skill. In this situation, the test taker must determine the BEST choice for a patient with a traumatic
brain injury and Rancho Los Amigos Levels of Cognitive Functioning, level IV behaviors. Review Levels of
Cognitive Function after traumatic brain injury and beneficial training strategies for each level.


Neuromuscular I Interventions

A patient has a lO-year history of Parkinson's disease and has been on levodopa (Carbidopa) for the past
6 years. The patient has fallen three times in the past month, resulting in a Colles' fracture. The therapist
decides to try postural biofeedback training using a platform balance training device. The training sequence
should focus on:

Choices:
1. Decreasing the limits of stability and improving increasing anterior weight displacement.
2. Increasing the limits of stability and improving increasing anterior weight displacement.
3. Decreasing the limits of stability and improving increasing posterior weight displacement.
4. Increasing the limits of stability and improving center of pressure alignment.

Teaching Points

Correct Answer: 4
The patient with Parkinson's disease exhibits significant balance impairments including loss of pos-
tural reflexes; decreased limits of stability; flexed, stooped posture that alters the center of pressure in an
anterior direction; freezing; and orthostatic hypotension. Platform balance training should work toward
improving the limits of stability and center of pressure alignment (the patient should focus on reducing
anterior displacement).

Incorrect Choices:
The patient is too unstable; decreasing limits of stability is contraindicated, as is promoting anterior
weight displacement.

Type of Reasoning: Inference


One must infer the most appropriate training sequence for biofeedback postural training in order to
arrive at the correct conclusion. This involves inferential reasoning, in which one must consider the
patient's diagnosis and current limitations in order to choose the most appropriate training sequence. If
this question was answered incorrectly, refer to balance deficits and training strategies for patients with
Parkinson's disease.

868 Examination C

Cardiovascular/Pulmonary and Lymphatic I Evaluation, Diagnosis

A patient is recovering from MI and is referred for supervised exercise training. While working out on a
treadmill, the patient begins to develop mild shortness of breath. Upon inspection of the ECG readout, the
therapist determines the presence of:

From: Jones. S (2005) ECG Notes. Philadelphia. F.A. Davis. p. 162. with permission.

Choices:
1. Sinus rhythm with upsloping ST segment depression.
2. Sinus rhythm with sinoatrial blocks.
3. Sinus rhythm with downsloping ST segment depression.
4. Tachycardia with abnormal P waves.

Teaching Points

Correct Answer: 1
This patient is exhibiting sinus rhythm with upsloping ST segment depression. An abnormal ECG re-
sponse is defined as ~1 mm of horizontal or downsloping depression at 80 msec beyond the J point
(ACSM Guidelines for Exercise Testing and Prescription, 8 th ed. Philadelphia: Lippincott Williams &< Wilkins) .
Associated clinical signs suggestive of myocardial ischemia include dyspnea and angina. After a review
of this patient's exercise responses coupled with the ECG findings, the therapist correctly determines that
the exercise session does not have to be terminated. The therapist should continue to closely monitor the
patient's responses.

Incorrect Choices:
This tracing does NOT indicate the other conditions listed (sinoatrial block downsloping ST segment
depression, or tachycardia with abnormal P waves).

Type of Reasoning: Analysis


Questions that require one to analyze visual information depicted in pictures, charts, etc., necessitate
analytical reasoning skill. This question requires the test taker to appropriately interpret the ECG readout
in order to determine the correct diagnosis. If this question was answered incorrectly, review ECG interpre-
tation guidelines.
Exami nation C 869

Musculoskeletal I Evaluation, Diagnosis

A female patient complains of right lumbosacral pain after giving natural childbirth to her first child 2
months ago . Pain has subsided somewhat, but remains high enough that she has to sit after walking more
than 2 blocks. Pain is noted in the right lumbosacral region, buttock, and groin and is aggravated with
weight bearing on the right. Active flexion, extension, and side bending reproduce the patient's symptoms.
Hamstrings are slightly tight on the right, but no neural tension is noted. Neurological findings (reflexes,
sensation, and motor) are unremarkable. SI provocation tests are positive. The MOST likely diagnosis for
this patient would be:

Choices:
1. Piriformis syndrome.
2. Sacroiliac sprain.
3. Lumbar disc protrusion at LS/Sl.
4. Quadratus lumborum strain.

Teaching Points

Correct Answer: 2
Child-bearing can place considerable stress on the sacroiliac joint, causing an overstretch of the ligaments.
Pain is reproduced with active movement testing, which is characteristic of a musculoskeletal condition.
Sacroiliac provocation tests are fairly valid as well, thus implicating the sacroiliac joint. The groin and but-
tock are common referral regions for the sacroiliac joint.

Incorrect Choices:
There is no neural tension or neurological signs that help to rule out disc dysfunction. There are no find-
ings indicating specific strain of the piriformis or quadratus lumborum as SI provocation is positive.

Type of Reasoning: Analysis


For this question, the test taker must analyze the symptoms present and determine the most likely diag-
nosis. Questions of this nature often require analytical reasoning skill. For this scenario, given the symp-
toms presented, the most likely diagnosis is sacroiliac sprain. If answered incorrectly, review signs and
symptoms of sacroiliac sprain/strain.

C190
Musculoskeletal I Evaluation, Diagnosis

A patient presents with a complaint of severe neck and shoulder pain of 2 days' duration. The patient
reports falling asleep on the couch watching 1V and has been stiff and sore since. There is tenderness of the
cervical muscles on the right, with increased pain upon palpation. Passive ROM is most limited in flexion,
then side-bending left, and then rotation left, and active extension. Side-bending right and rotation right
are also painful. Based on these examination findings, the patient's diagnosis is:

Choices:
1. Cervical radiculopathy.
2. Facet syndrome.
3. Cervical strain .
4. Herniated disc.
870 Examination C

Teaching Points

Correct Answer: 2
A facet syndrome presents with localized pain.

Incorrect Choices:
Cervical radiculopathy presents with arm pain in the dermatomal distribution and increased pain by
extension and rotation or side flexion. Cervical strain presents with pain on activity or when the muscle is
on stretch. Cervical disc herniation has a dermatomal pain distribution with an increase of pain on exten-
sion, and pain on flexion may either increase or decrease (most common).

Type of Reasoning: Analysis


This question requires the test taker to determine the diagnosis based on symptoms presented, which is an
analytical reasoning skill. For this patient, the symptoms are indicative of facet syndrome. Knowledge of
spinal disorders is required for arriving at the correct conclusion. If this question was answered incorrectly,
refer to symptoms of facet syndrome.

Musculoskeletal I Interventions

A postal worker ( mail sorter) complains of numbness and tingling in the right hand. Examination reveals
a median nerve distribution. When the therapist evaluates the patient's work tasks, the therapist notes that
the patient is required to key in the zip codes of about 58 letters per minute. An appropriate administrative
control to decrease exposure would be to:

Choices:
1. Provide the worker with a resting splint to support the wrist.
2. Provide a height adjustable chair to position the wrists and hands in a neutral alignment.
3. Use job rotation during the workday.
4. Require the worker to attend a cumulative trauma disorder educational class.

Teaching Points

Correct Answer: 3
Administrative controls reduce the duration, frequency, and severity of exposures to ergonomic stressors.
Job rotation reduces fatigue and stress by rotating the worker to jobs that use different muscle-tendon
groups during the workday.

Incorrect Choices:
The other choices represent either clinicaL engineering, or educational interventions and do not address
the essential question of an administrative control.

Type of Reasoning: Deductive


This question requires one to recall the parameters of an administrative control in ergonomic assessment.
This is factual recall of ergonomic knowledge, which encourages the use of deductive reasoning skill. If
this question was answered incorrectly, review administrative controls in ergonomic assessment.
Examination C 871

Nonsystem I Equipment, Devices , Therapeut ic Modalities

A patient with a traumatic brain injury presents with hemiparesis. The examination reveals slight cutane-
ous and proprioceptive impairment, fair (3/5) strength of the shoulder muscles and triceps, and slight
spasticity of the biceps. Voluntary control of the patient's left arm has not progressed since admission. The
therapist decides to use FES and place the active electrode on the triceps to facilitate active extension of the
elbow. The tim ing sequence BEST to apply is:

Choices:
1. 2-second ramp up, 5-second stimulation, 2-second ramp down.
2. 5-second ramp up, 5-second stimulation, 5-second ramp down .
3. 2-second ramp up, lO-second stimulation, no ramp down.
4. No ramp up, lO-second stimulation, 2-second ramp down.

Teaching Points

Correct Answer: 2
A relatively long ramp up time over a 5-second period is used to minimize stimulating the muscle too
quickly and increasing the spasticity. The ramp down time has no effect on spasticity.

Incorrect Choices:
The other ch oices have too short or no ramp up time, and this could increase the spasticity of the biceps.

Type of Reasoning: Inductive


This question requ ires one to predict the possible outcome of what will happen when each of the four
possible approaches is applied. Via clinical judgment and the process of prediction, an inductive reason-
ing skill, one should determine that a 5-second ramp up, 5-second stimulation, and 5-second ramp down
will be MOST beneficial for this patient. If this qu estion was answered incorrectly, refer to guidelines for
FES for spastic upper extremities .

C193

Neuromuscular I Interventions

After a traumatic brain injury, a patient presents with significant difficulties in learning how to use a wheel-
chair. Memory for new learning is present but limited (Rancho Los Amigos Levels of Cognitive Functioning,
level VII). The patient is wheelchair dependent and needs to learn how to transfer from the wheelchair to
the mat (a skill never done before). The BEST strategy to enhance motor learning is to:

Choices:
1. Provide consistent feedback using a blocked practice schedule.
2. Use only guided movement to ensure correct performance.
3. Provide summed feedback after evelY few trials using a serial practice schedule.
4. Provide bandwidth feedback using a random practice schedule.
872 Examination C

Teaching Points

Correct Answer: 1
Early learning should focus on consistent feedback given after every trial to improve initial performance. A
blocked practice schedule with repeated practice of the same skill will also reinforce early learning.

Incorrect Choices:
Variable feedback schedules (summed or bandwidth) and variable practice schedules (serial and random)
are indicated for later learning to improve retention. Using only guided movement is contraindicated in
this case because it minimizes active participation and active learning.

Type of Reasoning: Inference


One must infer the BEST strategy to enhance this patient's motor learning for transfer skills. Given the
patient's current functional level, it is BEST to provide consistent feedback using a blocked practice sched-
ule to reinforce early learning. Questions that require one to draw conclusions and infer strategies often
utilize inferential reasoning skills. If this question was answered incorrectly, refer to strategies to enhance
early learning after a traumatic brain injury.

C194

Neuromuscular I Evaluation, Diagnosis

A patient recovering from stroke presents with predominant involvement of the contralateral lower extrem-
ity and lesser involvement of the contralateral upper extremity. The patient also demonstrates mild apraxia.
These clinical manifestations are characteristic of:

Choices:
1. Middle cerebral artery syndrome.
2. Anterior cerebral artery syndrome.
3. Posterior cerebral artery syndrome.
4. Basilar artery syndrome.

Teaching Points

Correct Answer: 2
These clinical manifestations are consistent with anterior cerebral artery syndrome (lower extremity is
more involved than upper extremity).

Incorrect Choices:
Patients with middle cerebral artery syndrome demonstrate the opposite findings, greater involvement of
the upper than the lower extremity. Patients with posterior cerebral artery syndrome demonstrate primary
involvement of the visual cortex (contralateral homonymous hemianopsia) along with dyslexia (difficulty
reading), prosopagnosia (difficulty naming people on sight), and memory defect (temporal lobe lesion).
Patients with basilar artery syndrome demonstrate a combination of brainstem syndromes along with
signs of posterior cerebral artery syndrome.

Type of Reasoning: Analysis


This question requires the test taker to determine the location of the patient's CVA based on symptoms
presented, which is an analytical reasoning skill. Knowledge of neuroscience and the various arterial syn-
dromes related to CVA are beneficial for arriving at the correct conclusion. If this question was answered
incorrectly, refer to symptoms of cerebral artery syndromes.

Examination C 873

Cardiovascular/Pulmonary and Lymphatic I Evaluation, Diagnosis

A patient presents with significant intermittent claudication with onset after 2 minutes of walking. On
further examination, the therapist would expect to find:

Choices:
1. A brownish color just above the ankle in both gravity-dependent and -independent positions.
2. Little or no changes in color with changes in extremity position.
3. Persistent local redness of the extremity in both gravity-dependent and -independent positions.
4. Elevation-induced pallor and dependent redness with the extremity in the gravity-dependent position.

Teaching Points

Correct Answer: 4
Intermittent claudication (episodic muscular ischemia induced by exercise) is due to obstruction oflarge-
or middle-sized arteries by atherosclerosis. The Rubor of Dependency test is used to assess the adequacy
of arterial circulation by evaluating the skin color changes that occur with first extremity elevation (pal-
lor) and then lowering of the extremities (delayed color changes, redness) .

Incorrect Choices:
Color changes are expected with change of position. A brownish color just above the ankle suggests
chronic venous insufficiency. Persistent local redness is indicative of a thrombosed vein in the area.

Type of Reasoning: Inference


One must infer the likely findings for this patient, given that the patient's symptoms are in response to
walking. It is beneficial to have prior knowledge of the Rubor of Dependency test in order to arrive at the
correct conclusion. Through inferential reasoning, one should conclude that the patient would demon-
strate elevation-induced pallor and dependent redness with the extremity in a gravity dependent position.
Ifthis question was answered incorrectly, review examination of the patient with PVD.

C196
Genitourinary I Evaluation, Diagnosis

After her cesarean section, a patient tells the therapist that she is anxious to return to her prepregnancy level
of physical activity (working out at the gym three days/wk and running 5 miles every other day). The thera-
pist's BEST advice is to tell her to resume activities with:

Choices:
1. Pelvic floor exercises and refrain from all other exercise and running for at least 12 weeks.
2. Pelvic floor and gentle abdominal exercises for the first 4-6 weeks.
3. Abdominal crunches with return to running after 5 weeks.
4. A walking program progressing to running after 4 weeks.
874 Examination C

Teaching Points

Correct Answer: 2
Postcesarean physical therapy can include postoperative TENS, assisted breathing and coughing tech-
niques, and gentle abdominal exercises with incisional support provided by a pillow. Pelvic floor exercises
are also important because hours of labor and pushing are typically present before surgery.

Incorrect Choices:
Vigorous exercise (abdominal crunches, running) is contraindicated for at least 6 weeks. Without compli-
cations, 12 weeks is too long to refrain from exercises and running.

Type of Reasoning: Evaluation


This question requires one to evaluate the merits of each of the four statements and then determine which
response to the patient is BEST given her recent cesarean section. Evaluative reasoning skills are utilized
whenever one must make value judgments and assess the worth of ideas presented. If this question was
answered incorrectly, refer to guidelines on exercise for women postpartum .


Syste m Int eract ions I Evaluati on , D iagnosis

A 26-year-old who was diagnosed with schizophrenia, disorganized type, at the age of 22 is referred for gait
training after a compound fracture of the tibia. The individual recently experienced an exacerbation of the
condition. The therapist recognizes this from the patient's demonstrated behaviors, which include:

Choices:
1. Sleep disturbances and flashbacks .
2. Increased fear of going out in public.
3. Frequent verbalizations of pervasive feelings oflow self-esteem.
4. Poor ability to perform multistep tasks requiring abstract problem-solving.

Teaching Points

Correct Answer: 4
Schizophrenia is characterized by disordered thinking (fragmented thoughts, errors oflogic or abstract
reasoning, delusions, poor judgment, and so forth).

Incorrect Choices:
Sleep disturbances and flashbacks are common with posttraumatic stress disorder. Increased fear of going
out in public is agoraphobia. Pervasive feelings of low self-esteem can accompany depression or anxiety
disorder.

Type of Reasoning: Inference


This question requires one to infer or draw a conclusion about the likely symptoms of disorganized
schizophrenia. This necessitates inferential reasoning skills. For this situation, the therapist should expect
behaviors of poor ability to perform multistep tasks requiring abstract problem-solving. If this question
was answered incorrectly, review symptoms of schizophrenia, especially disorganized type.

Exa mination C 875

Nonsystem I Equipment, Devices, Therapeutic Modalities

The rehabilitation team is completing a home visit to recommend environmental modifications for a pa-
tient who is scheduled to be discharged next week. The patient is wheelchair dependent. The home h as not
been adapted. Which of the following recommendations is correct?

Choices:
1. Installing an entry way ramp with a running slope of 1:10.
2 . Widening the door entrance to 28 inches.
3. Adding horizontal grab bars in the bathroom positioned at 34 inches.
4. Raising the toilet seat to 25 inches.

Teaching Points

Correct Answer: 3
Horizontal grab bars should be positioned at an optimal height of 33-36 inches.

Incorrect Choices:
The minimum ramp grade (slope) is 1:12 (not 1:10, which would be too steep for functional use). The
toilet seat should be raised to a height of 17-19 inches (not 25). Minimum clearance width for doorways
is 32 inches (not 28); 36 inches is ideal.

Type of Reasoning: Inference


This question requires one to determine appropriate recommendations for patients utilizing wheelchairs
for mobility in the home. This necessitates recall of factual knowledge and clinical judgment, which is an
inferential reasoning skill. If this question was answered incorrectly, review guidelines for environmental
(home) modifications for patients utilizing wheelchairs for mobility.


Musculoskeletal I Examination

During an examination of gait, the therapist observes lateral pelvic tilt on the side of the swing leg during
frontal plane analysis. The therapist recognizes this finding functions to:

Choices:
1. Control forward and backward rotations of the pelvis.
2. Reduce peak rise of the pelvis.
3. Reduce physiological valgum at the knee.
4. Reduce knee flexion at mid stance.

Teaching Points

Correct Answer: 2
Lateral pelvic tilt in the frontal plane keeps the peak of the sinusoidal curve lower than it would have been
if the pelvis did not drop. Lateral pelvic tilt to the right is controlled by the left hip abductors.
876 Examination C

Incorrect Choices:
Forward and backward rotations of the pelvis assist the swing leg. The normal physiologic valgum at the
knee reduces the width of the base of support. Knee flexion at midstance is another adjustment in keeping
the center of gravity from rising too much. All are termed determinants of gait.

Type of Reasoning: Inference


One must infer the purpose of the lateral pelvic tilt on the side of the swing leg during gait in order to
arrive at the correct conclusion. This necessitates inferential reasoning skills, where one must draw con-
clusions based on the evidence presented. If this question was answered incorrectly, review gait analysis
guidelines .

C200

Nonsystem I Safety, Professional Responsibilities, Research

A group of researchers investigated the effect of tai chi on perceived health status in older, frail adults. The
subjects were 269 women who were older than 70 years of age and recruited from five independent senior
living facilities. Participants took part in a 48-week single-blind ReT. Perceived health status was measured
by five pretrained testers using the Sickness Impact Profile (SIP) . The researchers found significant perceived
health benefits. Analysis of the design reveals:

Choices:
1. Limited generalizability to a larger population.
2. Important findings on the effects of tai chi exercise in the frail elderly.
3. Errors in validity due to the selection of the outcome measure.
4. Errors in reliability due to the number of testers.

Teaching Points
Correct Answer: 2
This study provides important findings on the effects of tai chi in the elderly. It has a large number of
subjects from multiple centers and thus has good generalizability (not limited). The SIP is a gold standard
instrument with extensive testing and established validity and reliability.

Incorrect Choices:
Errors in reliability are not automatically inherent with multiple testers. Pretrial training reduces the likeli-
hood of errors in reliability. There is no error in validity because of the use of the SIP.

Type of Reasoning: Analysis


This question requires the test taker to analyze the design of the research study and then draw a conclu-
sion about this design. This necessitates analysis of many factors, which is an analytical reasoning skill.
For this scenario, the test taker should reasonably conclude that the study had good generalizability due
to the number of subjects in the study and multiple centers involved in the study and good validity and
reliability. If this question was answered incorrectly, review research design guidelines.

You might also like